多 岐 亡 羊 II

多 岐 亡 羊 II
Vol. 17
2013 年度西澤ゼミ学生論文集
同志社大学法学部・政治学科
2014 年 3 月
はしがき
中国の学者・楊子(ようし)が、分かれ道が多くて、逃げた羊を
見つけることができなかったことを深く悲しんだ。
「分かれ道が多い
ために羊を逃してしまったのと同じように、学問の道もあまりに多
方面にわたっているために、真理を見失ってしまうことを悲しんだ
のだ」と、後にその弟子の一人・心都子(しんとし)が説明したそ
うだ。
現代日本が抱える政治的な問題や日本あるいは外国の有権者の投
票行動・価値観について、多くの立場の研究に触れ、また、興味を
持った具体的な疑問点について、各自が独自な視点で分析を行った
わけだが、それには1年はあまりにも短かった。おかげで、政治の
メカニズムが理解できたというより、むしろよけいに分からなくな
ったというのが正直なところだろう。まさしく、
「多岐亡羊」である。
さて、このあたりで、「例年とは異なる 1 年だった」と、毎年、私
は書いているような気がする。ようするに、1 年として「同じよう
な年」はないというわけである。
「毎年異なること」が「例年」なの
だから、今年からは、
「例年どおり、今年も特別な 1 年だった」と書
くべきなのかもしれない。
いずれにしても、同志社国際学院の校長を兼務することになった
今年は、私は、
「超多忙」であった。ゼミ生には迷惑をかけないよう
にと心がけたつもりではあるが、ゼミ生もずいぶんと気を遣ってく
れた。そんな中、このように『多岐亡羊』がまとまったことを嬉し
く思う。それぞれに責任を持って、ゼミ運営に主体的にあたってく
れた。学年を超えた結束力の強さを感じた。私が理想とするゼミで
ある。
ところで、大学で学ぶことの究極の目的は、
「真理とは何か(ある
いは人間とはどういう生き物なのか)
」という問いに対する答えを見
いだすことである。それにしても、大学の 4 年は短すぎる。今年度
も、
「真理を問う」ことのおもしろさを、君たちが少しは体験できた
と私は信じているが、4 年生の諸君は、卒業後もこの問いに対して
自問を続けることになるだろう。また、在学生は、残る時間を大切
にして、今年以上に勉学に励んでほしい。来年度のさらなる飛躍を
期待したい。
西澤由隆 2014 年 1 月 10 日
光塩館の研究室にて
i
もくじ
はしがき ······················································ 西澤由隆
i
2 年生論文
1. 候補者の立候補パターンは
有権者の志向をどのように規定するか ····· 片山雅貴
2. 政党に関する理解度と無党派層の増加 ········ 加藤理歩
1
10
-なぜ、1990 年代に無党派層は急増したのか-
3. なぜ、若者は投票に行かないのか·············· 豊島由梨佳 17
-政治的有効性感覚に注目して-
3 年生論文
4. どのような女性が
より高い政治関心を抱くのか ········· 小和田早紀 32
5. テレビの選挙報道が
投票行動に与える影響とは ············ 濱田茜
43
6. 投票結果が有権者に与える影響················· 山田紗生
54
4 年生論文
7. 組織と政治関心 ·································· 浦岡教之
73
-どのような組織が政治関心を高めるのか-
8. 低信頼層の投票行動 ······························· 浦田祐香
85
-なぜ投票するのか-
9. 異なるレベルの選挙における
選挙関心の形成要因················· 林香織
102
-「ライブラリー・ポリティクス」を題材に-
10. 電子投票に賛成する有権者とは················· 水田紗希子 145
-京都府京都市に着目して-
11. 未成年期の擬似的な
政治参加経験の影響力 ············· 山内逸平
-生徒会選挙・模擬選挙に着目して-
ii
157
2 年生論文
1
候補者の立候補パタ
ーンは有権者の志向
をどのように規定す
るか
片山 雅貴
1. はじめに
中選挙区制はその制度自身に大きく 2 つの問題を抱えていた。1 つは
得票と議席の均衡の問題であり、三宅一郎(1989)は不均衡の指標を示
し、
「・・・最大の欠陥は選挙区内での政党別得票率と議席率の不均衡
である。
」と指摘している(三宅 1989, 34)
。もう 1 つは「金」の問題
であり、
平成 2 年の選挙制度及び政治資金制度の改革についての答申
(以
下、答申と略す)には、
「最近における『政治と金』の問題に端を発し
て、政治に対する国民の不信がいまだかつてないほどに増大するに至っ
ており、また、そのために、
『政治改革』への要望が各方面において高
まってきている。
」とある。
中選挙区制のこのような問題を解消するため、選挙制度改革が実施さ
れ、小選挙区比例代表並立制を新制度として採用した。答申によると、
「中選挙区制の下で生じているこれらの問題は、制度の運用のみではも
はや改善し得ないものであり、政策本位、政党本位の選挙を実現するた
めには、現行選挙制度を根本的に改革する必要がある。
」とあり、選挙
での有権者の志向を政党に向けることを改革は期待されていた。
候補者に着目すると、選挙制度改革による制度の変化は候補者の立候
補パターンが変化したことにある。つまり選挙区に同じ政党の候補者が
複数立候補できなくなったことである。1993 年衆議院総選挙(以下、
1993 年選挙と略す)以前には選挙区に同じ政党の候補者が複数立候補
できたが、1996 年衆議院総選挙(以下、1996 年選挙と略す)以後には
複数立候補できなくなった。
選挙制度が有権者に影響を与えることは知られているが、候補者の立
候補パターンが変化したことは有権者に影響を与えているだろうか。答
申に期待されるように選挙を政策本位や政党本位にしただろうか。
1
選挙制度改革以前の中選挙区制では、有権者が候補者を相対的に重視
した投票をするとされている。三宅(1989)はその理由を 3 つ挙げ説明
している。1 つ目は、中選挙区制度では、大政党は複数の候補者を擁立
していることだ。1 2 つ目は、党員は代議士や派閥によって一括で組織
に加入するため、地域組織が弱いことだ。3 つ目は、候補者の所属政党
に注意を払わない有権者が増加していることだ。以上の理由から、有権
者は候補者を重視するとしている(三宅 1989, 39-41)
。
三宅の挙げた 1 つ目の理由と選挙区ごとの立候補パターンに私は着目
し、候補者の立候補パターンが有権者の志向に与える影響を確認したい。
よって本稿では、
「候補者の立候補パターンが有権者の志向をどのよう
に規定するか」というリサーチクエスチョンを設定する。そのため「候
補者を複数擁立する政党を支持する有権者の志向が、選挙制度改革によ
り変化する」という仮説を設定する。そして、選挙制度改革以前と以後
の変化の差を比較確認することで数量的に検証する。
2. 仮説
前述のとおり本稿では、候補者の立候補パターンが有権者の志向に影
響があるというモデルを想定している。そのため、1993 年選挙と 1996
年選挙の立候補パターンの変化が有権者の志向を変化させるという仮
説を述べる。
2-1. 仮説の想定
中選挙区制から小選挙区制に選挙制度が変わったことで、選挙区に同
じ政党の候補者が複数立候補できなくなった。
しかし選挙制度の変化による影響を受け、候補者の立候補パターンが
変わったと想定できる選挙区・想定できない選挙区がある。前年の選挙
で同じ政党から候補者が複数擁立されていた選挙区では、後年の選挙で
候補者が複数擁立されなくなったため、候補者の立候補パターンに変化
があったといえるだろう。一方で、前年選挙で同じ政党から候補者が複
数擁立されていなかった選挙区では、候補者の擁立パターンに変化があ
ったとはいえないだろう。
日本の 1993 年選挙と 1996 年選挙に着目した場合、1993 年選挙で同
じ政党から候補者が複数擁立されていた選挙区は影響を受けたといえ
るだろう。1993 年選挙では 129 選挙区に候補者を擁立することができ
たが、そのうち 103 選挙区で自民党は候補者を複数擁立した。2 また、
社会党は 7 選挙区で候補者を複数擁立していた。
一方で、1993 年選挙で同じ政党から候補者が複数擁立されていなか
った選挙区は影響を受けなかったといえるだろう。1993 年では自民党
は 20 選挙区で候補者を複数擁立していなかった。一方で社会党は 102
選挙区で候補者を複数擁立していなかった。また共産党は 129 選挙区す
べてに候補者を擁立したが、どの選挙区にも候補者を複数擁立していな
2
かった。
以上のことから、候補者の擁立するパターンが有権者の志向に変化を
与えることを検証するため、次のようなメカニズムで仮説を考えた。
1993 年選挙で同じ政党から候補者が複数擁立されていた選挙区の有権
者の場合、1996 年選挙で有権者の志向は変化する。一方で、1993 年選
挙で同じ政党から候補者が複数擁立されていなかった選挙区の有権者
の場合、1996 年選挙で有権者の志向は変化しない。
上記をモデル図で示すと図 1 のようになる。
図 1 仮説のモデル図
立候補パターンの変化
有権者の志向
2-2. 仮説のメカニズム
選挙区内に同じ政党から候補者が複数立候補できた 1993 年選挙では、
有権者は候補者を相対的に重視した投票をしただろう。選挙区内にある
有権者の支持する政党から複数候補者が立候補していれば、その所属政
党によって候補者を選別しにくい。しかし有権者は何らかの方法で投票
する候補者を決定しなければならない。そのため有権者は候補者を重視
した投票をしただろう。
しかし 1996 年選挙では、同じ政党から候補者が複数立候補できなく
なったため、有権者は政党を相対的に重視した投票をしただろう。同じ
政党の候補者同士を有権者は選別することはなくなった。そのため投票
のうえで有権者が以前より重視することは、政党の主張といってよいだ
ろう。そのため有権者は政党を重視した投票をしただろう。
したがって 1993 年選挙と 1996 年選挙を比べると、後者の選挙で有
権者の重視したことが候補者から政党よりになったといえるだろう。こ
の変化は候補者の立候補パターンによるものである。
3. 分析枠組み
3-1. 使用するデータ
上記の分析をするために明るい選挙推進協会が実施した世論調査を
用いる。用いたデータは「第 40 回衆議院議員総選挙についての意識調
査(1993 年)
」
・
「第 41 回衆議院議員総選挙についての意識調査(1996
3
年)
」である。 有権者の志向の変化と候補者の擁立について作業定義
が可能な質問があったため、この 2 つのデータを選んだ。
3-2. 作業定義と分析方法
両選挙調査に同様の質問項目があったため、1993 年選挙と 1996 年選
挙を分析するための作業定義は同じである。
候補者の立候補パターンを独立変数に設定する。1993 年選挙で同じ
3
政党から候補者が複数擁立されていた選挙区・擁立されていなかった選
挙区に分ける。このように分けることで、どの選挙区にも候補者が複数
擁立されていない 1996 年選挙での立候補パターンの変化を確認できる。
しかしデータの都合により、立候補パターンの代理変数として有権者
の政党支持を独立変数に設定しなおしたい。候補者が複数擁立さている
選挙区を有権者の自民党支持とおきかえる。また、候補者が複数倒立さ
れていない選挙区を有権者の社会党支持と共産党支持におきかえる。
おきかえの判断基準は、同じ政党から候補者が複数擁立されていた選
挙区数とした。2-1. 仮説の想定で確認したとおり、1993 年選挙におい
て、
129 選挙区のうち 109 選挙区に自民党は候補者を複数擁立していた。
そのため、自民党を支持する多くの有権者には、1996 年選挙で候補者
の立候補パターンに変化があったといってよいだろう。一方で社会党は
7 選挙区に候補者を複数擁立していた。また共産党はどの選挙区にも候
補者を複数擁立していなかった。そのため、これら 2 政党を支持する多
くの有権者には、1996 年選挙で候補者の立候補パターンに変化がなか
ったといってよいだろう。
また従属変数には有権者の志向を設定する。有権者の志向を、政党志
向・候補者志向・一概にいえない、の 3 つとした。質問には候補者と有
権者のどちらを重視して投票したかについて項目がありそれを使用し
た。
したがって、代理変数を設定した分析のモデルを図で示すと図 2 のよ
うになる。
図 2 分析のモデル図
選挙制度改革の支持政党の候補者擁立パターンの変化あり
自民党支持者
志向が変化する
選挙制度改革の支持政党の候補者擁立パターンの変化なし
社会党支持者
志向が変化しない
共産党支持者
図 2 のモデルにしたがってクロス表集計をおこない、1993 年選挙と
1996 年選挙を比較し仮説を検証する。
4
4. 分析結果
志向
表 1 1993 年選挙(%)
政党
自民
社会
共産
38.6
51.5
58.5
52.5
42.3
36.6
9.0
6.3
4.9
100
100
100
N
791
239
41
出典:第 40 回衆議院議員総選挙についての意識調査
カイ 2 乗値:17.453 危険率:.002
政党
候補者
一概に言えない
合計
志向
政党
候補者
一概に言えない
合計
表 2 1996 年選挙(%)
政党
自民
社会
45.0
46.1
8.8
100
39.5
45.7
14.8
100
共産
76.0
20.0
4.0
100
N
635
81
50
出典:第 41 回衆議院議員総選挙についての意識調査
カイ 2 乗値:22.048 危険率:.000
有権者の支持政党ごとに、1993 年選挙(表 1)と 1996 年選挙(表 2)
を比較し志向の変化を確認する。4
自民党支持の有権者は、仮説の想定どおり 1993 年選挙から 1996 年
選挙でより政党志向になったといえる。一概にいえないと答えた有権者
の%がほぼ変わることなく、政党志向が 38.6%から 6.4%増加し 45.0%
になっているからだ。
社会党支持の有権者は、仮説の想定どおりにはならず 1993 年選挙か
ら 1996 年選挙でより候補者志向になったといえる。候補者志向は
42.3%から 3.4%増加し 45.7%になっているからだ。さらに政党志向が
51.5%から 12%減少し 39.5%になっていることからも、候補者志向にな
ったといえるだろう。また一概にいえないと答えた有権者の%は 6.3%
から 8.5%増加し 14.8%になっている。
共産党支持の有権者は、仮説の想定どおりにはならず 1993 年選挙か
ら 1996 年選挙でより政党志向になったといえる。政党志向は 58.5%か
ら 17.5%増加し 76.0%になっているからだ。さらに候補者志向は 36.6%
から 16.6%減少し 20.0%になっているからだ。一概にいえないと答えた
有権者の%はほぼ変わらなかった。
5
以上の分析結果から「候補者を複数擁立する政党を支持する有権者の
志向が、選挙制度改革以前と以後により変化する」という仮説は完全に
は実証されなかった。たしかに、複数候補者は擁立していた自民党を支
持する有権者の志向がより政党志向に変化したことを確認できた。しか
し、複数候補者を擁立していなかった社会党・共産党を支持する有権者
の志向も変化したことから、仮説は支持されなかった。
5. おわりに
本稿では、
「候補者の立候補パターンが有権者の志向をどのように規
定するか」というリサーチクエスチョンを設定し、
「候補者を複数擁立
する政党を支持する有権者の志向が、選挙制度改革により変化する」と
いう仮説の検証を試みた。その結果、仮説は支持されなかった。
仮説が支持されなかった主な原因として、次の 2 つを挙げることがで
きるだろう。
1 つは選挙区ごとではなく政党ごとの分析をし、代理変数を用いたこ
とである。代理変数を独立変数に用いたことは分析のうえで強引であっ
たかもしれない。選挙区内に複数候補者を擁立している政党は自民党で
あり、複数候補者を擁立しない政党は社会党であるモデルがすべての選
挙区にあてはまるわけではない。選挙区ごとに着目していれば、分析結
果が変わっていたかもしれない。
もう 1 つは立候補パターンの変化がただちに有権者の志向を変化させ
るとはかぎらないことである。立候補パターンが変化したとしても、有
権者が根強い候補者志向をもっていれば、志向の変化はすぐにはあらわ
れないかもしれない。
本稿では仮説が部分的にしか実証されなかったが、次の機会には立候
補パターンを選挙区ごとに分析することで、本稿よりも正確な仮説の検
証をしたい。
注
(1) 三宅(1989, 39-40)は選挙区ごとに、有権者の志向を分析してい
る。複数候補者を自民党から擁立していない選挙区では政党重視が
多くなる一方で、複数候補者を社会党から擁立している選挙区では
政党重視と候補者重視の差が小さいことも確認している。
2
( ) 1993 選挙の選挙区に関するデータは、菅原研究室(参考文献を参
照)をもとに筆者が候補者数を集計した。当ページの「データの扱
い」を守り使用した。データを公開していただいた菅原研究室の皆
様に感謝いたします。
3
( ) 「第 40 回衆議院議員総選挙についての意識調査
(1993 年)
」
・
「第 41
回衆議院議員総選挙についての意識調査(1996 年)
」は、明推協全
国明るい選挙推進協会が実施した世論調査である。それをレヴァイ
アサン・データ・バンク(LDB、木鐸社)より同志社大学が購入し
6
たものを使用した。同志社大学・法学部の西澤由隆先生のご指導と
便宜によりこのデータを利用できた。データを公開・寄託され、利
用できるようにしてくださった先生方に感謝いたします。
4
( ) 1996 年選挙では、
社会党の党名は社民党に変更されているが、
1993
年選挙時の社会党で統一した。
<補遺>
作業定義に用いた質問項目
●独立変数(有権者の支持政党)
1993 年選挙
問い
あなたは、ふだん何党を支持していらっしゃいますか。
選択肢
1 自由民主党 2 日本社会党 3 公明党 4 日本共産党 5 民社党 6 社
会民主連合 7 日本新党 8 新党さきがけ 9 新生党 10 その他
11 支持政党なし 12 わからない
1996 年選挙
問い
あなたは、ふだん何党を支持していらっしゃいますか。
選択肢
1 自由民主党 2 新進党 3 民主党 4 社会民主党 5 日本共産党 6 新
党さきがけ 7 その他 8 支持政党なし 9 わからない
●従属変数(有権者の志向)
1993 年選挙
問い
衆議院の選挙で、あなたは政党の方を重くみて投票しましたか、それと
も候補者個人を重くみて投票しましたか。
選択肢
1 政党を重くみて 2 候補者個人を重くみて 3 一概にいえない 4 わ
からない 0 非該当
1996 年選挙
小選挙区の選挙で、あなたは政党の方を重くみて投票しましたか、それ
とも候補者個人を重くみて投票しましたか。
選択肢
1 政党を重くみて 2 候補者個人を重くみて 3 一概にいえない 4 わ
からない ・非該当
シンタックス
*1993 年選挙.
FREQUENCIES
parcan partsupt.
7
*従属変数.
RECODE
parcan(1=1)(2=2)(3=3)(4=-9)
INTO
par.
MISSING VALUES par(-9).
VALUE LABELS
par 1 '政党' 2 '候補者' 3 '一概にいえない'.
FREQUENCIES par.
*独立変数.
RECODE
partsupt(1=1)(2=2)(4=3)(else=-9)
INTO
supt.
MISSING VALUES supt(-9).
VALUE LABELS
supt 1 '自民党' 2 '社会党' 3 '共産党'.
FREQUENCIES supt.
*クロス表.
CROSSTABS TABLES
par by supt
/CELLS COLUMN COUNT
/STATISTICS CHISQ.
*1996 年選挙.
FREQUENCIES
parcans partsupt.
*従属変数.
RECODE
parcans(1=1)(2=2)(3=3)(4=-9)
INTO
par.
MISSING VALUES par(-9).
VALUE LABELS
par 1 '政党' 2 '候補者' 3 '一概にいえない'.
FREQUENCIES par.
*独立変数.
RECODE
partsupt(1=1)(4=2)(5=3)(else=-9)
INTO
8
supt.
MISSING VALUES supt(-9).
value labels
supt 1 '自民党' 2 '社会党' 3 '共産党'.
FREQUENCIES supt.
*クロス表.
CROSSTABS TABLES
par by supt
/CELLS COLUMN COUNT
/STATISTICS CHISQ.
<参考文献>
・三宅一郎 1989. 『投票行動』 東京大学出版会.
・選挙制度審議会編 1990. 『選挙制度及び政治資金制度の改革につい
ての答申』
URL:
http://www.secj.jp/pdf/19900426-2.pdf#search='%E5%B9%B3%E
6%88%902%E5%B9%B4%E3%81%AE%E9%81%B8%E6%8C%9
9%E5%88%B6%E5%BA%A6%E5%8F%8A%E3%81%B3%E6%9
4%BF%E6%B2%BB%E8%B3%87%E9%87%91%E7%A8%8B%E
5%BA%A6%E3%81%AE%E6%94%B9%E9%9D%A9%E3%81%
AB%E3%81%A4%E3%81%84%E3%81%A6%E3%81%AE%E7%
AD%94%E7%94%B3' 2013/12/21 参照.
・菅原研究室編 『菅原研究室/日本政治データ/1993 年選挙衆議院議
員選挙結果』
URL:
http://freett.com/sugawara_taku/data/1993re.html 2013/12/21 参
照.
9
2
政党に関する理解度
と無党派層の増加
なぜ、1990 年代に無党派層は急増したのか
加藤 理歩
1. はじめに
日本における無党派層の割合は高い。社団法人 中央調査社の「衆議
院選挙に関する世論調査(1996 年総選挙前後調査, 1996)
」によると、
1996 年の時点で無党派層は 42.5%存在している。また、1993 年から
1996 年に蒲島郁夫らによって行われたパネル調査方式の全国世論調査
において、
「一度は無党派層であった」という回答が 50%を超えた(伊
藤・田中・真渕 2003, 119)
。
常に無党派層であるということはなくても、
一時的に無党派層になったことのある有権者が半数を超えたというこ
とである。
無党派層の選挙結果に与える影響は大きい。1996 年の衆議院議員選
挙における支持政党の有無と投票か棄権かをクロス表分析したものが
表 1 である。40%を超える無党派層のうち 74.6%の有権者が投票へ行っ
ている。つまり、有権者の約 30%が無党派層かつ投票へ行っているとい
うことになる。すると、無党派層が選挙結果に与える影響をあなどるこ
とはできない。
表 1 支持政党の有無と投票のクロス表分析 (%)
投票
支持政党
あり
なし
投票
92.5
74.6
棄権
計
7.5
100
25.4
100
カイ二乗値:74.894
出所:jeds96
(N)
(508)
(710)
危険率:0.000
無党派層は著しく増加し始めたというデータがある(伊藤・田中・真
渕 2003, 119)
。1 そこで本稿では、
「なぜ、1990 年代に無党派層は急
増したのか」というリサーチクエスチョンを 55 年体制の崩壊と新党の
登場という点に着目して分析する。次節で詳しい仮説のメカニズムを紹
介する。
10
2. 仮説
「なぜ、1990 年代に無党派層は急増したのか」というリサーチクエ
スチョンに対して、
「有権者の政党に関する理解度が低下し、無党派層
は急増した」という仮説を立てる。有権者の政党に関する理解度の低下
の原因を二つ挙げる。一つ目は、有権者が把握しなくてはならない政党
の数が増加したということだ。1993 年に自民党と社会党による、いわ
ゆる 55 年体制が崩壊した。日本新党・新党さきがけ・新生党といった
新しい政党が登場し、政党の数は増加した。政党の数が増えると、有権
者が政党に関する情報を集めるコストが高くなるので政党を理解しづ
らくなる。二つ目は、似通ったイデオロギーを持つ政党が複数存在する
ということである。自民党と社会党時代には、二つの全く反対のイデオ
ロギーを比較すれば良かった。しかし、似通ったイデオロギーを持つ政
党が複数登場したことでイデオロギーを材料に支持政党を選ぶことが
難しくなったと考える。
本稿では、独立変数を「政党に関する理解度」
、従属変数を「支持政
党の有無」と設定し分析を行う。
図 1:モデル図
独立変数
従属変数
政党に関する
理解度
支持政党の
有無
3. 分析枠組み
3-1. 使用するデータ
本稿では、社団法人 中央調査社の「選挙とデモクラシー研究会」に
よる「衆議院選挙に関する世論調査(1996年総選挙前後調査, 1996)
」
2
のデータ(jeds96と省略)を使用する。
3-2. 作業定義
本節では、政党に関する理解度(独立変数)と支持政党の有無(従属
変数)をどのように操作化しているのかを説明する。
・政党に関する理解度(独立変数)
分析では、
「政党に関する理解度」を低・中・高という段階に分けて
いる。分析で用いる質問では、自民党・共産党・新進党・新党さきがけ
民主党・社民党・その他の政党という7つの選択肢それぞれに対して「よ
11
くわかっている」
・
「ある程度わかっている」
・
「名前程度である」
・
「わか
らない」
・
「答えない」という評価を与えている。有権者ごとの政党に関
する理解度をまとめるために、前述の評価に点数を与えて合計点を算出
した。具体的には、
「よくわかっている」に 1 点、
「ある程度わかってい
る」に 0.67 点、
「名前程度である」に 0.33 点、
「わからない」
・
「答えな
い」に 0 点という点数を与えた。7 つの選択肢(自民党・共産党・新進
党・新党さきがけ・民主党・社民党・その他の政党)に対するそれぞれ
の点数の合計点(0 点から 7 点)の小数点以下を四捨五入して一桁で表
した。その点数の 0 点から 2 点を「低」
、3 点から 5 点を「中」
、6 点・7
点を「高」と分類した。
・支持政党の有無
いずれかの政党(自民党・共産党・新進党・新党さきがけ・民主党・
社民党・その他の政党)を支持しているという回答者を「支持政党あり」
、
「どの政党でもない」という回答者を「支持政党なし」と分類した。
4. 分析結果
「政党に関する理解度」と「支持政党の有無」のクロス表が以下の表
2 である。危険率が 0%であり、
「政党の理解度」と「支持政党の有無」
に相関関係が存在することを確認できた。
表 2 政党の理解度と支持政党の有無のクロス表分析(%)
支持政党
なし
あり
計
(N)
政党の理解度
低
50.9
49.1
100
(687)
中
34.1
65.9
100
(616)
高
23.0
77.0
100
(61)
カイ二乗値:47.447
危険率:0.000
しかし、本稿の「有権者の政党に関する理解度が低下し、無党派層は
急増した」という仮説は満足には証明されなかった。なぜならば、政党
に関する理解度の「低い」有権者においては、支持政党の有無に大差は
見られなかったからである。政党の理解度が「低い」有権者の中で支持
政党を持っている有権者が 49.1%、支持政党を持っていない有権者は
50.9%であった。支持政党なしの割合の方が大幅に高くなることを期待
していたが、その通りの結果を得ることはできなかった。
政党の理解度が低い有権者の中で、支持政党の有無に差があまり出な
かったことには理由が考えられる。それは、政党の理解度が低い有権者
の中には、特定の政党にだけ高い理解度を示す有権者が含まれているか
らである。特定の政党に関する理解度の高い有権者はその特定の政党を
12
支持している可能性が高い。例えば、極端な例ではあるが、自民党に関
しては「よくわかっている」
(1 点)
、その他の政党に関しては「わから
ない」
(0 点)とある有権者が回答したとするとその有権者の政党に関す
る理解度の合計点は 1 点になる。すると、政党に関する理解度の「低」
グループにこの有権者は分類される。しかし、自民党にだけ強い理解を
示すこの有権者は自民党を支持している可能性が高く、支持政党を持っ
ていると考えられる。よって、政党の理解度が低いからといって、支持
政党を持たない有権者が支持政党を持つ有権者よりも大幅に多いとい
う結果は得られなかったのではないだろうか。
ただし、政党の理解度が上がるほど支持政党を持つ有権者は増えると
いうことがこの分析から確認できた。政党の理解度が「中」程度の有権
者の中では、支持政党を持っている有権者の方が 31.8%高い。さらに政
党の理解が「高い」有権者の中では、支持政党を持っている有権者の方
が 54.0%も高くなっている。つまり、政党に関する理解度が高い方が支
持政党を持ちやすいということである。
5. おわりに
本稿では、
「なぜ、1990 年代に無党派層は急増したのか」というリサ
ーチクエスチョンに対し「政党に関する理解度が低下し、無党派層は急
増した」という仮説を立てた。独立変数を「政党に関する理解度」
、従
属変数を「支持政党の有無」として jeds96 のデータを使用しクロス表
分析した。その結果、政党の理解度の「低い」有権者において支持政党
の有無に大きな差が見られなかった。それは、ある特定の政党にだけ強
い理解度を示す有権者も政党に関する理解度の「低い」グループに分類
されているからだ。特定の政党にだけ理解を示す有権者はその政党を支
持している可能性が高いと考えられる。一方で、政党に関する理解度が
高くなればなるほど、支持政党を持つ有権者が増えることもわかった。
本稿の分析における反省点を2つあげる。1 つめは、政党の理解度の
「低い」グループに特定の政党にだけ高い理解を示す有権者も含まれて
しまったという点である。そのことによって、理解度の「低い」有権者
においての支持政党の有無に大差が見られず、理解度が「中くらい」
・
「高
い」と上がるにつれて支持政党を持つ有権者も増加するという大きな特
徴しか確認することができなかった。2 つめは、1996 年選挙の分析しか
行わなかったという点である。本稿の仮説は、新党の登場によって有権
者の政党に関する理解度が低下し、無党派層は急増したというものであ
った。従って、新党の登場前との比較研究が必要であった。これら 2 点
の反省を今後の研究に生かしたい。
また、無党派層の増加には政党に関する理解度以外にも様々な要因が
働いていると考えられる。それは、1990 年代に無党派層は「急増」し
たが、それ以前から漸次増加してきているからだ。3 このことから、無
13
党派層増加の要因のさらなる追究を今後の課題であると考える。
注
(1) 以下の図は 1990 年代に無党派層が急増したこと・1990 年代以前か
らも漸次無党派層は増加していることを示す図である。伊藤光利・
田中愛治・真渕勝の『政治過程論』 119 ページより引用した。
図 2 : 支持政党の推移
(2) ここで利用したデータである社団法人 中央調査社の「衆議院選挙に
関する世論調査(1996 年総選挙前後調査,1996)
」は「選挙とデモク
ラシー研究会(JEDS)
」
(西澤由隆教授も参加)が実施した世論調
査である。同志社大学・法学部の西澤由隆先生のご指導と便宜によ
り利用ができた。このデータを公開・寄託され、利用できるように
してくださった先生方に感謝申し上げます。
(3) 注(1)に同じ
14
<補遺>
①作業定義
・政党に関する理解度
Q3 日本の政党にはどんな政党がありますか、思いつくままにいくつ
でもあげてください。
(SQ1)あなたは、その政党がどんな政党か、大体おわかりですか。
政党:自民党/共産党/新進党/新党さきがけ/民主党/社民党/その他の政党
理解度:よくわかっている/ある程度わかっている/名前程度である/わか
らない/答えない
・支持政党の有無
Q6 選挙のことは別にして、ふだんあなたは何党を支持しています
か。
支持政党あり:自民党/新進党/民主党(鳩山・菅新党)/社民党(旧社会
党)/共産党/新党さきがけ/その他の政党
支持政党なし:どの政党でもない
欠損値:わからない/答えない
②シンタックス
・各党の理解度に点数を与える。
recode A3LDPS1(1=1)(2=0.67)(3=0.33)(4,5=0)(9=-9) into new 自民党.
missing values new 自民党(-9).
fre new 自民党.
recode A3NFPS1(1=1)(2=0.67)(3=0.33)(4,5=0)(9=-9) into new 新進党.
missing values new 新進党(-9).
fre new 新進党.
recode A3DPJS1(1=1)(2=0.67)(3=0.33)(4,5=0)(9=-9) into new 民主党.
missing values new 民主党(-9).
fre new 民主党.
recode A3SDPS1(1=1)(2=0.67)(3=0.33)(4,5=0)(9=-9) into new 社民党.
missing values new 社民党(-9).
fre new 社民党.
recode A3JCPS1(1=1)(2=0.67)(3=0.33)(4,5=0)(9=-9) into new 共産党.
missing values new 共産党(-9).
fre new 共産党.
recode A3NPHS1(1=1)(2=0.67)(3=0.33)(4,5=0)(9=-9) into new
さきがけ.
missing values new 新党さきがけ(-9).
fre new 新党さきがけ.
新党
recode A3OTHS1(1=1)(2=0.67)(3=0.33)(4,5=0)(9=-9) into newothers
15
missing values newothers(-9).
fre newothers.
・政党の理解度の合計を出し、小数点以下を切り捨てる。
compute ptyinf= rnd(SUM(new自民党,new 新進党,new民主党,new 社
民党,new 共産党,new 新党さきがけ,newothers)).
・政党の理解度を3つに分類する。
recode ptyinf(0 thru 2=1)(3 thru 5=2)(6,7=3) into pi.
fre pi.
・支持政党の有無
fre a6.
recode a6(1 thru 7=1)(8=0)(9,10=-9) into ps.
missing values ps(-9).
fre ps.
・政党の理解度と支持政党の有無のクロス表分析
crosstabs tables =pi by ps
/cells row
/statistics chiq.
<参考文献>
・伊藤光利・田中愛治・真渕勝 2003.『政治過程論』 有斐閣.
16
3
なぜ、若者は投票に行
かないのか
―政治的有効性感覚に注目して―
豊島 由梨佳
1. はじめに
長年の政治的懸念事項として若者の政治参加率の低さが挙げられる。
これまで、若者の政治参加についての検証を多くの研究者が行ってきた。
その中でも若者の投票率に注目すると、低投票率の規定要因として、社
会的属性である「年齢」は有意であることが実証されている。三宅(1990)
によると、年齢が 20 歳代であることは棄権する傾向を高めるとされて
いる(三宅 1990, 112)
。そして、小林(2005)にもあるように、三宅・
木下・間場は『異なるレベルの選挙における投票行動の研究』
(1967)
において、政治参加を社会的属性で説明しようと試み、その結果として
年齢の影響を実証している。
その上近年では、若者の政治離れが更に加速している。図 1 で明らか
なように、明るい選挙推進協会の調査結果によると 1993 年 7 月に行わ
れた第 40 回衆議院議員総選挙以来、20 歳代有権者の投票率は 50%を下
回るようになっている。2012 年 12 月に行われた第 46 回衆議院議員総
選挙においてはもはや、20 歳代有権者の投票率が約 38%にまで落ち込
んでしまっている。
17
図 1 20 歳代有権者の投票率推移(衆議院議員総選挙)
出所:明るい選挙推進協会 HP
このような現状を踏まえて筆者は 2 つの疑問を抱き、その疑問を明ら
かにすることで若者の投票率改善につなげることができるのではない
かと考えた。1 つ目の疑問は、若者に特有の低投票率の規定要因が存在
しているのではないかというものである。そして 2 つ目は、その要因は
外部からの働きかけにより投票率改善につなげることができるのでは
ないかというものである。若者に特有な低投票率の規定要因を明らかに
することが投票率向上の足掛かりとなり、日本の将来を支える若者の政
治離れを食い止めることにつながるだろう。
そこで数多く考えられる若者の低投票率の規定要因の中から、筆者は
政治的有効性感覚をとり上げて分析を行いたい。三宅(1990)によると、
「政治的有効性感覚は、政治的有力感とも呼ばれ、個人の政治行動が政
治的に有効(あるいは有能)でありうるという感覚であって、客観的に
みて事実そうであると否とを問わない。
(中略)自分の政治的行動が有
効だという感覚をもっているのと、もっていないのとでは、政治参加に
大きな違いをもたらす。
」とされている(三宅 1990, 125)
。
また政治的有効性感覚は投票行動の規定要因として既に実証されて
おり、
「個人の政治的関わりを問題とする」
ものである
(三宅 1986, 116)
。
政治的有効性感覚の低い人は投票に行かないと言えるのならば、若者の
低投票率は若者の政治的有効性感覚が他の世代よりも低いことによっ
て説明できるはずである。
このような考察から本稿では、
「若者の低投票率の規定要因はなにか」
というリサーチクエスチョンを検証するために「政治的有効性感覚が低
いため、若者(20・30 代)は投票に行かない」という仮説を設定する。
1
そして、若者の政治的有効性感覚を下げている要因として都市規模・
18
政治家への不信感を挙げ、段階的に計量分析を行う。若者の政治的有効
性感覚を低下させている要因を確認する理由は、若者に独特な政治的有
効性感覚への影響を確認することが投票率向上にむけたアプローチを
考える上で有益なことだと考えるからである。都市規模・政治家への不
信感という低下要因を挙げた理由は第 2 節で詳しく示す。
2. 仮説のメカニズム
数多く考えられる若者の低投票率の規定要因の中から、前節でも示し
たように政治的有効性感覚をとりあげて、
「政治的有効性感覚が低いた
め、若者は投票に行かない」という仮説をたてる。政治的有効性感覚の
高低は政治参加に影響をもたらすとされている。したがって、若者の政
治的有効性感覚が他の世代に比べて低ければ、若者の投票行動に影響を
及ぼすであろうと考えた。
また、政治的有効性感覚低下の要因として都市規模・政治家への不信
感を挙げる。この 2 つの要因を挙げた理由は、以下の通りである。都市
規模を挙げたのは、若者人口の都市集中と都市部における一票の価値の
相対的低下という要素の指標とするためである。若者は都市に集中し、
若者が住む都市部の一票の価値は地方に比べて低い。この要素が若者全
体の政治的有効性感覚低下に影響をもたらしているのではないかと考
えた。政治家への不信感を要因に挙げたのは、有権者の利益を代表する
政治家への不信感増大が政治的有効性感覚低下の要因になると考えた
ためである。そして、若者は他の年齢層に比べて政治家への不信感が大
きいと予想する。他の世代に比べて若者は、政治への接点が少ないだろ
う。その結果として、政治という抽象性の高いものに対する不信感が、
政治家という比較的容易にイメージのできる存在への不信感にあらわ
れやすいのではないかと考えた。図 2 が仮説のモデル図である。
図 2 モデル図
第二段階分析
第一段階分析
都市規模大
若者の
政治的有効性感覚
低下
政治家への不信感大
19
若者の
低投票率
3. 分析枠組み
3-1. 使用するデータと分析手法
上記の仮説を実証するために、本稿では「社会意識と生活に関する調
査」
(以下、jeds2000 とする)のデータを用いてクロス表分析を行う。2
そして本稿では、仮説を 2 段階に分けて検証している。まず、都市規
模・政治家への不信感と若者の政治的有効性感覚について分析を行い
(第一段階分析)
、次に若者の政治的有効性感覚と投票についての分析
を行う(第二段階分析)
。
3-2. 作業定義
本節では使用する変数の作業定義を示す。
分析における独立変数と従属変数は以下の通りである。第一段階の分
析における独立変数は「都市規模」
・
「政治家への不信感」であり、従属
変数は「年齢層別にみた政治的有効性感覚の高低」である。そして、筆
者が特に確認したい第二段階の分析における独立変数は「年齢層別にみ
た政治的有効性感覚の高低」であり、従属変数は「投票あるいは棄権」
である。
まず、第一段階の分析で使用する変数の操作について示す。
・都市規模
この変数は、被調査者の居住地域の規模を jeds2000 の分類を参考に
操作した。町村と人口 10 万人未満の都市に「1」
、10 万人以上 20 万人
未満の都市に「2」
、20 万人以上の都市に「3」
、13 大都市に「4」とい
う値を与えた。なお、13 大都市とは東京都区部・札幌市・仙台市・千葉
市・横浜市・川崎市・名古屋市・京都市・大阪市・神戸市・広島市・北
九州市・福岡市のことを指す。
・政治家への不信感
この変数は 3 つの質問において、
その回答項目にそれぞれ点数を与え、
各質問での獲得点数の合計点を確認した。質問は、
「さっそくですが、
国会議員は、大ざっぱにいって、当選したらすぐ国民のことを考えなく
なると思いますか、それともそうは思いませんか」
・
「国の政治を動かし
ている人々の中には、不正をする人がどのくらいいると思いますか」
・
「日本の政党や政治家は派閥争いや汚職問題に明け暮れして、国民生活
をなおざりにしていると思いますか、それともそうは思いませんか」の
3 項目を使用した。合計点が大きくなるほど、政治家への不信感も大き
くなる指標である。また、合計点の最小値は「0」であり、最大値は「4」
である。各質問における詳しい値の与え方に関しては補遺に示す。
・年齢層別にみた政治的有効性感覚の高低
この変数を使用するために 2 つの変数操作を行った。
まず、政治的有効性感覚の変数操作について示す。この変数には、
「
『自
20
分自身には政府のすることに対して、それを左右する力はない』につい
てどう思いますか。この中からお答えください。
」という質問を使用し
た。そして、
「そう思う」を「最低」
、
「まあそう思う」を「低」
、
「あま
りそうは思わない」を「高」
、
「そうは思わない」を「最高」とした。
次に、年齢層の変数操作について示す。筆者は若者の政治的有効性感
覚に注目しているため、この操作が必要であった。20 歳から 39 歳まで
を「若年層」
、40 歳から 59 歳までを「中年層」
、60 歳以上を「高齢層」
と分類した。
そして次に、第二段階の分析で使用する変数について示す。独立変数
は既に示したため省略する。
・投票―棄権
この変数は jeds2000 における調査日直近の衆議院総選挙への投票参
加有無に関する質問を使用する。投票に参加した被調査者には「1」
、棄
権した被調査者には「0」の値を与えた。
4. 分析結果
本節では分析結果を示す。第一段階分析の結果は第 1 項と第 2 項で示
し、第二段階分析の結果は第 3 項で示す。また、分析によっては 1 つの
セルに当てはまる度数が低くなってしまっているものもあるため、分析
結果の正当性が疑われる可能性もある。
4-1. 都市規模と政治的有効性感覚
第一段階の分析としてはじめに、都市規模と若者の政治的有効性感覚
の相関関係を確認する。
まず分析の前提として、若者人口が都市に集中しているのかを確認す
る。以下の表 1 が分析結果であり、1%水準で有意な結果となった。表 1
において各年齢層の人口分布に注目すると、中年層と高齢層に比べて、
若年層は都市部に集中している傾向にあると言えるであろう。若年層の
人口分布のみに目を向けると、確かに町村・10 万人未満の都市に住む若
者の割合が最も高い。しかし年齢層別に比較すると、若年層の 13 大都
市に住む割合は他の世代に比べて高いことが確認できる。
そして、都市規模と政治的有効性感覚の相関関係では予想とは異なっ
た結果になった。以下の表 2-1(若年層)
・表 2-2(中年層)
・表 2-3(高
齢層)
・表 2-4(全世代合計)が分析結果である。中年層は 1%水準、全
世代合計は 5%水準で有意な結果となったが、筆者が興味をもっている
若年層は残念ながら危険率が高いために有意な結果にならなかった。仮
説では当初、都市規模が大きい土地では、政治的有効性感覚が低い有権
者の割合が高いと予想していた。しかし、都市規模が小さい土地に住む
有権者の方が、政治的有効性感覚は低い傾向にあるという相関関係が表
21
2-2 と表 2-4 で確認できる。
またこの分析は年齢層別に、各都市規模に居住する有権者の政治的有
効性感覚の高低に関する傾向を確認している。そのため、表における各
都市規模の列を比較することによってその傾向を確認しようとした。
以上の分析結果より、都市規模は若者の政治的有効性感覚を低下させ
る要因であるとは、今回の分析では確認できなかった。確かに中年層と
全世代合計では都市規模と政治的有効性感覚の相関関係が確認できた。
しかしその相関関係は筆者の予想と異なり、都市規模が小さい土地に住
む有権者の政治的有効性感覚は低いというものであった。また、注目し
ていた若年層では有意な結果を得ることはできず、相関関係も確認でき
なかった。したがって、都市規模が大きい土地に住む若者は、政治的有
効性感覚が低くなるという仮説は支持されなかった。
表 1 年齢層と都市規模(%)
年齢層
(N=1618)
若年層
中年層
高齢層
年齢層
(N=382)
若年層
年齢層
(N=647)
中年層
合
計
都市規模
町村
10 万人未満
10 万人以上
20 万人未満
34.4
43.7
47.3
15.1
11.5
11.6
20 万人以上
(N)
13 大都市
25.9
24.6 100 (398)
24.0
20.7 100 (670)
25.1
16.0 100 (550)
カイ二乗値=21.412 危険率=0.002
表 2-1 都市規模と政治的有効性感覚(若年層)(%)
都市規模
政治的有効性感覚
最低
低
高
最高 合計
(N)
町村・10 万未満
48.1 36.8
8.3
6.8 100 (133)
10 万以上 20 万未満
46.6 31.0 12.1 10.3 100
(58)
20 万以上
40.8 39.8 12.2
7.1 100
(98)
13 大都市
43.0 35.5 11.8
9.7 100
(93)
カイ二乗値=3.787 危険率=0.925
表 2-2 都市規模と政治的有効性感覚(中年層)(%)
都市規模
政治的有効性感覚
最低
低
高
最高
合計
町村・10 万未満
10 万以上 20 万未満
20 万以上
13 大都市
54.8
42.1
35.5
34.8
22
25.3
27.6
38.7
31.9
11.7
8.2
25.0
5.3
14.8
11.0
17.8
15.6
カイ二乗値=33.225
(N)
100
(281)
100
(76)
100
(155)
100
(135)
危険率=0.000
表 2-3 都市規模と政治的有効性感覚(高齢層)(%)
年齢層
都市規模
政治的有効性感覚
(N=504)
最低
低
高
最高
合計
(N)
町村・10 万未満
49.6 25.6
14.3
10.5
100
(238)
高齢層
10 万以上 20 万未満
50.0 21.0
17.7
11.3
100
(62)
20 万以上
48.0 19.5
20.3
12.2
100
(123)
13 大都市
46.9 34.6
9.9
8.6
100
(81)
カイ二乗値=9.552 危険率=0.388
表 2-4 都市規模と政治的有効性感覚(全世代合計)(%)
年齢層
(N=1533)
全世代合計
都市規模
最低
町村・10 万未満
10 万以上 20 万未満
20 万以上
13 大都市
政治的有効性感覚
低
高
最高
51.5
45.9
41.0
40.5
27.8
26.5
32.7
33.7
合計
(N)
12.0
8.7 100
(652)
18.9
8.7 100
(196)
16.0
10.4 100
(376)
13.9
12.0 100
(309)
カイ二乗値=21.424 危険率=0.011
4-2. 政治家への不信感と政治的有効性感覚
第一段階の分析として次に、政治家への不信感と若者の政治的有効性
感覚の相関関係を確認する。
まず分析の前提として、若年層は政治家への不信感が高いのかを確認
しておく。以下の表 3 が分析結果であり、1%水準で有意な結果になっ
た。表 3 において、若年層に注目して政治家への不信感の分布を確認す
ると、若年層の約 8 割以上が「高」以上に位置している。また他の世代
に比べて、政治家への不信感が高い人の割合が高くなっている。したが
って、若者は政治家への不信感が高い人の割合が高いことが分析結果よ
り確認できる。
次に、政治家への不信感と若者の政治的有効性感覚の相関関係を確認
する。以下の表 4-1(若年層)
・表 4-2(中年層)
・表 4-3(高齢層)
・表
4-4(全世代合計)が分析結果であり、全世代合計は 5%水準、中年層と
高齢層は 10%水準で有意な結果になった。しかし、今回の分析において
も、注目している若年層は危険率が高いため残念ながら有意な結果にな
らなかった。都市規模と同様に、政治家への不信感と政治的有効性感覚
の間においても、予想していた相関関係は確認できなかった。政治家へ
の不信感が高い人は、政治的有効性感覚が低い人の割合が高いと予想し
ていたが、分析結果においては、政治家への不信感の高低にかかわらず
政治的有効性感覚が低い人の割合が高いことが確認できる。
以上の分析結果より、若者の政治家への不信感が高いことは確認でき
たが、政治家への不信感と若者の政治的有効性感覚の間には相関関係が
確認できず、仮説は支持されなかった。
23
表 3 年齢層別政治家への不信感の高低(%)
年齢層
政治家への不信感
合計
(N)
最低
低
中
高
最高
(N=630)
若年層
1.5
4.4
8.9 19.3 65.9
100
(135)
中年層
1.9
7.0
7.8 19.1 64.2
100
(257)
高齢層
7.6 10.1 17.2 17.2 47.9
100
(238)
カイ二乗値=34.485 危険率=0.00
年齢層
(N=121)
若年層
年齢層
(N=254)
中年層
年齢層
(N=228)
高齢層
表 4-1 政治家への不信感と政治的有効性感覚(若年層)(%)
政治家への不信感
政治的有効性感覚
合計 (N)
最低
低
高
最高
最低
0.0 100.0
0.0
0.0 100
(1)
低
50.0
16.7 16.7 16.7 100
(6)
中
70.0
30.0
0.0
0.0 100 (10)
高
40.9
40.9 13.6
4.5 100 (22)
最高
39.0
36.6 13.4 11.0 100 (82)
カイ二乗値=8.286 危険率=0.762
表 4-2 政治家への不信感と政治的有効性感覚(中年層)(%)
政治家への不信感
政治的有効性感覚
合計
(N)
最低 低
高
最高
最低
40.0
0.0 40.0 20.0 100
(5)
低
33.3 38.9 22.2
5.6 100
(18)
中
42.1 21.1 26.3 10.5 100
(19)
高
64.6 25.0 10.4
0.0 100
(48)
最高
44.5 26.2 18.9 10.4 100
(164)
カイ二乗値=16.537 危険率=0.168
表 4-3 政治家への不信感と政治的有効性感覚(高齢層)(%)
政治家への不信感
政治的有効性感覚
合計
(N)
最低 低
高
最高
最低
44.4 22.2 11.1 22.2 100
(18)
低
37.5 12.5 45.8
4.2 100
(24)
中
43.9 31.7
7.3 17.1 100
(41)
高
52.6 26.3 13.2
7.9 100
(38)
最高
49.5 26.2 15.9
8.4 100
(107)
カイ二乗値=23.967 危険率=0.021
24
表 4-4 政治家への不信感と政治的有効性感覚(全世代合計)(%)
年齢層
政治家への不信感
政治的有効性感覚
合計
(N)
最低 低
高
最高
(N=603)
最低
41.7 20.8 16.7 20.8 100
(24)
低
37.5 22.9 33.3
6.3 100
(48)
全世代合計
中
47.1 28.6 11.4 12.9 100
(70)
高
55.6 28.7 12.0
3.7 100
(108)
最高
44.8 28.6 16.7
9.9 100
(353)
カイ二乗値=23.314 危険率=0.025
4-3. 政治的有効性感覚と投票
最後に第二段階の分析として、若者の政治的有効性感覚と投票の相関
関係を確認する。
まず仮説の前提として、2 つの相関関係を確認したい。仮説前提の一
つ目は政治的有効性感覚と投票の相関関係であり、二つ目は年齢層と政
治的有効性感覚の相関関係である。
第 2 項までの分析手順に従えば、年齢層と政治的有効性感覚の相関関
係をはじめに確認することになるだろうが、ここでは先に政治的有効性
感覚と投票の相関関係を確認する。その理由は、今回の分析枠組みにお
いて政治的有効性感覚と投票の間に相関関係がみられなかった場合、仮
説に至るまでの前提が崩れてしまうからである。
それではまず、政治的有効性感覚と投票についての分析結果を表 6-4
で示す。これは 10%水準で有意な結果となり、政治的有効性感覚が低い
人は、棄権する人の割合が相対的に高いことが確認できた。
また、年齢層と政治的有効感覚の相関関係は予想していた通りのもの
が確認できた。表 5 によると、若年層では政治的有効性感覚が低い人の
割合が高いという結果になり、これは 5%水準で有意な結果になった。
このことは、政治的有効性感覚の「最低」
・
「低」のカテゴリーを合計し
た際に、若年層で政治的有効性感覚の低い人の割合は他の年齢層に比べ
て 5%以上高くなっているという結果により確認できる。
2 つの仮説の前提を確認できたので、第二段階の仮説検証を行う。筆
者は「政治的有効性感覚が低いため、若者は投票に行かない」という仮
説を立てている。この仮説を検証するために、年齢層別に政治的有効性
感覚と投票の相関関係を確認する。以下の表 6-1(若年層)
・表 6-2(中
年層)
・表 6-3(高齢層)
・表 6-4(全世代合計)が分析結果であり、若年
層は 5%水準で有意な結果になったが、中年層と高齢層は危険率が高く
なったので有意な結果にはならなかった。
表 6-1 で確認できるように、政治的有効性感覚の低い若者は棄権率が
非常に高くなっている。これは、先に表 6-4 で確認した政治的有効性感
覚と投票の相関関係よりも明確に関係性が表れている。それは、表 6-4
において政治的有効性感覚が「最低」である有権者の棄権率は 15.8%で
25
あったのに対して、表 6-1 においては政治的有効性感覚が「最低」であ
る若年層有権者の棄権率が 43.1%であることから確認できる。
以上より、他の世代に比べて若者は政治的有効性感覚が低い人の割合
が高い。そして、政治的有効性感覚が低い若者は棄権する人の割合が高
いという結果が実証でき、仮説は支持された。
年齢層
(N=1533)
若年層
中年層
高齢層
表 5 年齢層別政治的有効性感覚(%)
政治的有効性感覚
合計
最低
低
高
最高
44.8
44.5
48.8
年齢層
(N=122)
若年層
36.4
30.1
25.0
10.7
15.3
15.5
(N)
最低+低
8.1
100
(382)
81.2
10.0
100
(647)
74.6
10.7
100
(504)
73.8
カイ二乗値=16.636 危険率=0.011
表 6-1 政治的有効性感覚と投票(若年層)(%)
政治的有効性感覚
投票-棄権
合計
棄権
投票
最低
低
高
最高
43.1
56.9
20.0
80.0
13.3
86.7
9.1
90.9
カイ二乗値=10.807
(N)
100
(51)
100
(45)
100
(15)
100
(11)
危険率=0.013
表 6-2 政治的有効性感覚と投票(中年層)(%)
年齢層
政治的有効性感覚
投票-棄権
合計
(N)
棄権
投票
(N=251)
最低
11.7
88.3
100
(120)
中年層
低
10.8
89.2
100
(65)
高
8.7
91.3
100
(46)
最高
20.0
80.0
100
(20)
カイ二乗値=1.805 危険率=0.614
表 6-3 政治的有効性感覚と投票(高齢層)(%)
年齢層
政治的有効性感覚
投票-棄権
合計
(N)
棄権
投票
(N=227)
最低
7.4
92.6
100
(108)
高齢層
低
3.4
96.6
100
(58)
高
2.7
97.3
100
(37)
最高
0.0
100.0
100
(24)
カイ二乗値=3.373 危険率=0.338
26
表 6-4 政治的有効性感覚と投票(全世代合計)(%)
年齢層
政治的有効性感覚
投票-棄権
合計
(N)
棄権
投票
(N=600)
最低
15.8
84.2
100
(279)
全世代合計
低
10.7
89.3
100
(168)
高
7.1
92.9
100
(98)
最高
9.1
90.9
100
(55)
カイ二乗値=6.433 危険率=0.092
5. おわりに
本稿では、若者の高い棄権率を政治的有効性感覚に注目して分析を行
い、その結果として、自仮説はある程度支持されたと言えるだろう。筆
者が最も確認したかった政治的有効性感覚と若者の低投票率の相関関
係は、予想していた通りの結果となった。他の年齢層に比べて若者は、
政治的有効性感覚が低い人の割合が高く、政治的有効性感覚が低い若者
の棄権率は高くなった。今回の分析結果から確認できたことは政治的有
効性感覚の上昇が若者の投票率向上に影響を与える可能性が高いとい
うことである。しかし、若者の政治的有効性感覚を低下させる要因であ
ると予想した都市規模・政治家への不信感は、今回の分析では要因たり
えなかった。この点が、今回の分析において非常に心残りである。
そのため、政治的有効性感覚を低下させる要因を明らかにしていくこ
とが今後の課題であろう。これを明らかにしていくことにより、若者の
投票率を上昇させるためのアプローチの選択肢を増加させることがで
きる。また多くの選択肢を提供することで、複数のアプローチによる相
乗効果を望むこともできるはずだ。先にも述べた通り、若者の低投票率
は長年の政治的懸念事項である。投票率改善に向けたアプローチを考え
ることは、非常に重要な動きであろう。
それに加えて筆者個人としては、今回の分析において有効サンプル数
の少なさが懸念されることも非常に心残りであった。これも、今後の課
題として心に留めておきたい。
注
(1) 筆者は 20 代の若者の投票行動に注目して研究を始めた。しかし、使
用したデータでは 20 代有権者の有効サンプル数が少なくなってし
まうため、若者の投票行動についての先行研究を参考に 20・30 代
の有権者を若者と分類したことをはじめにことわっておく。20 代有
権者の投票行動研究に終始できなかったことは非常に心残りである
が、次につながる研究になったのではないかと感じている。
(2) ここで利用したデータ「社会意識と生活に関する調査」は、社会と
デモクラシー研究会(JEDS)
(西澤由隆教授も参加)により企画さ
27
れ、社団法人 中央調査社により実施された「日本人の民主主義観
と社会資本(2000)
」内で行われた主回答者に対する調査を使用し
ている。同志社大学・法学部の西澤由隆先生のご指導と便宜により
利用することができた。このデータを公開・寄託され、利用できる
ようにしてくださった先生方に感謝申し上げます。
<補遺>
・作業定義
政治家への不信感
各回答項目には、以下のように値を与えた。
質問文 1
「さっそくですが、国会議員は、大ざっぱにいって、当選したらすぐ
国民のことを考えなくなると思いますか、それともそうは思いませ
んか。
」
○考えなくなる/考えなくなる傾向にある→2
○場合による→1
○どちらかと言えば、国民のことを考えている/国民のことを考えてい
る→0
○DK/NA→欠損値
質問文 2
「国の政治を動かしている人々の中には、不正をする人がどのくらいい
ると思いますか。
」
○たくさんいる/ある程度いる→1
○それほどいない/ほとんどいない→0
○DK/NA→欠損値
質問文 3
「日本の政党や政治家は派閥争いや汚職問題に明け暮れして、国民生活
をなおざりにしていると思いますか、それともそうは思いませんか。
」
○そう思う/どちらかと言えばそう思う→1
○あまりそうは思わない/そうは思わない→0
○DK/NA→欠損値
・シンタックス
recode f49age(lowest thru 39=1)(40 thru 59=2)(60 thru highest=3)
into gene.
recode ff35age(lowest thru 39=1)(40 thru 59=2)(60 thru highest=3)
into gene2.
recode b1
(1,2=2)(3=1)(4,5=0)(7,8=-9)
into negl.
missing values negl(-9).
recode b2
28
(1,2=1)(3,4=0)(7,8=-9)
into fusei.
missing values fusei(-9).
recode b3
(1,2=1)(3,4=0)(7,8=-9)
into oshoku.
missing values oshoku(-9).
compute fushinkan=sum(negl,fusei,oshoku).
recode b13
(1=1)(2=0)(8=-9)
into vote.
MISSING VALUES vote(-9).
recode citysize
(1=4)(2=3)(3=2)(4,5=1)
into toshikibo.
fre a12.
MISSING VALUES a12(7,8).
第一段階の分析
CROSSTABS gene by toshikibo
/cells count row
/STATISTICS.
CROSSTABS toshikibo by a12 by gene
/cells count row
/STATISTICS.
CROSSTABS gene by fushinkan
/cells count row
/STATISTICS.
CROSSTABS fushinkan by a12 by gene2
/cells count row
/STATISTICS.
第二段階の分析
crosstabs gene by a12
/cells count row
/statistics.
CROSSTABS a12 by vote by gene2
/cells count row
/STATISTICS.
29
<参考文献>
・明るい選挙推進協会編 2013.
URL:
http://www.akaruisenkyo.or.jp/070various/071syugi/693/
2013 年 12 月 10 日 参照.
・小林良彰 2005. 『日本における有権者意識の動態』 慶応義塾大学出
版会.
・三宅一郎 1990. 『政治参加と投票行動―大都市住民の政治生活―』
ミネルヴァ書房.
・綿貫譲治・三宅一郎・猪口孝・蒲島郁夫 1986. 『日本人の選挙行動』
東京大学出版会.
30
3 年生論文
4
どのような女性がよ
り高い政治関心を抱
くのか
小和田 早紀
1. はじめに
一般的に女性は男性よりも政治に対する関心が低く、政治的に無知で
あるとされている。この原因は 2 点挙げられる。1 点目は政治との接触
度の差である。具体的には、女性は男性と比べると投票や地域活動とい
った政治的な活動との接触率が低く、政治との接触度が低い。2 点目は、
「
『男はそと、女はうち』あるいは『男は外で働き、女は家庭を守るべ
きである』という、ジェンダーに基づく社会や家庭における役割性別分
業に関する意識の存在」である(御巫 1999, 5)
。
しかし、御巫(1999)の言うように、政治で起こることは女性の人生
に関係していることが多いと言っても過言ではない。たとえば、育児休
業制度や時間外労働の制限の制度・児童手当制度などといった女性の人
生に関わるであろう制度は政治の世界で作られるものである。このよう
な制度や、その他多くの制度が存在するからこそ、安心・安全な生活が
でき、また、子育てにかかる経済的負担を軽減し安心して子育てができ
る環境が整う。つまり、政治は女性の人生に関係していると言っても過
言ではない。
では、実際に政治に関心がある女性はどれだけ存在するのだろうか。
冒頭で述べたとおり、これまで社会学の世界では女性は男性と比べると
政治に対する関心が低いとされていた。しかし、表1を見ていただきた
い。2005 年では、たしかに政治に対する関心を抱いていない女性は男
性と比べて多く存在する。一方で、政治に対する関心が非常にある女性
は 33.2%、多少はある女性は 59.2%存在する。つまり、政治に関心を抱
いている女性は確かに存在することが明らかである。
32
表 1:性別と政治関心
政治関心
多少はある
ほとんどない
59.2
62.4
40.6
37.6
100
100
(926)
(237)
カイ二乗値=88.171
非常にある
女性
性別
男性
合計(%)
(N)
33.2
66.8
100
(398)
まったくない
64.2
35.8
100
(53)
危険率=.000
では次に、政治に対して関心を抱く女性の中でもどのような女性がよ
り高い政治関心を抱くのだろうか。冒頭で述べたとおり、女性は男性と
比べると政治的な行動との接触率が低いため、政治に関心を抱く機会に
恵まれない。こういった状況下で女性は何をきっかけにより高い政治関
心を抱くのだろうか。
そこで、本稿では「どのような女性がより高い政治関心を抱くのか」
というリサーチクエスチョンを設定し、その仮説として「社会貢献度の
高い女性ほどより高い政治関心を抱く」という仮説を立て、計量的に分
析する。また、
「社会貢献度」以外に女性の政治関心を高めるであろう
と考えられる変数として「生活満足度」・
「地域愛着度」を取り上げる。
結論を先取りすると、
「社会貢献度が高い女性ほどより高い政治関心
を抱く」
という仮説は支持されず、
他の 2 つの仮説も支持されなかった。
以上のことを論じるために、本稿の構成を以下の通りとする。第 2 節で
は、本稿で確認したい 3 つの変数がなぜより高い政治関心を女性に抱か
せるのかについて、先行研究を用いて論じる。第 3 節では分析の枠組み
を提示し、どのように「社会貢献度」
・
「生活満足度」
・
「地域愛着度」を
定義づけるのかを示して分析を行うのかについて述べる。そしてこれら
をふまえた上で、第 4 節では分析結果と考察を述べる。最終節ではまと
めを行う。
2. 仮説
本稿の仮説は「社会貢献度の高い女性ほど、より高い政治関心を抱く」
と設定する。以後これを社会貢献度仮説と呼ぶ。また、より高い政治関
心を女性に抱かせるものとして生活満足度仮説・地域愛着度仮説の二つ
の仮説を提案する。以下に示した図 1 はそのモデル図である。
図 1:モデル図
社会貢献度
政治関心
生活満足度
地域愛着度
33
2-1. 社会貢献度仮説
社会貢献度仮説は「社会貢献度が高い女性ほど、より高い政治関心を
抱くこと」を想定している。
三宅(1990)によれば、男性と女性では関心のタイプに違いが存在す
るという。人が関心をもつ事柄を「私的領域」と「公的領域」に分ける
と、男性は女性と比べて公共・私領域にわたり広く関心をもつ「広領域
型」が多い。これに対して女性は私的領域の事項にのみ広く関心をもつ
「私集中型」が多いと三宅(1990)は述べている。2
さらに、Marcia(1976)によれば、
「女性は男性よりも環境、健康、
そして福祉問題を重視する傾向が見られる」
。ここでいう環境を、筆者
は身近な環境(地域)と定義する。
以上のような特徴をもつ女性は、身近な環境(地域)や健康・福祉問
題・消費に関係する団体に所属することで、政治との距離感が縮まり、
政治を私的領域の事項のように感じ、より高い政治関心を抱くのではな
いかと考えた。
2-2. 生活満足度仮説
生活満足度仮説は「生活満足度が低い女性ほど、より高い政治関心を
抱くこと」を想定している。一般に、女性は男性よりも平和を好み(御
巫 1999)
、
「生活満足度が低ければ、政治満足度も低くなるという傾向」
にある(明るい選挙推進協会 2011)
。
以上の先行研究から、女性は生活に満足していないほど、自身の生活
に直接関係する政府の業績に対して不満を抱き、より安心・安全で暮ら
しやすい生活を求めるためにより高い政治関心を抱くのではないかと
考えた。
2-3. 地域愛着度仮説
地域愛着度仮説は「地域愛着度が高い女性ほど、より高い政治関心を
抱くこと」を想定している。過去におこなわれた男女の意識調査につい
ての研究で「女性は男性よりも環境、健康、そして福祉問題を重視する
傾向が見られる」
(Marcia 1976, 313-314)という調査結果が発表され
ている。
環境といえば大気汚染や地球温暖化といった環境も考えられるが、本
稿における環境の定義はそれらを除く女性の生活に密着している身近
な環境、つまり地域とする。地域に対する愛着度が高いほど、地域的問
題に触れた時その問題を解決し、より安心・安全で暮らしやすい生活を
求めようとするために、より高い政治関心を抱くのではないかと考えた。
34
3. 分析枠組み
本節では本稿の分析枠組みを提示する。具体的には使用するデータ、
設定した独立変数・従属変数の作業定義について述べる。
3-1. 使用するデータと変数
上記の仮説を実証するために、本稿では「第 44 回衆議院議員総選挙
についての意識調査」を使用する。また、この調査で使用されている質
問文と回答を使用して多重クロス表分析を行う。1
3-2. 作業定義
(1) 従属変数
○政治への関心
従属変数である政治関心については、
「あなたはふだん国や 地方の
政治についてどの程度関心をもっていますか」という質問文を使用する。
そしてこの質問に対する回答として「非常に関心がある」を 4、
「多少は
関心がある」を 3、
「ほとんど関心をもっていない」を 2、
「全く関心を
持っていない」を 1 と再コードした。
「わからない」は欠損値として処
理を行った。
(2) 独立変数
○社会貢献度
社会貢献度については「あなたは、このような団体に加入しています
か。あればいくつでも結構ですからあげてください」という質問の回答
により定義する。
この質問に対して、女性が重視する傾向にある環境(地域)
・健康・
福祉問題・消費に関係する活動をおこなっている団体を選んでいるかど
うかで定義づけをおこなった。いずれの団体も社会に貢献ができる活動
をしているという前提である。
具体的には自治会、婦人会、青年団・消防団、宗教団体、住民運動・
消費者運動・市民団体のいずれかに加入していることを社会貢献と定義
する。その他の質問項目に関しては補遺を参照していただきたい。
上記 5 個の項目の内、選択した項目の数が「2 つ以上の有権者」を 2、
「1 つ選択した有権者」を 1、
「1 つも選択していない有権者」を 0 と再
コードした。
○生活満足度
生活満足度については「あなたは現在のご自分の生活にどの程度満足
していますか」という質問文を使用する。
この質問に対して「大いに満足している」を 4、
「だいたい満足してい
る」を 3、
「やや不満足である」を 2、
「大いに不満足である」を 1 と再
35
コードした。
「わからない」は欠損値として処理を行った。
○地域愛着度
地域愛着度については「あなたはこの市(区・町・村)にどの程度愛
着を感じますか。この中からお答えください」という質問文を使用する。
この質問に対して「非常に愛着を感じている」を 5、
「かなり愛着を
感じている」を 4、
「どちらともいえない」を 3、
「あまり愛着を感じて
いない」を 2、
「まったく愛着はない」を 1 と再コードした。
「わからな
い」は欠損値として処理を行った。
4. 分析結果
本節では前節で示した分析枠組みで分析をした結果を述べていく。
また、本稿では女性の政治関心のみに影響を与える変数であるのかを
確認するために、以下すべての分析結果で男性の分析結果も提示する。
4-1. 社会貢献度と政治関心
表 2-1 と 2-2 が「社会貢献度」と「政治関心」と「性別」を多重クロ
ス表にしたものである。
表 2-1:社会貢献度と政治関心のクロス表(女性)
非常にある
社会貢献度
政治関心
多少はある
ほとんどない
67.8
13.2
合計(%)
全くない
高
18.2
中
16.4
66.3
14.4
2.8
低
13.4
59.8
20.9
5.9
(N)
0.8
100
100
(339)
100
(327)
危険率=.009
カイ二乗値=17.164
(86)
表 2-2:社会貢献度と政治関心のクロス表(男性)
非常にある
社会貢献度
政治関心
多少はある
ほとんどない
43.0
7.0
合計(%)
全くない
(N)
高
48.8
1.2
100
中
36.6
52.5
10.3
0.6
低
30.6
49.8
14.7
4.9
100
(339)
100
(327)
危険率=.000
カイ二乗値=25.433
(86)
表 2-1 を見ると、社会貢献度の高い女性の中では政治関心が多少はあ
る女性の値が 67.8%と最高値を示し、非常に関心がある女性は 18.2%と
低い値を示した。社会貢献度が中くらいの女性・低い女性でも同じ傾向
が確認できる。つまり、社会貢献度仮説は支持されなかった。しかし、
危険率が示しているように、
「社会貢献度」と「政治関心」には相関関
係があることは確認できた。
表 2-2 を見ると、社会貢献度の高い男性の中では政治に非常に関心が
ある男性の値が 48.8%と最高値を示した。社会貢献度が中くらいの男性
36
の中では多少はある男性が 52.5%と最高値を示し、社会貢献度が低い男
性の中でも同じ傾向が確認できた。つまり、男性においても社会貢献度
仮説は支持されなかった。しかし、男性に関しては予想していた結果で
あった。また、こちらも危険率が示しているように、
「社会貢献度」と
「政治関心」には相関関係にあることが確認できた。
4-2. 生活満足度と政治関心
表 3-1 と 3-2 が「生活満足度」と「政治関心」と「性別」を多重クロ
ス表にしたものである。
表 3-1:生活満足度と政治関心(女性)
政治関心
非常にある
生活満足度
多少はある
ほとんどない
47.1
20.6
合計(%)
全くない
(N)
大いに満足
25.0
だいたい満足
13.6
67.9
15.9
2.5
100
やや不満足
22.9
51.4
17.1
8.6
100
(35)
大いに不満足
15.4
60.7
19.2
4.7
100
(234)
7.4
100
(68)
(521)
カイ二乗値=20.811
危険率=.014
表 3-2:生活満足度と政治関心(男性)
政治関心
非常にある
生活満足度
多少はある
ほとんどない
34.1
4.9
大いに満足
56.1
だいたい満足
32.5
56.6
やや不満足
53.6
大いに不満足
32.7
合計(%)
全くない
(N)
4.9
100
(41)
9.3
5.1
100
(452)
35.7
7.1
3.6
100
(56)
43.7
20.6
3.0
100
(199)
カイ二乗値=42.632
危険率=.000
表 3-1 を見ると、生活に大いに不満足な女性の中では多少はある女性
の値が 60.7%と最高値を示した。やや不満足・だいたい満足・大いに満
足である女性の中でも、同じ傾向が確認できた。つまり、生活満足度仮
説は支持されなかった。しかし、危険率が示しているように、
「社会貢
献度」と「政治関心」には相関関係があることは確認できた。
表 3-2 を見ると、生活に大いに不満足な男性の中では多少ある男性が
43.7%と最高値を示し、同じ傾向がだいたい満足している男性の中でも
確認できた。やや不満足である男性の中では非常に関心があるが 53.6%
と最高値を示し、同じ傾向が大いに満足している男性の中でも確認でき
た。つまり、生活満足度仮説は支持されなかったが、予想通りの結果で
あった。しかしながら危険率の低さが示しているように、
「生活満足度」
と「政治関心」には相関関係があることは確認できた。
4-3. 地域愛着度と政治関心
表 4-1 と 4-2 が「地域愛着度」と「政治関心」と「性別」を多重クロ
ス表にしたものである。
37
表 4-1:地域愛着度と政治関心(女性)
非常にある
地域愛着度
政治関心
多少はある
ほとんどない
55.3
18.6
合計(%)
全くない
(N)
非常に感じている
21.5
4.6
100
(237)
かなり感じている
12.6
71.7
14.4
1.3
100
(374)
どちらともいえない
10.9
61.2
21.2
6.7
100
(165)
あまり感じていない
17.4
58.0
18.8
5.8
まったくない
33.3
33.3
11.1
22.2
100
(69)
100
(9)
危険率=.000
カイ二乗値=42.347
表 4-2:地域愛着度と政治関心(男性)
非常にある
地域愛着度
政治関心
多少はある
ほとんどない
43.4
9.8
合計(%)
全くない
(N)
非常に感じている
45.7
1.2
100
(256)
かなり感じている
31.1
57.3
9.7
1.9
100
(309)
どちらともいえない
27.1
50.0
19.3
3.6
100
(140)
あまり感じていない
35.9
41.0
10.3
12.8
まったくない
14.3
57.1
28.6
0.0
カイ二乗値=48.963
100
(39)
100
(7)
危険率=.000
表 4-1 を見ると、地域に非常に愛着を感じている女性の中では政治に
多少は関心がある女性の値が 55.3%と最高値を示した。他のカテゴリー
でも似た傾向が確認できた。つまり、地域愛着度仮説は支持されなかっ
た。しかし危険率の低さから「地域愛着度」と「政治関心」は相関関係
にあることは確認できた。
表 4-2 を見ると、地域に非常に愛着を感じている男性の中では政治に
非常に関心がある男性の値が 45.7%と最高値を示した。他のカテゴリー
では女性と同様、全て政治に多少は関心があるという人の数値が最高値
を示す傾向が確認できた。つまり、男性においても地域愛着度仮説は支
持されなかった。しかしながらこの結果は予想していた結果であった。
また、危険率の低さが示すように「地域愛着度」と「政治関心」は相関
関係にあることも確認できた。
5. おわりに
本稿では、これまで一般的に男性と比べて政治的関心が低いとされて
いる女性の中でも、
「どのような女性がより高い政治関心を抱くのか」
について検討してきた。また、このリサーチクエスチョンに対して「社
会貢献度仮説」
・
「生活満足度仮説」
・
「地域愛着度仮説」の 3 つの仮説を
提案し、第 44 回衆議院議員選挙についての意識調査を利用し、分析を
おこなった。その結果、どの仮説も支持されず、設定した独立変数と従
属変数に相関関係があることのみが確認できた。
一方で、新たな発見が 2 点あった。
1 点目は、女性の有権者は自治体に多く参加しているということであ
る。具体的には、
「あなたは、このような団体に加入していますか。あ
38
ればいくつでも結構ですからあげてください」という質問文に対して、
ほとんど女性有権者は自治会のみを選択していたことが分かった。つま
り、自治体に参加している女性は多く存在するにもかかわらず、男性と
比べて女性は政治に対する関心が低い傾向にあるということである。こ
の結果をもとに考えると、自治体を除く他の社会貢献ができる団体に所
属している女性の政治関心度を確認すれば新たな発見や関係性が確認
できたかもしれない。もしくは、他の要因が大きく影響していただけか
もしれない。
2 点目は、社会貢献度が低くても、多少政治に関心を持っている有権
者は多く存在するということである。このような結果が出た原因として
は、今回使用したデータが 2005 年に行われた意識調査であったことが
考えられる。なぜならば、この年におこなわれた衆議院議員選挙は、小
泉純一郎の郵政民営化が最大の争点となり注目を浴びた選挙であり、普
段政治にあまり関心のない有権者でも社会貢献度や生活満足度・地域愛
着度に関係なく政治に関心を持ったと考えられるからである。
最後に課題として 3 点述べる。
1 点目は、データの選別である。本稿で使用した世論調査は、郵政民
営化を争点とする特殊な選挙があった年におこなわれたものであった。
つまり、この争点により他の効果が消える可能性が考えられるので、他
の年におこなわれた世論調査を使用すれば、分析結果も変わっていたか
もしれない。
2 点目は、社会貢献度の作業定義である。本研究では社会貢献度を社
会に貢献できる団体に所属しているかどうかを問う質問文で定義付け
を行った。しかし、この質問項目では、それぞれの組織内の具体的な活
動内容・活動頻度が不明であるため、それぞれの組織が客観的に見て社
会貢献をしている団体であるとは断言できない。また、たとえ社会貢献
ができる団体に所属していたとしても、所属しているだけで活動に参加
していない有権者が存在することも想定できる。これらを考慮すれば、
組織の活動内容や活動頻度を問う質問文を使用すれば、分析結果も変わ
ったかもしれない。今回はデータを入手するために必要な時間を考慮し
たうえで「第 44 回衆議院議員選挙についての意識調査」を使用したが、
早い段階でデータの選定が行えていれば予測していた分析結果に近づ
けていたかもしれない。
3 点目は、女性の政治関心に影響を与える変数して挙げた 3 つの変数
のうち、どの変数が 1 番女性の政治関心に影響を与えられているのかを
確認できなかったことである。また、今回の研究を行うにあたり、設定
した仮説がどちらの性別でも仮説が成り立つのかどうかを、男女ともに
分析を行うことで比較を行いたかった。しかし、残念ながら全ての仮説
が支持されなかったため確認が行えなかった。これも、先述のとおり、
2005 年は郵政民営化が争点となった特殊な年であったことが原因とし
て考えられる。今後機会があれば、これらを考慮したうえで改めて分析
39
を行いたい。
注
(1) ここで使用したデータ「第 45 回衆議院議員総選挙についての意識
調査」は 2005 年(平成 22 年)3 月に財団法人明るい選挙推進協会
が実施した世論調査である。同志社大学・法学部の西澤由隆先生の
ご指導と便宜により利用することができた。データを公開・寄託さ
れ、利用できるようにしてくださった先生方に感謝致します。
(2) 三宅(1990)は人が関心をもつ事柄を「私的領域」と「公的領域」の
2 つに区分している。
「公的領域」とは日本の政治情勢や選挙などを
指し、
「私的領域」とはファッションや恋愛などを指す。
そしてその関心のパターンをおおきく 4 つに分けている。
・広領域型:共・私両域にわたり広く関心をもつタイプ
・公共集中型:公共領域の事項にのみ広く関心をもつタイプ
・私集中型:私的領域のみ広く関心をもつタイプ
・狭小型:公共・私を問わず関心をもつ項目の少ないタイプ
以上の 4 つのタイプである。
<補遺>
○分析に使用した質問文とコード
●政治的関心
・質問文
「あなたはふだん国や地方の政治についてどの程度関心をもっていま
すか。
」
・質問項目
「
(ア)非常に関心がある (イ)多少は関心がある (ウ)ほとんど関
心をもっていない (エ)まったく関心をもっていない(オ)わか
らない」
●社会貢献度
・質問文
「あなたは、このような団体に加入していますか。あればいくつでも結
構ですからあげてください。
」
・質問項目
「
(ア)自治会 (イ)婦人会 (ウ)青年団・消防団 (エ)老人ク
ラブ(会) (オ)PTA (カ)農協・その他の農林漁業 (キ)
労働組合 (ク) 商工業関係の経済団体 (ケ)宗教団体 (コ)
同好会・趣味のグループ (サ)住民運動・消費者運動・市民団体
その他・どれにも加入していない・わからない」
●生活満足度
・質問文
「あなたは、現在のご自分の生活にどの程度満足していますか。
」
・質問項目
40
「
(ア)大いに満足している (イ)だいたい満足している (ウ)や
や不満足である(エ)大いに不満足である わからない」
●地域愛着度
・質問文
「あなたは、この市(区・町・村)にどの程度愛着を感じていますか。
この中からお答えください。
」
・質問項目
「
(ア)
非常に愛着を感じている (イ)
かなり愛着を感じている (ウ)
どちらともいえない (エ)あまり愛着を感じていない (オ)ま
ったく愛着はない わからない」
【シンタックス】
compute DANTAI = (neighb=1) + (women=1) + (yngfire=1) +
(pta=1)+(farm=1) + (religion=1) + (smoveorg=1).
fre DANTAI.
list neighb women yngfire pta farm religion smoveorg DANTAI
/cases from 1 to 50.
*社会貢献度
recode DANTAI (0=0) (1=1) (2,3,4=2) into DANTAI01.
fre DANTAI01.
*政治関心
fre polint.
recode polint (1=4) (2=3) (3=2) (4=1) (5=-9) into polint01.
missing value polint01(-9).
fre polint01.
*生活満足度
fre satlife.
recode satlife (1=4) (2=3) (3=2) (4=1) (5=-9) into satlife01.
missing value satlife01(-9).
fre satlife01.
*地域愛着度
fre attached.
recode attached (1=5) (2=4) (3=3) (4=2) (5=1) (6=-9)
into attached01.
missing value attached01(-9).
fre attached01.
*クロス表分析
crosstabs tables polint01 by sex
/cells count column expected resid
/statistics= chisq.
41
*多重クロス表分析
crosstabs tables satlife01 by polint01 by sex
/cells count row expected resid
/statistics= chisq.
crosstabs tables attached01 by polint01 by sex
/cells count row expected resid
/statistics= chisq.
crosstabs tables DANTAI01 by polint01 by sex
/cells count row expected resid
/statistics= chisq.
<参考文献>
・財団法人 明るい選挙推進協会 『第 44 回衆議院議員選挙の実態―調
査結果の概要―』
URL:
http://www.akaruisenkyo.or.jp/wp/wp-content/uploads/2012/07/se
arch_45syugin.pdf
2013 年 7 月 14 日参照.
・井上輝子 ・江原由美子 1991. 『女性のデータブック』 有斐閣.
・総務省選挙部 2012. 『目で見る投票率』
URL: http://www.soumu.go.jp/main_content/000153570.pdf
2013 年 7 月 14 日参照.
・婦人教育研究会編 1989. 『統計に見る女性の現状』 垣内出版.
・御巫由美子 1999. 『シリーズ《21 世紀の政治学》⑥「女性と政治」
』
新評論.
・三宅一郎 1990.『政治参加と投票行動―大都市住民の政治生活―』ミ
ネルヴァ書房.
・Huckfeldt, R. & Sprague, J. 1995. Citizens, politics, and social
communication: Information and influence in an election
campaign. Cambridge, MA: Cambridge University Press.
・Marcia Manning Lee “Why few women hold public office :
Democracy and sexual roles, ” Political Science Quarterly, vol.91,
no.2, 1976, pp.313-314.
42
5
テレビの選挙報道が
投票行動に与える影
響とは
濱田 茜
1. はじめに
有権者は選挙で投票する場合、様々な方法で情報を得るが、マスメデ
ィアが最も重要である。2005 年に行われた明るい選挙推進協会の第 44
回衆議院議員総選挙の実態の調査結果によると、役に立ったかは別とし
て有権者の 64.2%がテレビの報道を、49.0%が新聞報道を直接見たり聞
いたりしている。1 つまり、有権者が最も利用する情報源はテレビの報
道であり、次に利用されるのが新聞報道である。このことから多くの有
権者がマスメディアを見たり聞いたりして情報を得ている。
マスメディアの中でも最も利用されるテレビの報道は、政治や投票行
動に影響を与えていると考えられる。近藤(2003)は、単純に党派や縁
故などを重視して投票する人は減り、投票する相手を選ぶ判断基準を変
え、それがテレビの持つ特徴であるイメージ性や見た感じを映し出す力
と一致しているとも述べている。また、三宅(1989)は「特にテレビの
ような受け身のメディアは、報道番組が芸能化していることもあって、
強い政治関心がなくとも自然に接触する機会も少なくない」
(三宅 1989,
222)と指摘している。以上より、テレビが政治や投票行動に果たす役
割や与える影響は大きいと言えるだろう。
そこで本稿では有権者の接触率の高いテレビが、具体的に投票行動に
どのような影響を与えているかについて検証したい。本稿では、
「テレ
ビの選挙報道が投票行動に与える影響はなにか」というリサーチクエス
チョンを設定した。そして、これに対応するものとして「テレビでの選
挙報道は有権者に党首・政党・候補者のイメージでの投票行動を促す」
という仮説を立てた。仮説について詳しくは次節で述べる。
上記の仮設の検証を行う上で、本稿は以下のような構成をとる。第 2
節では仮説のメカニズム、第 3 節では分析の枠組みとして使用したデー
タの紹介・分析手順・作業定義をあげる。第 4 節では分析結果とその考
察、第 5 節では本稿のまとめを述べる。
43
2. 仮説のメカニズム
2-1. 仮説に入る前に
上記のリサーチクエスチョンを検証するため、今回はテレビの選挙報
道が有権者の投票行動にどのような影響を与えるかについて考える。こ
こでは「テレビでの選挙報道は有権者に党首・政党・候補者のイメージ
での投票行動を促す」という仮説を立てる。本稿では比較対象として新
聞報道も取り扱う。仮説については以下で詳しく説明する。
2-2. テレビの選挙報道が投票行動に与える影響
テレビでの選挙報道は党首・候補者の人柄や印象、政党の印象を報道
することがある。例えば主婦層の視聴者をターゲットとした番組では堅
苦しい選挙報道よりも、分かりやすく親しみのある選挙についての情報
を与える傾向があると考える。また、党首や候補者のスキャンダルや政
党の派閥争いについては、視聴者である有権者が興味を示しやすいため、
よく取り上げられている事柄と言えるだろう。視聴者である有権者はそ
のテレビの選挙報道を見たり聞いたりすることで、党首について取り上
げられれば、党首に親しみを感じたり、逆に不信感や嫌悪感を抱くとい
う可能性がある。また各政党について印象を左右する内容で取り上げら
れれば、その政党に対して良し悪しの印象を持つだろう。候補者の人柄
や親しみやすさについて報道されると印象が変わるかもしれない。これ
らの党首・政党・候補者に対する親しみや印象、人柄などをここでは「イ
メージ」という言葉で括ることにする。
テレビの選挙報道が与える影響としては、以下のような先行研究があ
る。池田(2007)は、テレビでの争点報道効果は新聞の争点報道効果よ
りも効果はみられないと指摘している。また谷藤(2005)は、政治が個
人化・劇化・対比化と二元化することで政策争点ではなく候補者個人と
個人の闘いになりやすいとも指摘している。
そこで本稿ではテレビの選挙報道は有権者をイメージでの投票を促
すのではないかと考えた。テレビの選挙報道を利用する人は、党首・政
党・候補者のイメージを用いて劇化・対比化された選挙報道を見たり聞
いたりすることで、党首・政党・候補者のイメージに重点を置き投票を
行うのではないかと仮定する。
以上から本稿では、メディアのなかで今回の選挙で役に立ったものを、
①テレビのみ、②テレビ・その他(新聞以外)、③テレビ・新聞・その
他、④新聞・その他(テレビ以外)、⑤新聞のみの 5 つに分け、どれほど
イメージでの投票を行っているかを確認する。
44
図 1 モデル図
テレビによる党
首・政党・候補者の
イメージ報道
有権者がイメージ
に重きを置いて
投票
3. 分析枠組み
3-1. 使用するデータ
テレビの選挙報道が投票行動に与える影響を検証するため、本稿では、
明るい選挙推進協会による意識調査「第 44 回衆議院議員総選挙の実態
(平成 18 年 3 月)
」のデータを使用する。2
平野(2007)は、
「郵政民営化問題は特定の政策次元に限定された意
味を持つものとしてではなく、むしろ小泉首相に対する信任・不信任の
シンボルとしての意味を持つ争点として有権者に理解されていたもの
と思われる」
(平野 2007, 123)と指摘している。3 本稿では、小泉へ
の信任をシンボルとしていることから、テレビの選挙報道でも小泉をは
じめとする政党・候補者をより取り上げられたと考えられる第 44 回衆
議院議員総選挙の意識調査を利用する。
3-2. 分析手順と予想される結果
テレビの選挙報道が投票に役に立ったと思う人がよりイメージに重
きを置いて投票しているかを確認したい。
前提として、支持政党を持つ有権者は、テレビが報道する党首・政党・
候補者のイメージよりも支持政党に重きを置いて投票することが想定
される。ゆえに、本分析では支持政党を持たない人を対象としている。
どのような方法で情報を得ているかについては、今回の選挙で役に立
ったものを問う質問から、テレビのみを選んだ人、テレビ・その他(新
聞を除く)を選んだ人、テレビ・新聞・その他からも選んだ人、新聞・
その他(テレビを除く)から選んだ人、新聞のみを選んだ人の 5 つに分
ける。なお、使用する質問が複数回答可であることと、有権者は一般的
に多くのメディアを利用しているであろうと考えられるため、テレビや
新聞以外のその他も含む場合も検討する。図示すると以下のようになる。
45
情報源
表 1 情 報 源 の 内 訳
情報として得た媒体
テレビ
新聞
その他
テレビのみ
○
‐
‐
テレビ・その他(新聞以外)
○
‐
○
テレビ・新聞・その他
○
○
○
新聞・その他(テレビ以外)
‐
○
○
新聞のみ
‐
○
‐
またどのような点を重く見て投票しているかについては、候補者や党
首その人自身や人柄、親しみを指す選択肢を選んでいる場合はイメージ
での投票、それ以外の選択肢を選んでいる場合はイメージ以外に重きを
置いた投票とする。また複数回答可の質問のため、どちらの選択肢も選
んでいる場合も含め、3 つに分ける。以上をクロス表分析により、検証
する。
予想される結果は、メディアのなかで今回の選挙で役に立ったものを
①テレビのみ、②テレビ・その他(新聞以外)
、③テレビ・新聞・その
他、④新聞・その他(テレビ以外)
、⑤新聞のみ、と分けた中で、イメ
ージのみでの投票の比率は①が最も多く、②もしくは③がその次に多い
と考える。このように考えるのはテレビの選挙報道の影響を受けている
と考えるからである。つまり④と⑤は②と③に比べて、イメージの投票
の比率がそれぞれ少ないと予測する。またイメージ以外での投票ではこ
の逆の結果になると予想する。仮説が正しければ以下に図示しているよ
うな結果になると予想する。
表2 予想される結果
イメージでの投票
イメージのみ
テレビのみ
テレビ・その他
(新聞以外)
情報源
テレビ・新聞・
その他
新聞・その他
(テレビ以外)
新聞のみ
イメージも含む イメージ以外
最も高い
最も高い
テレビでの選挙報道を利
用せず
⇒イメージでの投票は促
される
テレビでの選挙報道を利用
せず
⇒イメージでの投票は促さ
れず
合計
46
最も低い
合計
3-3. 作業定義
ここでは上記の分析手順における主な変数の作業定義を提示する。作
業定義の詳細については、補遺を確認していただきたい。
*どのような方法で情報得ているか
テレビのみを選んだ人=1、テレビとその他(新聞を除く)を選んだ人=
2、テレビも新聞もその他からも選んだ人=3、新聞とその他(テレビを
除く)から選んだ人=4、新聞のみを選んだ人=5
*どういう点を重く見て投票しているか
候補者や党首その人自身や人柄、親しみなどイメージでの投票の選択
肢を選んでいる=1、イメージでの投票の選択肢とイメージでの投票以
外の選択肢のどちらも選んでいる=2、イメージでの投票以外の選択肢
を選んでいる=3
*支持政党
自由民主党、民主党、公明党、日本共産党、社会民主党、国民新党、
新党日本、新党大地、その他=1、支持政党なし=2
4. 分析結果
前述の分析手順で得た結果は次の表の通りである。
表3 情報源とイメージでの投票
イメージでの投票
イメージのみ
情報源
イメージも含む イメージ以外
合計
N
テレビのみ
23.3
21.7
55.0
100
60
テレビ・その他
(新聞以外)
16.7
35.2
48.1
100
54
テレビ・新聞・
その他
27.4
32.3
40.3
100
62
新聞・その他
(テレビ以外)
15.0
35.0
50.0
100
20
新聞のみ
20.8
41.7
37.5
100
24
合計
21.8
31.4
46.8
100
220
カイ 2 乗値:6.936
47
危険率:0.544
上記の表 3 に示している分析結果を見ていく。ここで危険率も 0.544
とかなり高く、偶然性が高いということを念頭に置いておく。まずイメ
ージのみで投票した人を検討したい。最もイメージのみで投票していた
のは情報源がテレビと新聞とその他である人であった。その次に多かっ
たのは情報源がテレビのみの人である。予想していた結果とは異なるが
テレビの選挙報道をどちらも含んでいることが分かる。しかし、情報源
が新聞のみの人は、テレビのみの人よりもイメージのみで投票する人の
割合が 2.5%小さいだけである。このことからテレビを情報源とする 2
つが高い数値であったが、新聞のみの人と大差がないことから仮説は支
持されたとは言い切れないと考える。
次にイメージを含む投票した人を検討する。最も数値が高かったのは
新聞のみの人の 41.7%であった。最も低い数値であったのはテレビのみ
の人の 21.7%であった。もしもテレビの選挙報道がイメージでの投票を
促す影響があるとするならば、先ほどの数値からその影響は小さく、他
の要因の影響の方が大きいためイメージを含んではいるが、新聞のみの
人の数値が上がったとも考えられるだろう。ただし、イメージを含む投
票はイメージ以外の要因が入ってくるため、仮説が支持されたかを判断
するのは難しいと考える。
最後にイメージ以外で投票している人について検討したい。最も多か
ったのはテレビのみの人であり、最も少ないのが新聞のみの人であった。
これは予想していた結果に大きく反していた。
以上より、本稿の仮説を満たす結果には至らなかった。また最初に述
べた通り、危険率も 0.544 とかなり高いため、仮説が実証されたとは言
えないと考える。
5. おわりに
テレビは有権者にイメージでの投票を促すという仮説を立てたが、本
稿の分析からは有意な結果を得ることが出来なかった。以下に、3 つの
課題について提示する。
まず 1 つ目はテレビの選挙報道を細かく分類すべきであったという点
である。テレビの選挙報道といっても、ニュース番組からワイドショー
まで幅広く存在する。政策争点ではなく候補者・政党・党首のイメージ
に重点を置いて報道すると考えられるものとそうでないものを番組や
種類別に分けて分析をすれば、今回とはまた異なる分析結果が得られる
可能性はあると言えるだろう。
2 つ目は、分析で使用したデータが郵政民営化を争点とする近年稀に
見る特殊な 2005 年の選挙の時のものであったという点である。今回扱
った選挙が郵政民営化を争点とした選挙であり、小泉純一郎のイメージ
が強い選挙であったため、この選挙のデータを用いて分析することにし
た。小泉純一郎のイメージ・政党のイメージに着目してこの選挙のデー
48
タを用いたが、かえって情報源という要因以外が影響を及ぼした可能性
があったかもしれない。また小泉のイメージを念頭に置いているのであ
れば、自民党に投票した人とそうでない人とに分けて分析を行うと有意
な結果を得られたかもしれない。どちらにせよ、検討不足であったと考
えられる。
最後に 3 つ目は、本稿の仮説は実証されなかったが、イメージ以外の
結果を見ると最もイメージ以外で投票していたのはテレビであり、最も
割合が低かったのが新聞であったということは大変興味深い結果であ
った。確かにイメージのみで投票していたのはテレビの選挙報道を情報
源としていた人であった。しかし、それ以外の結果からはテレビの選挙
報道と比べて新聞の選挙報道を情報源としていた人がよりイメージで
の投票を行っていた可能性もあると考えられる。新聞の選挙報道がイメ
ージでの投票を促すかどうかについて本稿では全く言及しなかったの
が残念であった。またイメージ以外での投票を最も行っていたのがテレ
ビのみを情報源として役立ったと答える人であったことから、イメージ
以外で投票行動に何らかの影響を与えている可能性があるだろう。今回
はそちらを検証することは出来なかったが、テレビの選挙報道が投票行
動に与える影響についてはまだ検証の余地があると考える。
上記のように仮説は実証されず多くの課題が残る検証となったが、メ
ディアの選挙報道が投票行動に与える影響については今後も追究して
いきたい。
注
(1)明るい選挙推進協会の第 44 回衆議院議員総選挙の実態(発行 平成
18 年 3 月)の質問項目 Q8 の回答結果を参照
2
( )ここで使用したデータは「第 44 回衆議院議員総選挙についての意識
調査」
(平成 22 年 3 月 財団法人明るい選挙推進協会)である。同
志社大学・法学部の西澤由隆先生のご指導と便宜により利用ができ
ました。このデータを公開・寄託され、利用できるようにしてくだ
さった先生方に感謝いたします。
(3)平野(2007)は第 44 回衆議院議員総選挙における郵政民営化問題は
具体的な政策というよりも小泉に対する信任・不信任のシンボルと
して機能していたと考えている(平野 2007)
。平野はこのことにつ
いて記述している同章で争点態度と争点投票について述べている。
<補遺>
分析に使用した質問とコード
●イメージでの投票
・質問
あなたは、小選挙区で、候補者を選ぶとき、どういう点を重く見て投票
する人を決めたのですか。
(M.A.)
(ア)地元の利益を考えて spnt01
49
(イ)自分と同じような職業の利益を考えて spnt02
(ウ)国全体の政治について考えて spnt03
(エ)政党間の勢力バランスを考えて spnt04
(オ)候補者の政策や主張を考えて spnt05
(カ)候補者の人柄を考えて spnt06
(キ)候補者の属する党を考えて spnt07
(ク)候補者の属する党の党首を考えて spnt08
(ケ)テレビや新聞、雑誌などを通じて、なんとなく親しみを感じてい
るから spnt09
(コ)家族や知人のすすめだから spnt10
(サ)どれとはいえない spnt11
その他 spnt12
わからない spnt13
・コード
(サ)とその他とわからないは欠損値として除いた。上記(ア)~(シ)
までを「イメージのみ=1、イメージも含む=2、イメージ以外=3」
に再コードした。イメージのみは(カ)
(キ)
(ク)
(ケ)から 1 個以
上選択した人、イメージも含むは(ア)
(イ)
(ウ)
(エ)
(オ)
(コ)
から 1 個以上かつ(カ)
(キ)
(ク)
(ケ)から 1 個以上選択した人、
イメージ以外は(ア)
(イ)
(ウ)
(エ)
(オ)
(コ)から 1 個以上選択
した人である。
●情報源
・質問
今回の選挙で、あなたが役に立ったものがこの中にありましたら、全部
おっしゃってください(M.A.) uinfo01-31
(ア)候補者の経歴放送
(イ)政党の政見放送
(ウ)政党のテレビスポット広告
(エ)テレビの報道
(オ)候補者の経歴放送(ラジオ)
(カ)政党の政見放送(ラジオ)
(キ)政党のラジオスポット広告
(ク)ラジオの報道
(ケ)選挙広報
(コ)候補者の新聞広告
(サ)政党の新聞広告
(シ)新聞報道
(ス)週刊誌・雑誌の報道
(セ)候補者のビラ
(ソ)掲示場にはられた候補者のポスター
(タ)政党のビラ・ポスター
(チ)候補者の葉書
(ツ)政党の葉書
(テ)政党の機関紙
50
(ト)政党のマニフェスト
(ナ)党首討論会
(ニ)公開討論会
(ヌ)個人演説会
(ネ)政党の演説会
(ノ)候補者の街頭演説
(ハ)政党の街頭演説
(ヒ)連呼
(フ)インターネット・ホームページ上の選挙情報
(ヘ)携帯上の選挙情報(iモード等)
この中のどれも見聞きしない
わからない
・コード
このなかのどれも見聞きしないとわからないは欠損値扱い。上記(ア)
~(ヘ)を「テレビのみ=1、テレビとその他(新聞以外)=2、テ
レビと新聞とその他=3、新聞とその他(テレビ以外)=4、新聞の
み=5」に再コードした。テレビのみは(ア)(エ)を 1 個以上選択した
人、テレビとその他(新聞以外)は(ア)
(エ)を 1 個以上かつ(シ)
を除くそれ以外から 1 個以上選択した人、テレビと新聞とその他は
(ア)
(エ)を 1 個以上かつ(シ)かつ(ア)
(エ)
(シ)以外のその
他から 1 個以上選択した人、新聞とその他(テレビ以外)は(シ)
かつ(ア)
(エ)以外のその他から 1 個以上選択した人、新聞のみは
(シ)のみを選択した人である。
●支持政党
・質問
あなたは、ふだん何党を支持していらっしゃいますか。
partsupt1-11
自由民主党
民主党
公明党
日本共産党
社会民主党
国民新党
新党日本
新党大地
その他
支持政党なし
わからない
・コード
わからないは欠損値扱い。上記の回答コード「1.自民党、2.民主党、3.
公明党、4.日本共産党、5.社会民主党、6.国民新党、7.新党日
本、8.新党大地、9.その他、10.支持政党なし」を「支持政党、
あり=1、なし=2」に再コード。
51
●シンタックス
recode spnt01 (1=1)(else=0) into spnt001.
recode spnt02 (1=1)(else=0) into spnt002.
recode spnt03 (1=1)(else=0) into spnt003.
recode spnt04 (1=1)(else=0) into spnt004.
recode spnt05 (1=1)(else=0) into spnt005.
recode spnt06 (1=1)(else=0) into spnt006.
recode spnt07 (1=1)(else=0) into spnt007.
recode spnt08 (1=1)(else=0) into spnt008.
recode spnt09 (1=1)(else=0) into spnt009.
recode spnt10 (1=1)(else=0) into spnt010.
compute イメージでの投票=-9.
missing values イメージでの投票(-9).
if ((spnt006=1 or spnt007=1 or spnt008=1 or spnt009=1)) and
(spnt001=0 and spnt002=0 and spnt003=0 and spnt004=0 and
spnt005=0 and spnt010=0) イメージでの投票=1.
if ((spnt006=1 or spnt007=1 or spnt008=1 or spnt009=1)) and
((spnt001=1 or spnt002=1 or spnt003=1 or spnt004=1 or
spnt005=1 or spnt010=1)) イメージでの投票=2.
if (spnt001=1 or spnt002=1 or spnt003=1 or spnt004=1 or spnt005=1
or spnt010=1) and ((spnt006=0 and spnt007=0 and spnt008=0
and spnt009=0)) イメージでの投票=3.
compute 情報源=-9.
missing values 情報源(-9).
if (((uinfo01=1) or (uinfo04=1)) and (uinfo12=2)) and (uinfo02=2) and
(uinfo03=2) and (uinfo05=2) and (uinfo06=2) and(uinfo07=2) and
(uinfo08=2) and (uinfo09=2) and (uinfo10=2) and (uinfo11=2) and
(uinfo13=2) and (uinfo14=2) and (uinfo15=2) and (uinfo16=2) and
(uinfo17=2) and (uinfo18=2) and (uinfo19=2) and (uinfo20=2) and
(uinfo21=2) and (uinfo22=2) and (uinfo23=2) and (uinfo24=2) and
(uinfo25=2) and (uinfo26=2) and (uinfo27=2) and (uinfo28=2) and
(uinfo29=2) and (uinfo30=2) and (uinfo31=2) 情報源=1.
if (((uinfo01=1) or (uinfo04=1)) and (uinfo12=2)) and ((uinfo02=1) or
(uinfo03=1) or (uinfo05=1) or (uinfo06=1) or (uinfo07=1) or
(uinfo08=1) or (uinfo09=1) or (uinfo10=1) or (uinfo11=1) or
(uinfo13=1) or (uinfo14=1) or (uinfo15=1) or (uinfo16=1) or
(uinfo17=1) or (uinfo18=1) or (uinfo19=1) or (uinfo20=1) or
(uinfo21=1) or (uinfo22=1) or (uinfo23=1) or (uinfo24=1) or
(uinfo25=1) or (uinfo26=1) or (uinfo27=1) or (uinfo28=1) or
(uinfo29=1) or (uinfo30=1) or (uinfo31=1)) 情報源=2.
if ((uinfo01=1) or (uinfo04=1)) and (uinfo12=1) 情報源=3.
52
if (((uinfo01=2) and (uinfo04=2)) and (uinfo12=1)) and ((uinfo02=1) or
(uinfo03=1) or (uinfo05=1) or (uinfo06=1) or (uinfo07=1) or
(uinfo08=1) or (uinfo09=1) or (uinfo10=1) or (uinfo11=1) or
(uinfo13=1) or (uinfo14=1) or (uinfo15=1) or (uinfo16=1) or
(uinfo17=1) or (uinfo18=1) or (uinfo19=1) or (uinfo20=1) or
(uinfo21=1) or (uinfo22=1) or (uinfo23=1) or (uinfo24=1) or
(uinfo25=1) or (uinfo26=1) or (uinfo27=1) or (uinfo28=1) or
(uinfo29=1) or (uinfo30=1) or (uinfo31=1)) 情報源=4.
if (((uinfo01=2) and (uinfo04=2)) and (uinfo12=1)) and (uinfo02=2)
and (uinfo03=2) and (uinfo05=2) and (uinfo06=2) and(uinfo07=2)
and (uinfo08=2) and (uinfo09=2) and (uinfo10=2) and (uinfo11=2)
and (uinfo13=2) and (uinfo14=2) and (uinfo15=2) and (uinfo16=2)
and (uinfo17=2) and (uinfo18=2) and (uinfo19=2) and (uinfo20=2)
and (uinfo21=2) and (uinfo22=2) and (uinfo23=2) and (uinfo24=2)
and (uinfo25=2) and (uinfo26=2) and (uinfo27=2) and (uinfo28=2)
and (uinfo29=2) and (uinfo30=2) and (uinfo31=2) 情報源=5.
value labels イメージでの投票 1'イメージのみ' 2'イメージも含む'3'イ
メージ以外'.
fre イメージでの投票.
value labels 情報源 1'テレビのみ' 2'テレビとその他(新聞以外)'3'テレ
ビと新聞とその他'4'新聞とその他(テレビ以外)'5'新聞のみ'.
fre 情報源.
fre partsupt.
recode partsupt (1 thru 9=1)(10=2)(11=9) into 支持政党.
fre 支持政党.
missing values 支持政党 (9).
value labels 支持政党 1'あり' 2'なし'.
crosstabs tables=情報源 by イメージでの投票 by 支持政党
/cells count row
/statistics chisq.
<参考文献>
・池田謙一 2007. 『政治のリアリティと社会心理:平成小泉政治のダ
イナミックス』 木鐸社.
・谷藤悦史 2005. 『現代メディアと政治―劇場社会のジャーナリズム
と政治』 一藝社.
・平野浩 2007. 『変容する日本の社会と投票行動』 木鐸社.
・三宅一郎 1989. 『投票行動』 東京大学出版会.
53
6
投票結果が有権者に
与える影響
山田 紗生
1. はじめに
数ある政治参加の中で最も人々の参加度が高いものは投票である。蒲
島郁夫(1988)によれば、一般的に「政府の政策決定に影響を与えるべ
く意図された一般市民の活動」を政治参加と呼び、具体的には「投票、
選挙活動、地域活動、個別的な接触」の 4 つに分類される。またその中
で投票が最もポピュラーな理由としては「他の政治的活動と比べると、
参加コストが非常に少ない」ことがまず挙げられる。加えて蒲島は投票
の結果の影響力についても言及している。
「投票の結果は、投票した人
であれ、棄権した人であれ、投票権を持たない人であれ、その政策に反
対した人であれ、すべての人に影響を与えるといった集合的なもの」で
あり、その効果の及ぶ範囲の広さからも投票は政治参加の手段として選
ばれやすい(蒲島 1988, 3-9)
。
投票の結果が政治家に与える影響が大きいことは明らかである。なぜ
なら投票の結果という形で有権者の支持を得ることではじめて政治家
は代議士として議会に参加することが許可されるからである。逆を言え
ば有権者の支持を得られなかった政治家に対しては、投票の結果は代議
士としての活動を禁じる圧力となるからである。しかし投票の結果が影
響を与えるのは政治家だけではない。投票の結果の大部分を生み出して
いるのは他ならぬ有権者だが、この有権者自身に対しても投票の結果は
影響力を与えているのではないだろうか。
三宅一郎(1989)は、選挙結果が有権者に及ぼす影響として、
「有権
者個人の参加経験の自分自身へのフィードバック」を挙げている(三宅
1989, 241)
。つまり選挙に参加することで有権者は何らかの「フィード
バック」を得ているのだと三宅は指摘している。投票した後の有権者に
とって、自分が投票した候補者の当落こそが最も明白で、唯一把握する
ことの出来る投票の結果だろう。つまり有権者にとっての「フィードバ
ック」とは、この候補者の当落のことではないだろうか。有権者が投票
した候補者が当選した場合、有権者は当選票を投じたことになるし、ま
た逆は死票を投じたと言える。本稿ではこの当選票・死票という、投票
したことによって生じた票の結果を投票結果と呼び分析の対象とする。
投票棄権の要因については蒲島(1998)が以下のように述べている。
54
有権者が投票するか棄権するかを決める際に考えられる要因は「政治意
識に起因するもの」
・
「政党の行動に起因するもの」
・
「気象的要因」の 3
つにわけられ、その中でも始めに挙げた政治意識に着目して考えた場合、
以下のようなモデルが想定できる(蒲島 1998, 222)
。
図 1 投票-棄権モデルの構造
政治的
ネットワーク
社会的
属性
性別
年齢
所得
教育
職業
居住年数
都市規模
フィードバック効果
社会的心
理要因
ネットワーク
候補者近接度
候補者接触
メディア接触
政治的議論
組織加入
非政府組織
後援会
出典:蒲島(1998)をもとに作成
(太字表記の部分は筆者が加筆)
内閣評価
政治的信頼
シニシズム
政治的関心
伝統的価値
制度信頼
有効性感覚
投票義務感
争点意識
政党支持強
度
投票
結果
当選票
死票
投票―棄権
図 1 からわかるように投票・棄権を決定する際に、最終的に要因とし
て有権者に働きかけるのは社会的心理要因である。先に述べた投票結果
が「フィードバック」として有権者に影響を与えるとすれば、その影響
を受けるのはこの社会的心理要因ではないのだろうかと私は考えてい
る。なぜなら社会的心理要因は基本的に有権者個人が持つものであり、
心理的なものなので最も外部からの影響を受けやすく、またその立場を
変えることも容易である。しかしその他の 3 つは、フィートバックを受
けたとしても有権者個人だけのものではないため変えることが難しい
ものや、変えるにしても時間のかかるものである。だからこそフィード
バック効果が見られるのは、社会的心理要因だと考えた。
以上の内容をふまえて、本稿では投票結果が有権者の心理や選挙参加
に与える影響を検証するべく「投票結果は有権者にどのような影響を与
えるのか」というリサーチクエスチョンを設定する。
また以上のことを論じるために、本稿は以下の構成をとる。まず 2 節
でリサーチクエスチョンに対する仮説とそのメカニズムを紹介する。次
55
に 3 節で分析手順を示し、4 節で分析結果を提示し、5 節では本稿の総
括をおこなう。
2. 仮説とそのメカニズム
本稿では、投票結果が影響を与えるものとして、選挙参加・政治的関
心・政治満足度・選挙有効性感覚・投票義務感の 5 つを挙げて検証する。
5 つそれぞれについて投票結果との因果関係があるかを確認する。以下
で投票結果が 5 つの変数にどのような影響を与えると私が考えているの
かを説明する。
図 2 分析モデル図
選挙参加 (t)
政治的関心(t)
投票結果(t-1)
政治満足度(t)
選挙有効性感覚(t)
投票義務感(t)
※t, t-1 は時系列を表す
2-1. 選挙参加
先回の選挙において有権者が投票しその票が当選票となった場合、有
権者は次回も選挙に参加する傾向にあると考える。一方で票が死票とな
った場合、有権者は次回の選挙は棄権する傾向にあると考える。当選票
を投じた有権者は自らの投票が政治に反映されたと感じ、選挙をポジテ
ィブな思い出として捉えるだろう。またそうすると次回も選挙に参加し
て影響を与えようと前向きな気持ちになるのではないだろうか。反対に
死票だった場合は自らの意見は反映されなかったため、投票経験はネガ
ティブな思い出となり、次回の投票は避ける傾向にあるのではないかと
考える。
2-2. 政治的関心
先回の選挙において有権者が投票しその票が当選票となった場合、
「政治的関心」は高くなり、反対に死票だった場合は政治的関心が低く
56
なると考える。票が当選票となった有権者は、自らの票が政治に反映さ
れたために、投票した候補者と政治がその後どうなるか気になり、より
関心を持つのではないだろうか。一方で票が死票となった有権者は、自
らの意思とは違った形で動く政治に対しての興味関心を失う傾向にあ
るのではないかと考える。
2-3. 政治満足度
先回の選挙において有権者が投票しその票が当選票となった場合、
「政治満足度」は高くなり、死票だった場合は低くなると考える。有権
者にとって当選票だったということは自らの意見が政治に反映された
ということで、自分の意見が反映されている現在の政治に対しても満足
感を抱くのではないだろうか。一方で死票だった場合は、自らの意見が
通らなかった政治に対して、思い通りにならなかったということで不満
を抱く傾向にあると考える。
2-4. 選挙有効性感覚
先回の選挙において有権者が投票しその票が当選票となった場合は
「選挙有効性感覚」が高くなり、反対に死票だった場合は低くなると考
える。先行研究によると、政治的有効性感覚は 2 種類に分けられる。具
体的には「自分自身の政治的能力に関する評価である内的有効性感覚」
と「政府の応答性に対する評価・信頼感である外的有効性感覚」の 2 つ
である(加藤・岩渕 2005, 161)
。今回の分析においては、投票結果の
フィードバック効果について考えたいので、自分自身の政治的能力につ
いて考える前者の方が分析するには適切だと私は判断した。その中でも
ここでは選挙における自らの影響力について考える有権者の感覚を選
挙有効性感覚と定義して仮説に挙げた。
メカニズムとしては以下のように考える。有権者が投じた票が当選票
となったということは自分の投票結果が実際の政治に反映されたとい
うことなので、選挙に対して肯定的なイメージを持つことが出来るだろ
う。一方で票が死票となった場合は、選挙という手段を用いたにもかか
わらず自分の意見が政治に通らなかったということなので、選挙の意義
について懐疑的になるのではないかと考える。
2-5. 投票義務感
先回の選挙において有権者が投票しその票が当選票となった場合、
「投票義務感」は高くなり、死票となった場合は低くなると考える。こ
こでいう投票義務感というのは投票という行為を義務と感じているか
権利と感じているかという質問をもとに分析する。当選票を投じた有権
者は、選挙において自分の意見が政治に反映されたこととなり、政権の
一部を担っているという認識が生まれるだろう。ここから投票に対して
も、票が死票となった有権者よりは義務的なものだと捉える傾向がある
57
のではないかと考える。1
3. 分析枠組み
3-1. 使用するデータ
本稿では、上記の仮説を実証するために、JESⅢを使用する。2 JES
Ⅲは 2001 年から 2005 年まで選挙の前後にわたってパネル調査を実施
しており、同じ有権者の投票行動・政治参加を数年にわたって追い続け
て調査している。そのため、同一有権者の連続した行動を確認したい今
回の分析でこのデータを使用することとした。また今回は 2001 年参議
院議員選挙(前後)
、2003 年衆議院議員選挙(前後)
、2004 年参議院議
員選挙(前後)の 6 種類のデータを使用する。
なお独立変数としてあげている投票結果に関するデータに関しては、
私自身が以下の手順で作成し用いる。まず JESⅢの調査には回答者が投
票した小選挙区の候補者名を尋ねる質問がある。ここで得られたデータ
をもとに、その候補者が当選していたら当選票を投じたということで1、
落選なら死票を投じたということで 0 といった形で変数を作成した。詳
しくは作業定義の部分でまた触れたい。
3-2. 分析手順と予想される結果
分析は以下の手順で行う。
投票結果が先ほど挙げた 5 つの変数にどのような影響を与えているの
か、それぞれの変数ごとに 5 つのクロス表分析で確認する。具体的には
独立変数が投票結果、従属変数が選挙参加・政治的関心・政治満足度・
選挙有効性感覚・投票義務感の 5 つである。仮説が正しければ、有権者
が投じた票が当選票となった場合、これら 5 つの変数はプラスに働き強
化される。反対に死票だった場合は、5 つの変数はマイナスに働く。
なおいずれの分析も先回の選挙結果が次回以降の有権者自身の行
動・心理にどのような影響を与えるかということが見たいので、2 つの
選挙の連続した投票行動を見る。そこで本稿では以下の 3 つの組み合わ
せを用意した。それらは 2001 年参議院議員選挙(以下、2001 年参院選)
と 2003 年衆議院議員選挙(以下、2003 年衆院選)
、2001 年参院選と
2004 年参議院議員選挙(以下、2004 年参院選)
、2003 年衆院選と 2004
年参院選の 3 パターンである。これは衆議院と参議院という 2 つの種類
が異なる議員選挙があることを考慮したためである。3 つのパターンを
見ることで、投票結果が有権者に影響を与える際に衆参の種類が異なる
ことも何か関係するのか確認したい。なお衆議院選挙同士のパターンは
用意できなかったが、これは以下のような理由がある。JESⅢにおける
衆議院議員選挙のデータは 2003 年と 2005 年の選挙があり分析自体は
可能だったが、
2005 年の衆議院議員選挙は小泉内閣の郵政選挙であり、
影響を確認するには特殊なケースであり除いたほうがいいと私が判断
58
したためである。
図 3 分析に際して用意した組み合わせ
◇参院選(2001)―◆衆院選(2003)
◇参院選(2001)―◇参院選(2004)
◆衆院選(2003)―◇参院選(2004)
※データの都合上、衆院選同士の分析はできず。
3-3. 作業定義
【独立変数】
投票結果
今回の分析にあたり、ほとんどは JESⅢのデータをそのまま使用した
が、先述の通り独立変数として挙げている投票結果に関しては私自身が
JESⅢのデータをもとに新たに作成した。具体的には JESⅢの調査で
「どの候補者に投票しましたか」という質問があり、回答者に候補者名
を記入してもらっている。そこで Yahoo!みんなの政治というホームペー
ジを参考に、その候補者が当選したのかどうかを私が確認した。そして
その候補者が当選していたら 1、落選なら 0 という形にコード化した。
【従属変数】
投票行動
「選挙の投票に行きましたか」という質問への回答をもとに定義する。
投票日に投票した、不在者投票あるいは期日前投票で投票した場合を 1、
投票しなかった場合を 0 に再コード化した。なお答えないは欠損値とし
て分析対象から除外した。
政治的関心
「あなたは政治上のできごとに、どれくらい注意を払っていますか」
という質問への回答をもとに定義する。有権者は「いつも注意を払って
いる」から「全く注意していない」まで 4 段階で回答しており元々のコ
ードから 1~4 に関してはそのまま使用した。なおわからない・答えな
いは欠損値として分析対象から除外した。
政治満足度
「あなたは現在の政治に対してどの程度満足していますか」という質
問に対する回答で定義する。有権者は「かなり満足している」から「か
なり不満である」まで 5 段階で回答しており、1 から 5 までは元データ
のコードをそのまま使用した。なおわからない・答えないは欠損値とし
て分析対象から除外した。
59
選挙有効性感覚
「選挙では大勢の人が投票するのだから、自分一人ぐらい投票しても
しなくてもどちらでもかまわない」という考えについての質問回答をも
とに定義する。有権者は「そう思う」から「そうは思わない」まで 5 段
階で回答しておりコードも順に1から 5 となっていた。しかし本分析で
は有効性感覚の高さを測りたいため、
「そうは思わない」を 1 とし、
「そ
う思う」を 5 とする形で並び替え再コード化した。なお、わからない・
答えないは欠損値として分析対象から除外した。
投票義務感
「投票に行くことについて、この中からあなたのお気持ちに最も近い
ものを 1 つあげてください」という質問への回答をもとに定義する。
「投
票へ行くことは有権者の義務」と答えた人を1、
「有権者はできるだけ
選挙に参加したほうがよい」と答えた人は 2、
「必ずしも選挙に参加しな
くてもよい」
と答えた人は 3 とされていた元データをそのまま使用した。
わからない・答えないは欠損値として分析対象から除外した。
4. 分析結果
4-1. 選挙参加
表 3 2001 年参院選の投票結果と 2003 年衆院選の選挙参加 (%)
2003年の選挙参加
2001年の投票結果
投票
棄権
合計
(N)
当選票
97.0
3.0
100.0
(368)
死票
98.8
1.2
100.0
(162)
カイ二乗値=1.447 危険率=.362
表 4 2001 年参院選の投票結果と 2004 年参院選の選挙参加 (%)
2004年の選挙参加
2001年の投票結果
投票
棄権
合計
(N)
当選票
95.1
4.9
100.0
(326)
死票
97.8
2.2
100.0
(137)
カイ二乗値=1.811 危険率=.210
60
表5 2003年参院選の投票結果と2004年参院選の選挙参加 (%)
2004年の選挙参加
2003年の投票結果
投票
棄権
合計
(N)
当選票
93.4
6.6
100.0
(632)
死票
93.8
6.2
100.0
(481)
カイ二乗値=0.75 危険率=.807
まずは表 3 から表 5 で投票結果と選挙参加の相関関係について結果を
示した。結論から述べると、いずれの結果も危険率が高く仮説は支持さ
れなかった。また 3 つの表すべてに言えることだが、投票した人の割合
を見たときに、前回の投票での結果が当選票であったとしても死票であ
ったとしても、投票に行った有権者が 90%以上を占めている。つまり投
票結果がどうであったかにかかわらず、先回の投票に参加した有権者は
選挙に参加する傾向にあるのだということがわかる。
4-2. 政治的関心
表6 2001年参院選の投票結果と2003年衆院選の政治的関心 (%)
2003年の政治関心
強い
2001年の
投票結果
やや強い
やや弱い
弱い
合計
(N)
当選票
18.9
50.8
25.5
4.8
100.0
(392)
死票
19.6
51.4
26.8
2.2
100.0
(179)
カイ二乗値=2.202 危険率=.532
表7 2001年参院選の投票結果と2004年参院選の政治的関心 (%)
2004年の政治関心
強い
2001年の
投票結果
やや強い
やや弱い
弱い
合計
(N)
当選票
18.2
53.9
22.3
5.6
100.0
(319)
死票
19.7
52.8
23.9
3.5
100.0
(142)
カイ二乗値=1.156 危険率=.764
61
表8 2003年衆院選の投票結果と2004年参院選の政治的関心 (%)
2004年の政治関心
強い やや強い やや弱い 弱い
2003年の
投票結果
合計
(N)
当選票
14.6
51.8
26.8
6.8 100.0
(650)
死票
22.3
47.9
24.6
5.2 100.0
(484)
カイ二乗値=11.716 危険率=.008
表 6 から表 8 では投票結果と政治的関心の相関関係について示した。
2001 年と 2003 年、2001 年と 2004 年の組み合わせパターンでは仮説
は支持されなかった。分布を見ると投票結果が当選票だったか死票だっ
たかにかかわらず、政治的関心が強い・やや強い人が半数以上いること
がわかる。
しかし 2003 年の投票結果と 2004 年の政治的関心(表 8)では、危険
率がほぼ 0 に近く、有意な結果となった。実際に分布を見てみると、政
治関心が強いという項目において、当選票を投じた有権者より死票の有
権者の方が、政治的関心の強い人の割合が多いことが分かる。つまり当
初私が想定していたメカニズムとは逆の結果が生じている。ここから考
えられるのは、票が死票となった有権者は自らの意見が政治に反映され
なかったことにより、現在の政治と自らの考えのギャップに気づかされ
る。そして今まではあまり関心を払っていなかったかもしれない政治に
対して、より強い関心を抱き注目するようになるのではないだろうか。
実際に有意ではなかった表 6 と表 7 を見ても、
死票だった有権者の方が、
当選票だった有権者と比べて、政治的関心が強い人の分布の割合がわず
かながらに多い。つまり自分の投じた票が死票となって政治にはいかさ
れなかったという事実のインパクトが、死票だった場合には当選票に比
べて大きいからこのような結果になるのではないかと考えた。
しかし他の 2 パターンに比べて母数が圧倒的に多い 2003 年の 2004
年の組み合わせだけが有意となったこと、また全体的な割合の分布は 3
パターンともそれほど変わらないことから、この分析結果は偶然の結果
だとみなした方がよいとも考えられる。
62
4-3. 政治満足度
表9 2001年参院選の投票結果と2003年衆院選の政治満足度 (%)
2003年の政治満足度
2001年の 当選票
投票結果 死票
かなり
やや
どちらでも
やや
かなり
満足
満足
ない
不満
不満
合計
(N)
1.0
26.0
30.3
29.0
13.6
100.0
(389)
0.6
17.9
29.6
31.8
20.1
100.0
(179)
カイ二乗値=7.365 危険率=.118
表10 2001年参院選の投票結果と2004年参院選の政治満足度 (%)
2004年の政治満足度
2001年の 当選票
投票結果 死票
かなり
やや
どちらでも
やや
かなり
満足
満足
ない
不満
不満
合計
(N)
1.9
30.2
29.0
26.8
12.1
100.0
(321)
0.7
17.7
27.0
33.3
21.3
100.0
(141)
カイ二乗値=13.717 危険率=.008
表11 2003年衆院選の投票結果と2004年参院選の政治満足度 (%)
2004年の政治満足度
2003年の 当選票
投票結果 死票
かなり
やや
どちらでも
やや
かなり
満足
満足
ない
不満
不満
合計
(N)
1.1
26.9
28.2
31.0
12.8
100.0
(648)
1.0
19.4
24.6
34.7
20.3
100.0
(479)
カイ二乗値=18.442 危険率=.001
表 9 から表 11 では、投票結果と政治満足度の相関関係について示し
た。2001 年と 2003 年の組み合わせパターンに関しては、危険率が 10%
水準を大きく上回り仮説は支持されなかった。しかし、2001 年と 2004
年(表 10)
、2003 年と 2004 年(表 11)パターンの政治満足度に関して
は、かなり満足・満足と回答した有権者の割合を当選票と死票で比較し
た際に、当選票の有権者の方が政治満足度の高いことが明らかとなった。
有意ではなかったが 2001 年と 2003 年の組み合わせも同じような分布
となっている点も考慮すると、やはり仮説で述べたとおり、自らの意見
が政治に反映されると有権者の政治満足度が高まることがこの結果か
ら言えると思う。
63
4-4. 選挙有効性感覚
表12 2001年参院選の投票結果と2003年衆院選の選挙有効性感覚 (%)
2003年の選挙有効性感覚
強い
2001年の 当選票
投票結果 死票
合計
やや
どちらでも
やや
強い
ない
弱い
弱い
(N)
65.8
15.7
8.5
6.9
3.1
100.0
(389)
72.7
12.5
4.5
8.5
1.7
100.0
(176)
カイ二乗値=5.555 危険率=.235
表13 2001年参院選の投票結果と2004年参院選の選挙有効性感覚 (%)
2004年の選挙有効性感覚
強い
2001年の 当選票
投票結果 死票
合計
やや
どちらでも
やや
強い
ない
弱い
弱い
(N)
65.0
19.1
6.3
8.1
1.6
100.0
(320)
73.4
16.1
4.2
4.2
2.1
100.0
(143)
カイ二乗値=4.636 危険率=.327
表14 2003年衆院選の投票結果と2004年参院選の選挙有効性感覚 (%)
2004年の選挙有効性感覚
強い
2003年の 当選票
投票結果 死票
合計
やや
どちらでも
やや
強い
ない
弱い
弱い
(N)
65.0
18.9
6.4
7.3
2.5
100.0
(645)
66.9
15.9
7.9
6.4
2.9
100.0
(484)
カイ二乗値=2.953 危険率=.566
表 12 から表 14 では、投票結果と選挙有効性感覚の相関関係について
結果を示した。この仮説についてもすべて危険率が高く実証はされなか
った。3 つの表すべてにおいて選挙有効性感覚が強い有権者の割合を見
ると、投票結果が当選票だったか死票だったかに関係なく、選挙有効性
感覚が高い有権者が半数以上だということがこの結果から分かる。
64
4-5. 投票義務感
表15 2001年参院選の投票結果と2003年衆院選の投票義務感 (%)
2003年の投票義務感
行くべき
2001年の 当選票
投票結果 死票
行ったほうがいい 行かなくてもいい
合計
(N)
66.4
30.0
3.6
100.0
(393)
70.9
26.3
2.8
100.0
(179)
カイ二乗値=1.198 危険率=.549
表16 2001年参院選の投票結果と2004年参院選の投票義務感 (%)
2004年の投票義務感
行くべき
2001年の 当選票
投票結果 死票
行ったほうがいい 行かなくてもいい
合計
(N)
65.4
29.3
5.3
100.0
(321)
67.8
28.7
3.5
100.0
(143)
カイ二乗値=.775 危険率=.679
表17 2003年衆院選の投票結果と2004年参院選の投票義務感 (%)
2004年の投票義務感
行くべき
2003年の 当選票
投票結果 死票
行ったほうがいい 行かなくてもいい
合計
(N)
65.5
30.5
4.0
100.0
(649)
61.1
32.5
6.4
100.0
(486)
カイ二乗値=4.305 危険率=.116
最後に表 15 から表 17 では、投票結果と投票義務感の相関関係を示し
ている。なおこの表では投票義務感について、義務だと答えた人を「行
くべき」
、
「義務ではないが行く方がいい」と答えた人を「行った方がい
い」
、
「権利だ」と答えた人を「行かなくてもいい」というように示して
いる。結果を見る限りこちらも危険率は高く、仮説は支持されなかった。
「行くべき」と答えた有権者の割合分布を見ればわかるように、投票結
果が当選票だったか死票だったかに関わらず、選挙には行くべきだと考
える有権者が半数以上だと言うことが分かった。
5. おわりに
本稿では「投票結果はその後の投票行動にどのような影響を与えるの
か」というリサーチクエスチョンに対し、選挙参加・政治的関心・政治
満足度・選挙有効性感覚・投票義務感にプラスの影響を与えるという仮
65
説を立てて検証した。その結果ほとんどの仮説は支持されなかったもの
の以下の 3 つに関しては危険率 10%水準で考えると有意な結果となっ
た。2001 年選挙の投票結果と 2004 年の政治満足度(表 10)
、2003 年
選挙の投票結果と 2004 年の政治的関心(表 8)
・政治満足度(表 11)の
3 つである。
政治的関心に関しては、死票を投じた有権者の方が政治的関心を高め
るという結果になり、当初予想していた当選票が政治的関心を高めると
いう結果とは逆となった。これについては前述の通り、死票となること
で現在の政治と自分の意見とのギャップに気づき、そのことがきっかけ
でより政治への関心を抱くのではないかと考えている。しかし 2003 年
の選挙と 2004 年の政治的関心(表 8)だけ相関関係が見られたので、
これは偶然の結果である可能性も否めないことは指摘しておく。
また政治満足度に関しては表 10 と表 11 において、仮説で述べたとお
り当選票を投じた有権者の方が政治満足度は高いという結果になった。
2001 年選挙の投票結果と 2003 年の政治満足度(表 9)では危険率が高
く有意とは言えなかったが、値の分布は他の 2 パターンと類似していた
ことを考えると、票が当選票となった有権者は自分の意見が政治に反映
されたとして政治満足度を高める傾向にあるとはいえるかもしれない。
しかし逆に票が死票となった有権者が政治に対して不満を抱くのかど
うかまでは確認できなかった。またこの分析も偶然の結果である可能性
は否めない。
最後に課題としては以下のことが延べられる。まずはデータの関係上、
小選挙区に投票した有権者のみしか確認できなかったという点である。
JESⅢでは回答者の居住地区、つまり選挙区についても回答を得ている
がデータとしては公開されていない。そのため比例代表選挙で有権者が
どの政党に投票したかまでは確認できたものの、それがどの選挙区の候
補者なのかがわからないため、投票結果に関するデータが作成できなか
った。独立変数に投票結果を使用するためにデータ数が限られたものと
なったので、もし有用なデータが見つかれば、今度は比例代表も含めた
確認作業を行いたいと思う。
さらに今回の分析では 3 つのパターンを用意して分析したが、衆議院
選挙同士の組み合わせについては確認できていない。これは前述の通り、
2005 年選挙の特殊性を考慮したからである。JESⅢは 2005 年衆議院議
員選挙についての調査結果もあり分析は可能であったのだが、2005 年
は小泉首相による郵政選挙と呼ばれる特殊な選挙であり、前回の投票結
果に関係なく投票にいった有権者が多いと考えたため分析対象から除
いた。他の 3 つの組み合わせでは相関関係が確認されなかったため可能
性は低いとは思うが、投票結果と投票行動に直接的に相関関係がないと
断定するには、2005 年以外の選挙データを使ってこの点も分析してみ
る必要があるだろう。
66
注
(1)
(2)
ここまで今回の分析で使う 5 つの変数について説明したが、分析
を始める前に一つ注意したい点がある。これらのメカニズムは裏返
せば逆のメカニズムとして指摘することもできるという点である。
たとえば本稿の 2-1 で、票が当選票になった有権者の方が、次回の
選挙に参加する傾向にあると仮説を立てた。その理由としては、当
選票を投じたことがポジティブな思い出となり、次回も投票したい
と思うのではないかと考えたからである。
しかしこの仮説には逆のメカニズムも考えられる。先の理由から
考えるに死票を投じたことはネガティブな思い出となるだろう。そ
してこのネガティブな思い出をひきずって次回の投票にはいかない
という考えが先述のものである。しかし逆にこのネガティブな思い
出を払拭したいと、次回の選挙は積極的に参加する可能性もある。
このように逆のメカニズムを想定することは 5 つの変数すべてにお
いてできる。
だがあくまで本稿では逆のメカニズムの存在も認めつつ、サイク
ルとして強められるのは 2 節で述べた仮説であると考えている。な
ぜなら逆のメカニズムがあるとは言えど、そうした行動をとる人は
選挙や政治に対して何かしら強い思いがある人ではないだろうか。
数年に 1 回しかない選挙に対して継続的に関心を持ち続けている人
は有権者全体で見ればそう多くはないし、割合的には仮説でのべた
メカニズムのように行動する人の方が圧倒的に多いだろうと考えた
ため、今回は 2 節で述べたメカニズムをもとに分析した。
JESⅢとは、
「21 世紀初頭の投票行動の全国的・時系列的調査研
究(JESⅢ SSJDA 版)
,2001-2005」で、JESⅢ研究会が実施され
た世論調査である。その個票データについて、東京大学社会科学研
究所附属日本社会研究情報センター・データアーカイブ(Social
Science Japan Data Archive)より教材としての利用許可(申請者:
西澤由隆教授)を得たものを使用した。いずれも、同志社大学・法
学部の西澤由隆先生のご指導と便宜により利用ができた。それぞれ
のデータを公開・寄託され、利用でるようにしてくださった先生方
に感謝いたします。
<補遺>
・分析に使用した質問文とコード
○選挙参加
Q1. さっそくですが、あなたは選挙の投票に行きましたか。この中から
1つだけ答えてください。
1. 投票日に投票した、2. 不在者投票をした、3. 棄権した、4. わから
ない 5. 答えない
SQ2. どの候補者に投票しましたか。
(候補者名を記入)
67
○政治満足度
Q22. あなたは、現在の政治に対してどの程度満足していますか。
1. かなり満足している、2. やや満足している、3. どちらでもない、
4. やや不満である、5. かなり不満である、6. わからない、7. 答え
ない
○政治的関心
Q24. 選挙のある、なしに関わらず、いつも政治に関心を持っている人
もいますし、そんなに関心を持たない人もいます。あなたは政治上
のできごとに、どれくらい注意を払っていますか。この中ではどれ
にあたりますか。
1.いつも注意を払っている、 2. 時々注意を払っている 3. たまに注
意を払っている、4. まったく注意していない、5. わからない、6. 答
えない
○選挙有効性感覚
Q32. 次に、国民と選挙や政治との関わり合いについての意見を読み上
げます。それぞれについてあなたのお気持ちをお答えください。ま
ず、
「(1)選挙では大勢の人が投票するのだから、自分一人ぐらい投
票してもしなくてもどちらでもかまわない」についてはいかがです
か。
(1)選挙では大勢の人が投票するのだから、自分一人ぐらい投票しても
しなくてもどちらでもかまわない
1. そう思う、2. どちらかといえばそう思う、3. どちらともいえない 4.
どちらかといえばそう思わない、5. そうは思わない、6. わからな
い、7. 答えない
○投票義務感
Q4. 投票に行くことについて、
この中からあなたのお気持ちに最も近い
ものを 1 つあげてください。
1.投票へ行くことは有権者の義務であり、当然、選挙に行かなくては
ならない、2. 有権者はできるだけ選挙に参加したほうがよい、3. 投
票に行くかどうかは有権者が決めることなので、必ずしも選挙に参加
しなくてもよい、4. わからない、5. 答えない
・シンタックス
*投票行動
fre e1.
recode e1 (1,2=1)(3=0)(else=-9) into vote2003.
missing values vote2003 (-9).
value labels vote2003 1'投票' 0'棄権'.
fre vote2003.
fre h1.
68
recode h1 (1,2=1)(3=0)(else=-9) into vote2004.
missing values vote2004 (-9).
value labels vote2004 1'投票' 0'棄権'.
fre vote2004.
*選挙参加
crosstabs tables=win2001 by vote2003
/cells count row/statistics=chisq.
crosstabs tables=win2001 by vote2004
/cells count row/statistics=chisq.
crosstabs tables=win2003 by vote2004
/cells count row/statistics=chisq.
*政治的関心
fre d25.
recode d25 (1 thru 4=copy)(else=-9) into inte2003.
missing values inte2003 (-9).
fre inte2003.
fre g27.
recode g27 (1 thru 4=copy)(else=-9) into inte2004.
missing values inte2004 (-9).
fre inte2004.
*政治満足度
fre d23.
recode d23 (1 thru 5=copy)(else=-9) into satis2003.
missing values satis2003 (-9).
fre satis2003.
fre g25.
recode g25 (1 thru 5=copy)(else=-9) into satis2004.
missing values satis2004 (-9).
fre satis2004.
69
*選挙有効性感覚
fre d32x1.
recode d32x1 (1=5)(2=4)(3=3)(4=2)(5=1)(else=-9) into effe2003.
missing values effe2003 (-9).
fre effe2003.
fre g34x1.
recode g34x1 (1=5)(2=4)(3=3)(4=2)(5=1)(else=-9) into effe2004.
missing values effe2004 (-9).
fre effe2004.
*投票義務感
fre d4.
recode d4 (1 thru 3=copy)(else=-9) into duty2003.
missing values duty2003 (-9).
fre duty2003.
fre g4.
recode g4 (1 thru 3=copy)(else=-9) into duty2004.
missing values duty2004 (-9).
fre duty2004.
*政治的関心クロス表
crosstabs tables=win2001 by inte2003
/cells count row/statistics=chisq.
crosstabs tables=win2001 by inte2004
/cells count row/statistics=chisq.
crosstabs tables=win2003 by inte2004
/cells count row/statistics=chisq.
*政治満足度クロス表
crosstabs tables=win2001 by satis2003
/cells count row/statistics=chisq.
crosstabs tables=win2001 by satis2004
70
/cells count row/statistics=chisq.
crosstabs tables=win2003 by satis2004
/cells count row/statistics=chisq.
*選挙有効性感覚クロス表
crosstabs tables=win2001 by effe2003
/cells count row/statistics=chisq.
crosstabs tables=win2001 by effe2004
/cells count row/statistics=chisq.
crosstabs tables=win2003 by effe2004
/cells count row/statistics=chisq.
*投票義務感クロス表
crosstabs tables=win2001 by duty2003
/cells count row/statistics=chisq.
crosstabs tables=win2001 by duty2004
/cells count row/statistics=chisq.
crosstabs tables=win2003 by duty2004
/cells count row/statistics=chisq.
<参考文献>
・蒲島郁夫 1988. 『政治参加』 東京大学出版会.
・三宅一郎 1989. 『投票行動』 東京大学出版会.
・蒲島郁夫 1998. 『政権交代と有権者の態度変容』木鐸社.
・加藤秀治郎・岩渕美克 2005. 『政治社会学』第 2 版 一藝社.
・Yahoo! みんなの政治-2001 参議院選挙結果
http://seiji.yahoo.co.jp/guide/election/sangiin2001/
2013 年 10 月 22 日参照.
・Yahoo! みんなの政治-2004 参議院選挙結果
http://seiji.yahoo.co.jp/guide/election/sangiin2004/
2013 年 10 月 22 日参照.
・第 43 回衆議院議員選挙結果
http://www.geocities.co.jp/HeartLand-PoplPo/5755/shugiin2003.h
tm 2013 年 10 月 24 日参照.
71
4 年生論文
7
組織と政治関心
-どのような組織が政治関心を高めるのか-
浦岡 教之
1. はじめに
組織加入は、政治参加における重要な資源の一つである。Sidney
Verba and Norman H.Nie (1972) によると、市民は組織に加入するこ
とによって、政治的刺激に触れる機会が増える。組織での活動や他の加
入者との交流などを通じて、政治に関心をもち、その結果政治参加する
(Verba and Nie 1972, 174-208)
。
組織加入によってもたらされる政治的刺激として、もっとも容易に想
像できるものが政治的会話だろう。組織加入者は、他の加入者と政治的
会話をすることによって、今まで持っていなかった知識を獲得すること
ができるだろう。また、自分と考え方の異なる相手と会話をすることで、
多角的な視点も身につけられる。政治的会話が政治関心を高めるという
メカニズムは、妥当な判断のように思える。
しかし、本当にどのような組織においても、政治的会話が政治関心を
高めるのだろうか。もし何らかの理由によって、政治的会話が自由かつ
オープンにおこなうことができない状況が生じているとすれば、政治関
心を高めるはたらきは見られないのではないだろうか。
以上から、本稿では、
「どのような組織が政治関心を高めるのか」と
いうリサーチクエスチョンを設定する。なお本稿では、組織の種類では
なく、組織内部の特徴に注目する。すなわち、自治会・町内会や労働組
合、後援会といった組織の種類ではなく、政治的会話がなされているの
か、また上下関係があるのかなどに焦点をあてる。それによって、ある
特定の組織だけではなく、幅広い組織に共通する説を導き出せると考え
るからである。
上記のリサーチクエスチョンに対する答えとして、本稿では、
「政治
的会話があり、かつ水平的な構造をもつ組織は、政治関心を高める」と
いう仮説を提示する。これは、パットナム (1993) が指摘したような組
織構造の違いをヒントに考案したものである。水平的な構造をもつ組織
では、垂直的構造をもつ組織と比べて、政治的会話を自由におこなうこ
とができるので、多様な意見に触れる機会が増えるだろう。そして、そ
の自由な議論を通じて、政治関心は高まるというメカニズムを想定して
いる。結論を先取りして述べるなら、この仮説は支持されなかった。本
73
稿で想定していたほど組織構造が重要な役割を果たしていないことが
わかるのみの結果となった。
以上のことを論じるために、本稿では、次の構成ですすめていく。ま
ず、第 2 節では、政治関心を高める鍵となりうる 2 つの要因を紹介し、
そのメカニズムについて説明する。そして、それらが単独ではなく、掛
け合わされることではじめて政治関心を高めるとする本稿の仮説を提
示する。第 3 節では、使用データや分析手順、各変数の作業定義といっ
た分析枠組みについて紹介する。第 4 節で分析結果を提示して、最後の
第 5 節では本稿のまとめをおこなう。
2. 仮説とそのメカニズム
本節では、政治関心を高める鍵となりうる 2 つの要因に触れ、それぞ
れどのようなメカニズムを想定しているのかについて紹介する。そして、
それらの要因が単独では不十分であり、2 つを掛け合わせてはじめて政
治関心を高めるという本稿の仮説を提示する。
2-1. 政治的会話
上述のように、政治的会話が政治関心を高めるというメカニズムは理
解しやすいものだろう。政治的な会話をすることで、多様な意見に触れ
ることができ、政治的な知識も蓄積されていく。池田 (2002) は、いさ
さか楽観的な考えとしているが、政治的会話によって、
「異なる視点か
ら同じ争点を見る機会」が増加する可能性について言及している (池田
2002, 7) 。政治的会話を通じて、政治に関する知識や多角的な視点を身
につけていくことで、政治関心は高まっていくだろう。
政治関心に対してではないが、政治的会話が政治参加にプラスの効果
を与えることを実証的に確認した研究は、数多く蓄積されている。池田
(1997;2001) は、周囲他者との政治的会話量が多い人ほど、政治参加
行動をとる傾向があることを明らかにした。それに加えて、池田 (2002)
は、スノーボール・パネル全国調査を用いて、集団内の政治的会話とネ
ットワーク他者との政治的会話が、政治参加行動にプラスの効果を与え
ていることも確認した。また、平野 (2002) は、池田 (2002) と同様の
全国調査に基づき、加入団体・グループでの政治的会話が、選挙活動に
プラスの効果を与えていることを明らかにした。
以上のような先行研究を見る限り、政治的会話が政治関心を高めると
いうメカニズムも充分想定できる。従属変数こそ異なっているものの、
政治的会話が政治関心を高めると考えるのは、妥当な判断と言えるだろ
う。
しかし、本当にどのような状況下においても、政治的会話は政治関心
を高めるのだろうか。もし何らかの理由によって、政治的会話が自由か
つオープンにおこなわれない状況が生じているとしたら、政治関心を高
74
めるはたらきはなくなるのではないだろうか。その一つの考えとして、
本稿では、組織構造の違い、すなわち組織構造が垂直的であるか、ある
いは水平的であるかに着目する。
2-2. 組織構造-垂直的か水平的か
パットナム (1993) は、1970 年代のイタリアで、ほぼ同時期に新しい
州制度が導入されたのにもかかわらず、北部と南部の制度パフォーマン
スに著しい差異が見られる理由を分析した。そして、その制度パフォー
マンスの差異は、従来言われてきたような経済的近代性によるものでは
なく、市民的共同体によるものであったことを明らかにした。すなわち、
「互酬性の規範・相互信頼・水平的ネットワーク」といった社会関係資
本が重要な役割を果たしていたのである。
パットナム (1993) によれば、社会関係資本を生み出す際に、組織構
造の違いが重要な意味をもつ。具体的に言うと、組織が垂直的構造なの
か、あるいは水平的構造なのかが、社会関係資本の生成に大きくかかわ
っているということである。
まず、垂直的な構造をもつ組織においては、社会関係資本はほとんど
生成されない。垂直的ネットワーク下では、従属者と支配者は、非対称
的かつ不平等な関係であり、相互疑念をいだきやすい。公的な問題に関
心を寄せるものはほとんどおらず、誰か他の人が勝手にやればよいと考
えるものが多くを占める。また、従属者は、
「搾取されることに対する
自己防衛の壁として情報の出し惜しみ」などをおこなう (パットナム
1993, 翻訳 217)。どれほど緊密なネットワークであったとしても、垂
直的構造をもつ組織では、社会的信頼や協力を維持していくことは難し
いだろう。
一方で、水平的な構造をもつ組織においては、社会関係資本が生成さ
れやすい。水平的ネットワーク下では、個々人は同等の地位・権力を有
している。それゆえ、お互いが信頼し合い、互いに協力しようとする。
公的な問題に対しても関心をもち、自ら積極的に関わろうとする。意見
が対立する者と妥協することも恐れない。水平的構造をもつ組織では、
相互信頼や互酬性の規範が促進されていく。
本稿では、以上で見てきたパットナムによる社会関係資本の生成に関
する組織構造の理論を政治関心に応用する。すなわち、社会関係資本を
政治関心に置きかえて、水平的な組織は政治関心を高めるが、垂直的な
組織には政治関心を高めるはたらきがないとする。
しかし、どのような組織構造であったとしても、それが単独で政治関
心に影響を与えるとは言い難い。1 なぜなら、その組織内における日常
的な会話の中では、政治的な話をする機会はほとんどないと考えられる
からである。Schudson (1997) も、政治的な議論は、対立をひきおこ
しかねないものであり、日常の生ぬるさに適していないと述べている。
たとえ水平的な構造をもつ組織であっても、政治的な会話がないのであ
75
れば、政治関心を高めるとは言えないだろう。つまり、組織構造だけで
は、政治関心に影響を与えると考えるには不十分である。
2-3. 政治的会話×組織構造
以上で見てきたように、政治的会話と組織構造は、それぞれが単独で
政治関心に影響を与えると言うには不十分である。組織内に政治的会話
があっても、自由かつオープンな議論がなされていなければ、政治関心
を高めないだろう。また、水平的な構造であっても、そこで政治的な議
論がなされていなければ、政治関心を高めるはたらきはないだろう。
そこで、本稿では、それら 2 つの要因を掛け合わせて、
「政治的会話
があり、かつ水平的な構造をもつ組織は政治関心を高める」という仮説
を提示する。政治的会話と組織構造という 2 つの要因は、相互に作用し
あってはじめて政治関心に影響を与えると想定する。
政治的会話があり、かつ水平的な構造をもつ組織は、政治関心を高め
るだろう。水平的なネットワーク下においては、地位や権力もほとんど
同じ相手と自由に政治的な議論をおこなうことができる。そこでは、相
互信頼が醸成されていると考えられるので、自身の政治的な意見を躊躇
なく発表することができる。自身とは異なる多様な意見にたくさん触れ
ることができ、政治的な知識も蓄積されていくだろう。それらの会話を
通じて、政治関心も高まっていくと想定することができる。
一方、政治的会話があっても、垂直的構造をもつ組織は、政治関心を
高めないだろう。垂直的なネットワーク下では、地位や権力が異なる相
手と政治的な議論をおこなうことになる。そこでは、水平的な構造をも
つ組織ほど、相互信頼が醸成されていないと考えられるので、情報の出
し惜しみが起きるかもしれない。また、信頼していない相手に対して、
自身の政治的な意見を伝えるとは考えにくい。それらの会話は、表面的
で陳腐なものになり下がってしまうはずである。したがって、垂直的ネ
ットワーク下での政治的会話は、政治関心を高めるはたらきをもたない
だろう。
図 1 仮説のモデル図
政治的会話
組織構造
政治関心
政治的会話×組織構造
76
3. 分析枠組み
3-1. 使用するデータ
本稿では、上記の仮説を実証するために、
「選挙とデモクラシー協会
(JEDS) 」がおこなった「日本人の民主主義観と社会資本に関する調査
-2000」を使用する。2
3-2. 分析手順と予想されうる結果
分析は、フォーマルな組織とインフォーマルな組織にわけておこなう。
フォーマルな組織として、
「自治会・町内会」
・
「PTA」
・
「同業者の団体」
・
「農協」
・
「労働組合」
・
「生協・消費者団体」
・
「ボランティア団体」
・
「住
民運動団体」
・
「市民運動団体」
・
「宗教団体」
・
「学校の同窓会」
・
「政治家
の後援会」の合計 12 の組織を使用する。またフォーマル組織において
は、1 番目と 2 番目に積極的に参加している組織について分析する。一
方、インフォーマルな組織として、
「職場仲間のグループ」
・
「習い事や
趣味のグループ」
・
「趣味や遊び仲間のグループ」の合計 3 つの組織を使
用する。インフォーマル組織については、1 番目に積極的に参加してい
る組織を分析する。
どちらの組織においても、政治的会話があり、かつ水平的構造をもつ
組織が、政治関心にプラスの効果を与えているのかについて確認する。
そのために、
「政治的会話」
・
「組織構造(水平的か垂直的か)
」
・
「政治的
会話×組織構造」を独立変数に、
「政治関心」を従属変数に設定して、
重回帰分析をおこなう。
予想される結果は、
「政治的会話×組織構造」のみが「政治関心」に
対してプラスの効果をもたらし、その一方で、
「政治的会話」
・
「組織構
造」については、効果が確認できないというものである。なぜなら、本
稿では、
「政治的会話」や「組織構造」は、それぞれ単独で効果をもた
らすものではなくて、それぞれが掛け合わされてはじめて効果をもたら
すものと想定しているからである。
表 1 予想されうる結果
従属変数:政治関心
政治的会話
組織構造
政治的会話×組織構造
×
×
○
○=有意なプラスの効果がみられる
×=有意な効果がみられない
3-3. 作業定義
本節では、上記の分析における変数の作業定義をそれぞれ提示する。
なお、より詳細な作業定義については、補遺に掲載しているので参照し
77
て頂きたい。
独立変数
●政治的会話の作業定義
政治的会話については、
「その団体ではメンバーの間で政治に関する
会話がよく交わされますか」という質問項目を使用する。政治的な会話
があるを「1」
、政治的な会話がないを「0」に再コードした。
●組織構造の作業定義
組織構造については、
「その団体ではメンバー間で上下関係がはっき
りしているか」という質問項目を使用する。上下関係がないと答えた場
合は水平的構造をもつ組織とみなして「1」
、逆に上下関係があると答え
た場合は垂直的構造をもつ組織とみなして「0」と再コードした。
●交互作用変数(政治的会話×組織構造)の作業定義
政治的会話×組織構造の交互作用変数については、政治的会話があり、
かつ水平的構造をもつ組織を「1」
、それ以外のパターンを「0」に再コ
ードした。
従属変数
●政治関心の作業定義
政治関心については、
「あなたは政治に関心がありますか、それとも
ありませんか」という質問項目を使用する。政治に関心があるを「1」
、
政治に関心がないを「0」に再コードした。
4. 分析結果
4-1. フォーマルな組織の分析結果
1 番目
積極参加
2 番目
積極参加
表 2 フォーマルな組織の分析結果
B値
政治的会話
.146
組織構造
-.148
政治的会話×組織構造
-.027
政治的会話
.158
組織構造
政治的会話×組織構造
危険率
.207
.068
.868
.184
.159
.072
.028
.854
調整済み R 二乗値 .052
まず、1 番目に積極的に参加しているフォーマルな組織の分析結果か
ら確認していこう。政治的会話を見ると、危険率が高く、有意な結果と
78
は言えない。フォーマルな組織において、政治的会話が政治関心を高め
るというメカニズムは確認できない。
次に、組織構造を見ると、-.148 とマイナスの効果を与えていること
がわかる。すなわち、組織構造が垂直的であれば、政治関心を高めると
いうことである。危険率も 10%水準ではあるが有意となっている。この
結果は、組織構造が単独では政治関心に影響を与えないとする本稿の想
定メカニズムとは、異なるものであった。また、水平的構造ではなく、
垂直的構造が政治関心を高めるというのも、想定外の結果であった。だ
が、後述するように、この組織構造の効果は偶然である可能性が非常に
高い。
そして、本稿の主たる関心である政治的会話×組織構造の効果を確認
しよう。すると、危険率が非常に高くなっていることがわかる。この危
険率の高さを見るかぎり、有意な結果とは到底言えない。
次に、2 番目に積極的に参加しているフォーマルな組織の分析結果に
うつろう。政治的会話を見ると、先ほどと同様に危険率が高くなってお
り、有意な結果ではない。
組織構造に目をうつすと、.159 とプラスの効果を与えていることがわ
かる。つまり、組織構造が水平的であれば、政治関心を高めるはたらき
があるということである。加えて、危険率も 10%水準ではあるが有意と
なっている。この結果は、組織構造の効果は、単独では影響を与えない
とする本稿の予想とは食い違うものである。しかし、ここで注意してお
きたいのは、先ほどの 1 番目の組織と比較すると、組織構造が全く正反
対の効果を与えていることである。分析によってプラスの場合もあれば
マイナスの場合もあることを考慮すると、どのようなメカニズムが働い
ているのかを想定することは難しい。したがって、組織構造の効果は、
偶然の結果にすぎないととらえるべきだろう。
最後に、政治的会話×組織構造の効果は、先ほどと同じく、危険率が
高く、有意な結果とはならなかった。
4-2. インフォーマルな組織の分析結果
1 番目
積極参加
表 3 インフォーマルな組織の分析結果
B値
政治的会話
組織構造
政治的会話×組織構造
危険率
.087
.453
-.126
.028
.226
.093
調整済み R 二乗値 .041
次に、インフォーマルな組織の分析結果にうつろう。まず政治的会話
を見ると、危険率が.453 と高く、有意な結果とはなっていない。インフ
ォーマルな組織において、政治的会話が政治関心につながるというメカ
79
ニズムは確認できない。
組織構造の効果に目をうつすと、-.126 とマイナスの効果を与えてい
ることがわかる。つまり、組織構造が垂直的であれば、政治関心を高め
るはたらきがあるということである。危険率も 5%水準で有意な結果と
なっている。この結果は、本稿の想定とは異なるものであるが、フォー
マルな組織において組織構造が重要な意味を持っていなかったことを
考慮すると、偶然の結果ととらえるべきだろう。
最後に、本稿の主たる関心である政治的会話×組織構造の効果を見る
と、.226 とプラスの効果を与えていることが確認できる。すなわち、政
治的会話があり、かつ水平的な構造をもつ組織は、政治関心を高めると
いう本稿の想定どおりの結果であった。危険率も 10%水準ではあるもの
の、有意なものとなっている。だが、この結果を数値のとおりに受け取
ることは難しい。なぜなら、上記で見てきたように、組織構造の影響は、
分析によって、プラスの場合もあれば、マイナスの場合もあり、明確な
メカニズムを説明することは難しいからである。政治的会話×組織構造
がプラスの効果を与えているのも、あくまでも偶然の結果にすぎないと
考えるべきだろう。
4-3. 分析結果のまとめ
以上の分析結果をまとめると、次の通りとなる。
第一に、パットナム (1993) の指摘するような組織構造の違いは、政
治関心を従属変数にすると、その効果が不確定なものになるということ
である。フォーマルな組織とインフォーマルな組織のどちらも、分析に
よって、政治関心に与える効果がプラスを示す場合もあれば、逆にマイ
ナスを示す場合もあった。たとえ分析の結果では有意なものとなってい
たとしても、組織構造の効果は、ほとんどないとして考えた方が良いだ
ろう。
第二に、政治的会話があり、かつ水平的な構造をもつ組織が、政治関
心を高めるという本稿の仮説は実証されなかった。インフォーマルな組
織において、10%水準ではあるが、政治的会話×組織構造が有意な結果
となっていたのは、偶然ととらえた方が良いだろう。なぜなら、上述の
ように、組織構造が政治関心にあたえる影響は、分析によって様々であ
り、明確なメカニズムが想定できないからである。
5. おわりに
本稿では、
「どのような組織が政治関心を高めるのか」というリサー
チクエスチョンに対して、
「政治的会話があり、かつ水平的な構造をも
つ組織は政治関心を高める」という仮説を設定して分析をおこなった。
分析の結果、この仮説は支持されなかった。
その主な理由として、パットナム (1993) が指摘するほど、 組織構造
80
の違いが、重要な意味をもっていないことが考えられる。パットナム
(1993) は、
社会関係資本を生み出すのは、
垂直的ネットワークではなく、
水平的ネットワークであると結論づけた。だが、従属変数こそ政治関心
にしているが、本稿における分析結果を見ると、組織構造が明確なメカ
ニズムに基づいて効果を与えているとは言い難い。池田 (2002) も、垂
直的な構造をもつ組織において、政治参加行動にプラスの効果を与えて
いることを確認している。パットナム的な組織構造の見方を一概に正し
いと考えるのは、あまり良いことではないだろう。
組織構造が重要な意味をもたないのには、いくつかの理由が考えられ
る。まず第一に、ほとんどの組織が、垂直的構造と水平的構造をあわせ
もっている点を挙げることができる。自治会・町内会においても会長が
存在しており、会長と親しく付き合う人もいれば、そうでない人もいる。
前者は水平的な構造と答えるが、後者は垂直的な構造と捉えるかもしれ
ない。パットナム (1993) も、ほとんどのネットワークは、
「二つの要素
の混在物」であると指摘している (パットナム 1993, 翻訳 215) 。ある
組織を垂直的構造あるいは水平的構造であると一概に判断するのは、か
なり難しいと言える。
第二に、パットナム (1993) の組織構造の理論を、現在の日本にあて
はめるには、
少々無理があった点が挙げられる。
パットナム (1993) は、
垂直的ネットワークにおいては、
「従属者が搾取されることを恐れて自
己防衛の為に情報の出し惜しみ」をするとしている(パットナム 1993,
翻訳 217)。しかし、現在の組織を考えた場合、従属者が搾取されると
いう関係は想像しにくいものだろう。上下関係があるといっても、それ
が搾取につながるほど厳格なものではないだろう。パットナム的な組織
構造の理論をそのまま現代日本の組織にあてはめるには、いささか時代
錯誤であったのかもしれない。
注
(1) もちろん、日常のなにげない会話を通じて、政治のリアリティが形
成されるとする先行研究もある。池田 (2001) は、日常的なコミュ
ニケーションの産物として、政治のリアリティが形成されていく点
を指摘している。また、同書のなかで、池田は、多かれ少なかれ日
常のコミュニケーションのなかに政治に関連する言及が意図せず含
まれているとも主張している。
(2) ここで利用した「日本人の民主主義観と社会資本に関する世論調査
-2000」は、選挙とデモクラシー協会 (JEDS) が実施した世論調査
である。いずれも、同志社大学・法学部の西澤由隆教授のご指導と
便宜により使用することができた。データを公開・寄託され、利用
できるようにしてくださった先生方に感謝いたします。
81
<補遺>
*フォーマルな組織
・まず「次に、あなたが加入されているいろいろな組織や団体について
うかがいます。ここにあげている組織や団体の中で加入しているも
のをすべてあげてください。あなたはその○○の活動に積極的に参
加していますか」と尋ねて、
「1.自治会・町内会、2.PTA、3.同業者
の団体、4.農協、5.労働組合、6.生協・消費者団体、7.ボランティア
団体、8. 住民運動団体、9.市民運動団体、10.宗教団体、11.学校の
同窓会、12.政治家の後援会」の中から選択してもらう。そして、
「た
いへん」あるいは「かなり」積極的に参加していると答えたひとに、
1 番目と 2 番目に積極的に参加している団体を選択してもらい、そ
れぞれの組織について、以下の内容を質問していく。
●政治的会話(フォーマル)
・質問文 「その団体ではメンバーの間で政治に関する会話がよく交わ
されますか」
・回答コード「1.はい、2.いいえ、3.どちらとも言えない、4.わからな
い 5.無回答」から「3.どちらとも言えない、4.わからない、5.無回
答」を欠損値として取り除き、
「1.はい=1、2.いいえ=0」に再コー
ドした。
●組織構造(フォーマル)
・質問文 「その団体ではメンバー間の上下関係がはっきりしています
か」
・回答コード「1.はい、2.いいえ、3.どちらとも言えない、4.わからな
い 5.無回答」から「3.どちらとも言えない、4.わからない、5.無回
答」を欠損値として取り除き、
「1.はい=0、2.いいえ=1」に再コー
ドした。
●交互作用変数(フォーマル)
・上記の政治的会話と組織構造の回答のうち、
「政治的会話があり、上
下関係がない」の組み合わせを「1」とし、それ以外の組み合わせに
ついては「0」とした。
●政治関心
・質問文「あなたは政治に関心がありますか、それともありませんか」
・回答コード「1.関心がある、2.関心がない、3.わからない、4.無回答」
から「3.わからない、4.無回答」を欠損値として取り除き、
「1.関心
がある=1、2.関心がない=0」に再コードした。
*インフォーマルな組織
・まず「次に、あなたの身近なグループについてうかがいます。あなた
は、仕事を離れたつき合いのある職場仲間のグループに入っていま
すか。では、習い事や学習のグループに入っていますか。趣味や遊
び仲間のグループはいかがですか」と尋ねる。そして、加入してい
るグループのみに「グループでのつき合いや活動に積極的に参加し
82
ていますか」と尋ねる。
「たいへん」あるいは「かなり」積極的に参
加していると答えたひとに、最も積極的に参加しているグループを
答えてもらい、そのグループについて以下の内容を質問していく。
●政治的会話(インフォーマル)
・質問文「そのグループではメンバーの間で政治に関する会話がよく交
わされますか」
・回答コード「1.はい、2.いいえ、3.どちらとも言えない、4.わからな
い 5.無回答」から「3.どちらとも言えない、4.わからない、5.無回
答」を欠損値として取り除き、
「1.はい=1、2.いいえ=0」に再コー
ドした。
●組織構造(インフォーマル)
・質問文「そのグループではメンバー間の上下関係がはっきりしていま
すか」
・回答コード「1.はい、2.いいえ、3.どちらとも言えない、4.わからな
い 5.無回答」から「3.どちらとも言えない、4.わからない、5.無回
答」を欠損値として取り除き、
「1.はい=0、2.いいえ=1」に再コー
ドした。
●交互作用変数(インフォーマル)
・上記の政治的会話と組織構造の回答のうち、
「政治的会話があり、上
下関係がない」の組み合わせを「1」とし、それ以外の組み合わせに
ついては「0」とした。
●政治関心
・上記のフォーマル組織の質問項目と作業定義と同様。
*シンタックス
●フォーマルな組織に関する分析
MISSING VALUES a20a5 (3,7,8).
MISSING VALUES a20a9 (3,7,8).
MISSING VALUES a20b5 (3,7,8).
MISSING VALUES a20b9 (3,7,8).
MISSING VALUES a10 (7,8).
recode a20a5 (1=0)(2=1) into jougekankei1.
recode a20a9 (1=1)(2=0) into political1.
recode a20b5 (1=0)(2=1) into jougekankei2.
recode a20b9 (1=1)(2=0) into political2.
recode a10 (1=1)(2=0) into interest.
compute formal1 = jougekankei1 * political1.
compute formal2 = jougekankei2 * political2.
REGRESSION VARIABLES=interest
political1
jougekankei1
83
formal1
political2
jougekankei2
formal2
/DEPENDENT interest
/METHOD=ENTER.
●インフォーマルな組織に関する分析
MISSING VALUES a23c5 (3,7,8).
MISSING VALUES a23c9 (3,7,8).
MISSING VALUES a10 (7,8).
recode a23c5 (1=0)(2=1) into jougekankei3.
recode a23c9 (1=1)(2=0) into political3.
recode a10 (1=1)(2=0) into interest.
compute informal = jougekankei3 * political3.
REGRESSION VARIABLES=interest
political3
jougekankei3
informal
/DEPENDENT interest
/METHOD=ENTER.
<参考文献>
・池田謙一 1997. 『転変する政治のリアリティ』 木鐸社.
・池田謙一 2001. 「メディエイティド・コミュニケーションと政治」
池田謙一編 『政治行動の社会心理学』北大路書房 所収.
・池田謙一 2002. 「2000 年衆議院選挙における社会関係資本とコミュ
ニケーション」 『選挙研究』17 号, p.5-18.
・蒲島郁夫 1988. 『政治参加』 東京大学出版会.
・平野浩 2002. 「社会関係資本と政治参加-団体・グループ加入の効
果を中心に」 『選挙研究』 17 号, p.19-30.
・ロバート・D・パットナム 1993. 『哲学する民主主義-伝統と改革
の市民的構造』 河田潤一訳 NTT 出版 2001.
・Schudoson, Michael 1997. Why conversation is not the soul of
democracy. Critical Studies in Mass Communication,
14 ,p.297-309.
・Verba, Sidney and Norman H. Nie, 1972. Participation in America :
Social Equality and Political Democracy, New York : Harper &
Row, pp.174-208.
84
8
低信頼層の投票行動
-なぜ投票するのか-
浦田 祐香
1. はじめに
有権者の政治的信頼が低い状態、政治不信には、小林(1997)によると
大きく分けて三つの段階が存在する。まず、第一段階は「政権担当者に
対する不信」である。政治腐敗は現政府に起因する問題であり、現政府
は信頼できないと考えている状態である。第二段階は「既成の政党や政
治家全体への不信」であり、与野党ともに信頼できないと考えている状
態である。そして第三段階は「間接代表制そのものへの不信」である。
この不信は政党や政治家を通して自分たちの意向を政治に反映するこ
とができないと考えている状態である(小林 1997, 4)
。
この二段階目の「既成の政党や政治家全体への不信」
、つまり政治全
体への抽象的な信頼が低い状態に本稿では注目する。なぜならこの段階
の不信は「
『政府は国民のことを考えていないのではないか』とか『政
治家は自分達のためにしか働いていない』といった気持ち」であり、政
治の知識をあまり持たなくても有権者が容易に想起でき、かつ政治全体
を総合的に評価している意識であると思ったからである(小林 2000,
117)
。また、詳しくは次項で記述するが、本分析では「国の政治への信
頼」について尋ねるデータを用いる。そのため、本分析で指す不信は「国
の政治への信頼」が低い状態を表す。この不信から有権者が想像するの
は、
「既成の政党や政治家全体への不信」であると本稿で考えているこ
ともこの段階の政治不信に注目する理由である。
本稿ではこの「既成の政党や政治家全体への不信」を政治的信頼が低
い状態と定義し、政治的信頼が低い有権者の投票行動を分析する。
「最
も多くの市民が参加する政治活動」である投票参加に政治的信頼が与え
る影響について分析することで、より多くの有権者に政治的信頼が与え
る影響を分析できると考える(蒲島 1998, 7)
。
政治的信頼が投票行動に与える影響についての分析は今までいくつ
かなされてきた。蒲島(1998)は、実証分析の結果、政治的信頼は一般
的に投票にプラスの効果を持つと述べている。三船(2005)の実証分析
でも政党支持・政治義務感・政治関心に比べると影響はかなり小さいが、
政治的信頼の低下が投票率の低下に影響を与えていることを明らかに
している。また、善教(2013)は、影響は大きくはないが信頼の低下が
85
棄権率の増加につながっているという分析結果を示している。これらの
先行研究より政治的信頼は投票に効果を持ち、また政治的信頼が低いと
棄権率が上がるということがわかった。1
しかし棄権率は上がるとしても、反抗心をあらわすためにか、政治的
信頼が低くても投票へ行く有権者は存在する。そこでなぜ政治的信頼が
低いのに棄権という選択肢を選ばず、わざわざ投票へ行くのかというこ
とに疑問を持った。政治的信頼が低いのならば、
「選挙にいったところ
で政府・政治家は市民のために働いてくれない」と判断し、コストがか
かる投票には行かないほうが合理的であるのではないか。この疑問を解
決するために、低信頼層で投票へ参加する人にはどのような投票行動の
メカニズムがあるのかを本稿では分析したいと思う。そのためリサーチ
クエスチョンを「低信頼層はなぜ投票するのか」とし、低信頼層の有権
者が棄権を選択せず投票へ行く要因を分析していく。
上記のリサーチクエスチョンに対する仮説として、有権者の積極性と
いう部分に注目した仮説を本稿では提示している。棄権せず投票へ行く
低信頼層の有権者は、
「政治は信頼できないけれど、積極的に自分の意
見を訴えなければならない」という考えを持っているのではないかと考
えるからだ。このような考えを持つ低信頼層の有権者の積極性を表す仮
説を考えた。
「政治への積極性仮説」と「政治以外での積極性仮説」の 2
つの仮説である。結論を先に述べると、前者は有意な結果がでて仮説が
証明され、後者も仮説に用いたデータの度数が小さいため断言はし難い
が、低信頼層の投票行動に影響を与えているという結果が出た。そのた
め、本稿の分析より政治に対する積極性、あるいは政治以外での積極性
があることが、低信頼層の有権者が投票に参加する要因となっているこ
とを明らかにすることができた。
本稿では以上の分析について論じるため以下の構成ですすめる。第 2
節で信頼の程度が投票行動に与える影響を分析し、信頼の程度ごとの投
票率を示す。第 3 節でリサーチクエスチョンに対する二つの仮説とその
メカニズムを述べ、第 4 節では分析のモデル図を提示する。そして第 5
節ではリサーチクエスチョンと二つの仮説の分析結果とその解釈を示
す。第 6 節でまとめを述べる。
2. 投票と信頼
本節では信頼の程度が投票行動に与える影響を分析し示す。独立変数
に信頼の程度、従属変数に投票参加の有無を投入しクロス表に整理した。
信頼の程度には、 質問「あなたは国の政治にどれくらい信頼できると
お考えでしょうか。
」 回答「かなり信頼できる」
・
「やや信頼できる」
・
「あまり信頼できない」
・
「ほとんど信頼できない」という質問項目を用
いた。また本稿で特に注目している低信頼層は「ほとんど信頼できない」
と答えている有権者を指す。本稿の分析ではすべて JESⅢの 2003 年衆
86
表 1 がクロス表分析の結果であ
議院議員選挙事後データを用いる。2
る。
表1 信頼の程度 と 投票 のクロス表分析
投票(%)
棄権
合計(%)
N
参加
ほとんど信頼できない
25.1
74.9
100
207
あまり信頼できない
10.9
89.1
100
1021
やや信頼できる
6.3
93.8
100
864
かなり信頼できる
3.8
96.2
100
78
合計(%)
10.1
89.9
100
2170
カイ二乗値 69.334 危険率 0.000
図 1 信頼の程度と投票のクロス表分析
100%
95%
90%
投
票 85%
率
80%
信頼のみ
75%
70%
0
1
2
3
信頼の程度
(横軸 0=ほとんど信頼できない・1=あまり信頼で
きない・2=やや信頼できる・3=かなり信頼できる)
クロス表分析の結果、危険率が 0.00%で有意な結果となり、信頼の程
度と投票参加は相関関係にあることがわかった。そして、低信頼層が他
の信頼の程度よりも極めて投票率が低いという結果がでた。ほとんど信
頼できないと答えた有権者の投票率が 74.9%と最も低く、国の政治を信
頼するほど投票率が上がっている。そのため上述した先行研究で示され
ていた、信頼が低いほど棄権率が上がるという仮説がここでも証明され
た。
他の信頼の程度に比べると低いが、低信頼層でも 75%近くの人々は投
票に参加している。第 1 節でも述べたように、この低信頼層の有権者は
87
なぜ棄権せず投票へ参加しているのかを分析していく。
3. 仮説
3-1. 政治への積極性仮説
本稿では「なぜ低信頼でも投票へ行くのか」というリサーチクエスチ
ョンに対し、政治への積極性仮説を設定する。本稿で用いる政治への積
極性がある有権者とは、政治に積極的に働きかけ意見を述べていかなけ
ればならないという意識を持つ、政治に積極的に参加する有権者と示す。
本仮説は、政治に対して積極的に意見を主張し関与していかなければな
らないと思っている有権者は、政治的信頼が低くてもコストがかからな
い棄権を選択せず投票へ行くのではないかというものである。以下、仮
説のメカニズムを述べる。
低信頼層の有権者は他の信頼の程度の有権者と比べ、投票に参加する
か棄権するかの二つの選択肢の選び方に特徴がある。2 節までにすでに
述べたが、低信頼層の有権者は棄権という選択肢を選びやすいのだ。そ
して、低信頼層の有権者は投票に参加するか棄権するかという二つの選
択肢を選ぶ際、つぎのような意識を持つと本稿では考えている。
「政治
は信頼できなくても、投票へは行かなければならない」と考える有権者
は投票に参加し、
「政治は信頼できず投票しても自分が利益を得られる
ことはないので、わざわざコストをかけてまで投票へ行く必要はない」
と考える有権者は棄権する。
本仮説では、低信頼層の有権者が投票に参加するか棄権するかを分け
る一つの要因として、有権者の政治に対する積極性が挙げている。つま
り、
「政治は信頼できなくても、投票へは行かなければならない」と考
える有権者は、政治への積極性が高いから投票に参加していると考えて
いるのだ。
政治への積極性が高い有権者は、政治に積極的に働きかけ自らの意見
を述べていかねばならないと考えている。そのため、政治的信頼が低く
ても政治に積極的に参加しなければならないという意識を持つため、コ
ストをかけてまで投票に参加すると考えている。これが本仮説のメカニ
ズムである
なお、低信頼層の有権者は投票によって政治への反抗的な意見を表明
するのではないかと考えられる。これは「不信感が強い場合にも政府に
対する対抗関係から抗議活動として<個別的接触>や<市民・住民運動
>に携わる」という理論が投票行動においても適用できるのではないか
と考えるからである(三船 2008, 199)
。
また、本稿では低信頼層の政治への積極性を<意識面>と<行動面>
の二段階で分析する。二段階で分析することで、低信頼層の政治への積
極性を総合的に捉えることができると考える。詳しい分析方法は 4 節で
述べる。
88
3-2. 政治参加以外での積極性仮説
二つ目の仮説として政治参加以外での積極性仮説を設定する。政治参
加以外での積極性仮説とは、政治的信頼が低くても有権者が積極的な性
格であれば投票に参加するというものである。積極的な性格とは、物事
を自ら進んで行うような性格のことであり、政治参加以外での積極性が
ある人とは、俗に言う性格が積極的な人を指す。
この政治参加以外での積極性が低信頼の有権者が投票へ行く一つの
要因になっていると考えている。本仮説は、積極的な性格であれば、自
ら持つ政治への意見を積極的に述べていかなければならない・政治に関
わっていかなければならないと考え、政治的信頼が低くてもコストを顧
みず投票に参加するというものである。
この分析を行うことで一見、政治とは関係ないようにみえる積極的な
性格という有権者の素質が投票行動という政治参加に与える影響を明
らかにする。
有権者の積極的な性格は政治参加以外の組織や団体の活動に対して
どれだけ積極的に活動しているかで定義する。
4. モデル図
<低信頼層>
政治への積極性<意識面>
<行動面>
投票参加
政治参加以外での積極性
89
5. 分析方法・予想される結果
<政治への積極性仮説>
政治への積極性仮説では二段階の分析を行う。政治への積極性仮説の
分析は 2 つとも三重クロス表分析を行った。
A-1. 政治への積極性<意識面>の分析
独立変数に信頼の程度と政治への積極性<意識面>、従属変数に投票
参加の有無を投入しクロス表分析を行い、政治への積極性<意識面>が
ある有権者の投票率を信頼の程度ごとに導き出す。そして 2 節の表 1 の
信頼の程度ごとの投票率と比較する。政治への積極性<意識面>は各信
頼の程度の投票率にプラスの影響を与える。そのため、すべての信頼の
程度において、表 1 の投票率より投票率は上がる。しかし、政治への積
極性<意識面>は低信頼層の有権者に最も大きな影響を与えるため、低
信頼層の(ほとんど信頼できないと答える)有権者の投票率が、他の信
頼の程度の有権者の投票率と比べ最も伸びると考える。よって、図 1 の
ように表 1 で最も低かった低信頼層の投票率が大きくなり、曲線の傾き
がゆるやかになると予想する。
政治への積極性<意識面>で使用する質問項目は以下の通りである。
質問「次に、あなたにとって、政治とはどのような存在かについておた
ずねします。あなたのお考えに近いものは次のうちどれでしょうか。
」
(1)
政治とは自分から積極的に働きかけるもの 回答「そう思う」
・
「ややそ
う思う」
・
「あまりそう思わない」
・
「そう思わない」である。
「そう思う」
と「ややそう思う」を政治への積極性<意識面>あり、
「あまりそう思
わない」と「そう思わない」を政治への積極性<意識面>がないとした。
なお、わからない・答えないは欠損値として処理した。
A-2. 政治へ積極性<行動面>の分析
独立変数に信頼の程度、政治への積極性<行動面>従属変数に投票参
加の有無を投入しクロス表分析を行う。以降の分析の手順・予想される
結果は政治への積極性<意識面>と同様である。
政治への積極性<行動面>で使用する質問項目は以下の通りである。
質問「この中のうち、あなたが 5 年間に経験したことをすべてお答えく
ださい」 回答「選挙で投票した」
・
「自治会や町内会活動で活動した」・
「必要があった地元の有力者と接触した(会う、手紙を書くなど)
」
・
「必
要があって政治家や官僚と接触した」
・
「議会や役所に請願や陳情に行っ
た」
・
「選挙や政治に関する集会に出席した」
・
「選挙運動を手伝った(候
補者の応援など)
」
・
「市民運動や住民運動に参加した」
・
「請願書に署名
した」
・
「献金やカンパをした」
・
「デモに参加した」
・
「インターネットを
通して意見を表明した」である。12 種類の政治参加のうち一つでも経験
したことがあると答えた有権者を政治への積極性<行動面>あり、とし
た。なお、わからない・答えないは欠損値として処理した。
90
<政治参加以外での積極性仮説>
B. 政治参加以外の積極性の分析
次に政治参加以外での積極性仮説の分析方法を述べる。この分析も三
重クロス表分析を用いた。独立変数に信頼の程度と政治参加以外での積
極性、従属変数に投票参加の有無を投入してクロス表分析を行った。以
降の分析の手順・予想される結果は政治への積極性<意識面><行動面
>と同様である。
政治参加以外での積極性の分析で使用する質問項目は以下の通りで
ある。 質問「あなたは、この中にあげている組織や団体、グループの
メンバーになっていますか(参加していますか)
。メンバーになってい
る場合、その団体の活動に積極的に参加していますか」 回答「NPO・
NGO」
・
「ボランティア団体」
・
「学校の同窓会」
・
「仕事を離れたつき合
いのある職場仲間のグループ」
・
「習い事や学習のグループ」
・
「趣味や遊
び仲間のグループ」である。なお、この 6 種類の組織や団体は、回答項
目のなかでも政治参加以外に当てはまるものを抽出した。6 種類の組織
や団体のうち一つでも積極的に参加していると答えた人は政治参加以
外での積極性があるとした。なお、わからない・答えないは欠損値とし
て処理した。
図 2 分析 A-1・A-2・B の予想される結果
100%
95%
90%
投
票
率
85%
信頼のみ
80%
予想(信頼×積
極性あり)
75%
70%
0
1
2
3
信頼の程度
(横軸 0=ほとんど信頼できない・1=あまり信頼できな
い・2=やや信頼できる・3=かなり信頼できる)
91
6.分析結果
まず、最初に全体の分析結果を述べる。政治への積極性仮説の分析は
<意識面>・<行動面>ともに有意であり、独立変数が低信頼層の投票
率に与える影響の大きさに差はあるが低信頼層の投票率が上がり、傾き
もゆるやかになった。そのため、仮説は支持されたといえる。政治参加
以外での積極性仮説の分析でも、仮説が低信頼層の投票行動にプラスの
影響を与え、仮説が支持された。しかし、すべての分析において度数が
5 以下のセルがあるため、分析の有効性が疑われる。特に政治以外での
積極性仮説の分析は、分析全体の度数が小さい。この点は懸念せねばな
らない点である。以下、分析結果について詳しく解説していく。
92
<政治への積極性仮説>
1.<意識面>
表2 信頼の程度 と 投票 と <意識面> のクロス表分析
(積極性ありの対象者のみ)
投票(%)
政治への積極性<意識面>あり
棄権
合計(%)
N
参加
ほとんど信頼できない
12.8
87.2
100
117
あまり信頼できない
8.0
92.0
100
525
やや信頼できる
4.7
95.3
100
507
かなり信頼できる
1.8
98.2
100
55
合計(%)
6.8
93.2
100
1204
カイ二乗値 13.434 危険率 0.004
図 3 信頼の程度と投票と政治的積極性<意識面>の
クロス表分析のグラフ
100%
95%
90%
投
票 85%
率
80%
(意識面)
信頼のみ
75%
70%
0
1
2
3
信頼の程度
(横軸 0=ほとんど信頼できない・1=あまり信頼できな
い・2=やや信頼できる・3=かなり信頼できる)
表 2 を見てわかるように、政治への積極性<意識面>の分析の危険率
は 0.004 で有意な結果となった。そして、政治への積極性<意識面>が
ある有権者の投票率を信頼の程度ごとにみると、
「ほとんど信頼できな
い」
(低信頼層)は 87.2%、
「あまり信頼できない」は 92.0%、
「やや信
頼できる」は 95.3%、
「かなり信頼できる」は 98.2%となっている。こ
の数値をグラフに表したのがグラフ 2 である。図 3 をみると、表 1 の信
93
頼の程度ごとの投票率と比べ、
「ほとんど信頼できない」と答える(低
信頼層の)有権者の投票率の伸び率は 12.3%、
「あまり信頼できない」
と答える有権者のそれは 2.9%、
「やや信頼できる」と答える有権者のそ
れは 1.5%、
「かなり信頼できる」と答える有権者のそれは 2%となって
いる。よって、低信頼層の有権者の投票率の伸び率が最も大きく、図 1
の信頼の程度ごとの投票率の分析結果より傾きがゆるやかになってい
る。この分析では、予想した結果が出て仮説が支持された。つまり、政
治的信頼が低くても政治への積極性<意識面>がある有権者は投票へ
参加する傾向にあると考えられる。
94
3.<行動面>
表3 信頼の程度 と 投票 と <行動面> のクロス表分析
(積極性ありの対象者のみ)
投票(%)
政治への積極性<行動面>あり
棄権
合計(%)
N
参加
ほとんど信頼できない
25.1
74.9
100
190
あまり信頼できない
10.9
89.1
100
988
やや信頼できる
6.3
93.8
100
854
かなり信頼できる
3.8
96.2
100
78
10.1
89.9
100
2110
カイ二乗値 40.739 危険率 0.000
合計(%)
図 4 信頼の程度と投票と政治への積極性<行動面>の
クロス表のグラフ
100%
95%
90%
投
票 85%
率
80%
(行動面)
信頼のみ
75%
70%
0
1
2
3
信頼の程度
(横軸 0=ほとんど信頼できない・1=あまり信頼できな
い・2=やや信頼できる・3=かなり信頼できる)
政治への積極性<行動面>の分析の危険率は 0.000 で有意な結果とな
った。そして、政治への積極性<行動面>がある有権者の投票率を信頼
の程度ごとにみると、
「ほとんど信頼できない」
(低信頼層)は 81.6%、
「あまり信頼できない」は 92.10%、
「やや信頼できる」は 94.8%、
「か
なり信頼できる」は 96.2%となっている。この数値をグラフに表したの
が図 4 である。図 4 をみると、
「かなり信頼できる」以外では図 1 の信
頼の程度のみの分析に比べ、投票率は上がっている。そのなかでも「ほ
95
とんど信頼できない」と答える(低信頼の)有権者の投票率の伸び率は
6.7%、
「あまり信頼できない」と答える有権者のそれは 3%、
「やや信頼
できる」と答える有権者のそれは 1%、
「かなり信頼できる」と答える有
権者のそれは 0%となっている。よって、低信頼層の有権者の投票率の
伸び率が最も大きい。しかし、<意識面>よりは他の信頼の程度の伸び
率との差は小さい。そのため傾きも<意識面>ほどはゆるやかになって
いない。このことから、<行動面>では低信頼層の投票行動へのプラス
の影響は比較的小さいといえる。しかし、予想した結果ではあったため
仮説が支持されたと考える。政治的信頼が低くても政治への積極性<行
動面>がある有権者は投票へ参加する傾向があると考える。
96
<政治参加以外での積極性>
表4 信頼の程度 と 投票 と 政治参加以外 のクロス表分析
(積極性ありの対象者のみ)
投票(%)
政治参加以外での積極性あり
棄権
合計(%)
N
参加
ほとんど信頼できない
12.5
87.5
100
40
あまり信頼できない
9.7
90.3
100
216
やや信頼できる
4.4
95.6
100
203
かなり信頼できる
.0
100.0
100
22
7.3
92.7
100
481
カイ二乗値 7.691 危険率 0.053
合計(%)
図 5 信頼の程度と投票と積極性(政治参加以外)の
クロス表分析のグラフ
100%
95%
90%
投
票
率
85%
(政治参加以
外)
80%
信頼のみ
75%
70%
0
1
2
3
信頼の程度
(横軸 0=ほとんど信頼できない・1=あまり信頼できな
い・2=やや信頼できる・3=かなり信頼できる)
政治参加以外での積極性の分析の危険率は 0.053 であり、他の分析に
比べると少し高かった。政治以外での積極性がある有権者の信頼の程度
ごとの投票率は、
「ほとんど信頼できない」
(低信頼層)は 87.5%、
「あ
まり信頼できない」は 90.3%、
「やや信頼できる」は 95.6%、
「かなり信
頼できる」は 100%となっている。この数値をグラフに表したのが図 5
である。図 5 をみると、すべての信頼の程度で信頼のみの分析に比べ投
97
票率は上がっている。
「ほとんど信頼できない」と答える(低信頼の)
有権者の投票率の伸び率は 12.6%、
「あまり信頼できない」と答える有
権者のそれは 1.2%、
「やや信頼できる」と答える有権者のそれは 1.8%、
「あまり信頼できない」と答える有権者のそれは 3.8%となっている。
よって、特に低信頼層の伸び率が大きく、傾きもゆるやかになっている。
そのため、予想通りの結果が出て仮説が支持されたように見える。しか
し、この分析では、政治参加以外での積極性の変数の度数が小さいこと
より、分析の有効性が疑われるため仮説が支持されたと断言しがたい。
7. 考察
以上の分析をまとめる。政治への積極性の二つの分析により、意識と
して政治への積極性を持っている場合と実際に積極的に政治参加して
いる場合両方で影響の大きさに差はあるが、低信頼層の投票率が上がる
という結果が出た。したがって 3 節の仮説のメカニズムとしても述べた
ように、低信頼層のなかでも「政治は信頼できないので、わざわざコス
トをかけてまで投票へ行く必要はない」と考える有権者は棄権し、
「政
治は信頼できなくても、自らの意見を政治に訴えることは必要である」
と考える有権者は投票へ行く傾向があると考えられる。この分析によっ
てどのような低信頼層の有権者が棄権でなく投票参加を選択するかが
部分的ではあるが示すことができた。なお、上述のように政治以外での
積極性仮説では分析の有効性が低いため、政治参加以外での積極性が低
信頼層の投票率に与える影響を示すことができたと断言しがたい。
本稿で分析しきれなかった課題もある。低信頼層がどのような意見を
政治へ訴えるために投票に参加しているかということである。3 節の政
治への積極性仮説の項でも少し述べたように、政治的信頼が低くても投
票に参加する有権者は、政治への反抗的な意見を表明するために投票に
参加するのではないかと考えているが、本稿で使用したデータでは分析
できない。しかし、低信頼層の有権者がどのような想いをもって投票に
参加しているのかということは、大変興味深いことである。そのためこ
の点を分析することは今後の課題である。
注
(1) 三船(2005)
・善教(2013)で述べられている政治的信頼が低い状
態は、各々の定義付けがなされているため、本稿で定義した政治的
信頼が低い状態とは異なる。
(2) JESⅢは、
「21 世紀初頭の投票行動の全国的・時系列的調査研究(JES
Ⅲ SSJDA 版), 2001-2005」 で、JESⅢ研究会が実施された世
論調査である。その個票データについて、東京大学社会科学研究所
付属日本社会研究情報センター・データアーカイブ(Social Science
Japan Data Archive) より教材としての利用許可(申請者:西澤由
98
隆教授)を得たものを使用した。いずれも、同志社大学・法学部の
西澤由隆先生のご指導とご便宜により利用ができた。それぞれのデ
ータを公開・寄託され、利用できるようにしてくださった先生方に
感謝致します。
<補遺>
○分析に使用した質問文とコード
●投票参加
Q1「さっそくですが、あなたは 11 月 9 日(日曜日)の衆議院選挙の投
票に行きましたか。この中から 1 つだけ答えてください。
」e1
(1)投票日に投票した( 2)不在者投票をした (3)棄権した (4)わからない
(5)答えない」
⇒ 1,2=1 3=0 4,5=欠損値 として再コード
●政治的信頼
Q15「あなたは日本の政治をどれくらい信頼できるとお考えでしょうか。
この中ではどれにあたりますか。
」e15x1
(1)かなり信頼できる(2)やや信頼できる(3)あまり信頼できない(4)ほと
んど信頼できない(5)わからない(6)答えない
⇒ 1=3 2=2 3=1 4=0 5,6=欠損値 として再コード
●政治への積極性(意識面)
Q13「次に、あなたにとって、政治とはどのような存在かについておた
ずねします。あなたのお考えに近いものは次のうちどれでしょうか。
」
<1>政治とは自分から積極的に働きかけるもの e13x1
(1)そう思う(2)ややそう思う(3)あまりそう思わない(4)そう思わない(5)
わからない(6)答えない
⇒ 1=3 2=2 3=1 4=0 5,6=欠損値 として再コード
3,2=1 1,0=0 のダミー変数に再コード
●政治への積極性(行動面)
Q11「この中のうち、あなたが5年間のうちに経験したことをすべてお
答えください」
<1>選挙で投票した e11_1・<2>自治会や町内会で活動し e11_2
・<3>必要があって地元の有力者と接触した (会う、手紙を書くなど)
e11_3・<4>必要があって政治家や官僚と接触した e11_4・ <5>議
会や役所に請願や陳情に行った e11_5 ・<6>選挙や政治に関する集
会に出席した e11_6 ・<7>選挙運動を手伝った(候補者の応援など)
e11_7 ・<8>市民運動や住民運動に参加した e11_8 ・<9>請願書に
署名した e11_9・ <10>献金やカンパをした e11_10 ・<11>デモに
参加した e11_11・<12>インターネットを通して意見を表明した
e11_12 ・<13>どれもしたことがない e11_13 ・<14>わからない
e11_14・ <15>答えない e11_15
⇒<1>~<12>のうち 1 つ以上 1 を選んでいるものを積極性ありと再コ
ードした。
99
●政治参加以外での積極性
Q12「あなたはこの中にあげている組織や団体、グループのメンバーに
なっていますか。
(参加していますか)メンバーになっている(参加
している)場合、その団体の活動に積極的に参加していますか」
<6>NPO・NGO e12x6・ <7>ボランティア団体 e12x7 ・<12>仕事を
離れたつき合いのある職場仲間のグループ e12x12・ <13>習い事
や学習のグループ e12x13・ <14>趣味や遊びのグループ e12x14
(<11>政治家の後援会は仮説の趣旨と外れるので省いた)(1)メンバ
ーとして積極的に参加している(2)メンバーになっている程度(3)メ
ンバーではない(4)わからない(5)答えない
⇒ <6>・<7>・<10>・<12>・<13>・<14>のなかで 1 つ以上 1 を
選んでいるものを積極性ありと再コードした。
【シンタックス】
*信頼 e15x1
RECODE e15x1(1=3)(2=2)(3=1)(4=0)(else=COPY) into 信頼.
MISSING VALUES 信頼(8,9).
FREQUENCIES 信頼.
*投票参加 e1
recode e1(1=1)(2=1)(3=0)(else=COPY) into 投票.
MISSING VALUES 投票(9).
FREQUENCIES 投票.
*信頼と投票のクロス表分析
CROSSTABS TABLES = 信頼 by 投票
/CELLS COUNT row
/STATISTICS = CHISQ.
*政治への積極性(意識面)e13x1
RECODE e13x1(1=3)(2=2)(3=1)(4=0)(else=COPY) into 意識面.
MISSING VALUES 意識面(8,9).
FREQUENCIES 意識面.
RECODE 意識面(3=1)(2=1)(1=0)(0=0) into 意識面 2.
FREQUENCIES 意識面 2.
*信頼と政治への積極性(意識面)と投票のクロス表分析
CROSSTABS TABLES 信頼 by 投票 by 意識面 2
/CELLS count row
/STATISTICS = CHISQ.
*政治への積極性(行動面)e11_1 ~ e11_12
compute 行動面 = 0.
COMPUTE 行動面 = (e11_1=1) + (e11_2=1) + (e11_3=1) + (e11_4=1)
+ (e11_5=1) + (e11_6=1) + (e11_7=1) + (e11_8=1) + (e11_9=1) +
(e11_10=1) + (e11_11=1) + (e11_12=1).
100
FREQUENCIES 行動面.
RECODE 行動面(1,2,3,4,5,6,7,8,9,10,11=1)(0=0) into 行動面 2.
FREQUENCIES 行動面 2.
*信頼と政治への積極性(行動面)と投票のクロス表分析
CROSSTABS TABLES 信頼 by 投票 by 行動面 2
/CELLS COUNT row
/STATISTICS = CHISQ.
*政治参加以外での積極性
e12x6,e12x7,e12x10,e12x12,e12x13,e12x14
RECODE e12x6(1=1)(2=0)(3=0)(else=-9) into NPO.
RECODE e12x7(1=1)(2=0)(3=0)(else=-9) into ボランティア.
RECODE e12x10(1=1)(2=0)(3=0)(else=-9) into 学校.
RECODE e12x12(1=1)(2=0)(3=0)(else=-9) into 職場仲間.
RECODE e12x13(1=1)(2=0)(3=0)(else=-9) into 学び.
RECODE e12x14(1=1)(2=0)(3=0)(else=-9) into 遊び.
compute 積極性 = 0.
compute 積極性 = (NPO=1) + (ボランティア=1) + (学校=1) + (職場仲
間=1) + (学び=1) + (遊び=1).
FREQUENCIES 積極性.
RECODE 積極性(0=0)(1,2,3,4,5=1) into 積極性 2.
FREQUENCIES 積極性 2.
*信頼と政治参加以外での積極性と投票のクロス表分析
CROSSTABS TABLES 信頼 by 投票 by 積極性 2
/CELLS count ROW
/STATISTICS = CHISQ.
<参考文献>
・蒲島郁夫 1988. 『政治参加』東京大学出版会.
・蒲島郁夫 1998. 『政権交代と有権者の態度変容』木鐸社.
・三船毅 2008. 『現代日本における政治参加意識の構造と変動』慶應
義塾大学出版会.
・善教将大 2013. 『日本における政治への信頼と不信』木鐸社.
・小林良彰 1997. 『日本人の投票行動と政治意識』木鐸社.
・小林良彰 2000. 『選挙・投票行動』東京大学出版会.
・小林良彰 2005. 『日本における有権者意識の動態』慶應義塾大学出
版会株式会社.
101
9
異なるレベルの選挙
における選挙関心の
形成要因
―「ライブリー・ポリティクス」を題材に―
林 香織
1. はじめに
明るい選挙推進協会が 2011 年に行った「第 17 回 統一地方選挙に関
する意識調査」によると、人が選挙に興味を持つ度合いは選挙の種類ご
とに異なるという結果が出ている。この調査では「ここに 6 つの選挙が
あげてありますが、あなたがとくに関心をお持ちになる選挙を 2 つあげ
てください」と質問している(明るい選挙推進協会 2012, 18)
。この回
答の集計結果(図 1)を見ると、衆議院選挙においては 50%以上の有権
者が関心を持つと答えているのに対して、その他の選挙では関心を持つ
人が約 10%から 30%と少ない。
図1:関心のある選挙 ※2つ選択
0
10
20
30
40
衆議院議員総選挙
参議院議員通常選挙
都道府県知事選挙
都道府県議会議員選挙
市区町村長選挙
市区町村議会議員選挙
どれも関心を持たない
50
60 (%)
N=1910
出典:明るい選挙推進協会(2012)を元に筆者作成
この結果から、選挙に対する関心が形成されるメカニズムは選挙の種
類によって異なるということが考えられる。もし選挙関心度を規定する
要因が、国政選挙でも地方選挙でも同じなのであれば、関心度の差は図
1 ほど開かないはずである。地方選挙の関心度には影響を与えるが、国
政選挙の関心度には影響を与えないもの、あるいはその逆のものが存在
102
するのではないだろうか。
選挙関心度に差がある理由に、注目を集める争点・話題があるかどう
かが選挙によって異なるということが考えられる。国政選挙、とりわけ
衆議院選挙の場合はメディアがその選挙の争点や政権交代の可能性な
どについて取り上げることで話題になるうえに、選挙結果によって政治
が大きく変わる可能性もある。一方、地方選挙の場合は国政選挙ほど話
題性のある選挙は少なく、メディアもさほど大きく取り上げないことが
多い。
しかし大きな話題性はなくとも、地方選挙では地方選挙ならではの、
自分の暮らしに密着した事柄が争点となりえる。その地域ならではの課
題の解決策や、地域住民の安心・安全な生活を守るための方法などは、
地域によって具体的な政策が掲げられる。例えば 2012 年 1 月に行われ
た「京都市長選挙マニフェスト志向型公開討論会」では、翌月に控えた
京都市長選挙の立候補者である門川大作氏と中村和雄氏が京都市政に
ついて討論をした。その際に配られた参考資料には両氏による京都市政
の現状分析と改善目標、そしてその目標を達成するための具体策が書か
れていた。例えば門川氏は
(a) 「京都市版地域包括ケアシステム」をつくることでお年寄りが住み
やすいまちづくりを行うこと
(b) 保護者や地域住民の声を反映した「地域ぐるみの教育」を展開する
こと
などを具体案として挙げていた。1 そして中村氏は
(a) 子育て環境の整備が遅れている京都市に児童相談所や福祉センター
を増やすこと
(b) 災害発生時にも安心して受けられるような医療体制を確立させるこ
と
などを挙げていた。2 このことから国政選挙よりも地方選挙での方が地
域に合わせた政策が主張されると考えられる。そしてそれらの政策に興
味を持っている有権者は、そうでない者よりも地方選挙に関心を寄せる
と考えた。
以上のことから、地方選挙の場合は「自分の暮らしに密着した事柄」
に関心を寄せているかどうかが選挙関心度に結びつくのに対して、国政
選挙ではその影響があまりないと考えた。そこで本稿では「どのような
人が地方選挙に関心を持つのか」というリサーチクエスチョンについて
分析する。そして「自分の暮らしに密着した政治」として篠原(1985)
の「ライブリー・ポリティクス」の概念を用いて「
『ライブリー・ポリ
ティクス』に関心を寄せている人は、そうでない人よりも地方選挙に関
心を持つ」という「ライブリー・ポリティクス仮説」を立てた。国政選
挙と地方選挙で同じモデルの分析を行うことで、両者の関心度に影響を
与えるものに差があることを確認する。異なるレベルの選挙における政
治的態度の違いを扱った研究は少ないため、この分野にわずかでも貢献
103
ができればと思い、ライブリー・ポリティクス仮説を検証する次第であ
る。
この分析を行った結果、ライブリー・ポリティクス仮説をおおむね実
証することができた。国政選挙でも地方選挙でも、ライブリー・ポリテ
ィクスに関心を持っている人ほど選挙関心が強くなるが、ライブリー・
ポリティクスがより大きい効果を発揮するのは国政選挙より地方選挙
であることが確認できた。
これらのことを論じるために、本稿は以下の構成をとる。まず第 2 節
では政治関心についての理論や先行研究を簡単にレビューし、それらと
は異なるメカニズムが選挙関心度の差を説明するためには必要である
ことを論じる。そしてライブリー・ポリティクスについて説明し、なぜ
それが選挙関心度に影響を与えうるのか論じることで仮説を導く。第 3
節では仮説を検証するためのリサーチデザインを提示し、分析に使用し
た変数の定義方法を説明する。またライブリー・ポリティクスは 3 種類
の定義方法を考えたため、それらについても説明する。第 4 節では分析
結果を示し、ライブリー・ポリティクスの効果が、国政選挙より地方選
挙での方が大きいという結論を分析結果からどのように導いたか説明
する。最終節では本稿全体のまとめをおこなう。
2. 仮説のメカニズム
選挙を含めた、一般的な「政治」に対する関心や政治的無関心を規定
する要因が何なのかについてはいくつかの先行研究・理論が存在する。
例えば Dawson・Prewitt・Dawson(1977)は、教育程度が高い人の
方が低い人よりも 1) 政治知識が多く、それを活用する能力を持ってい
る 2) 政治的な出来事が自分の生活に関わるということを自覚している
から政治に積極的だという理論を提示している(Dawson・Prewitt・
Dawson 1977, 242-243)
。このことから教育程度は政治関心に影響を与
えると考えられる。実際にその相関関係は Almond・Verba(1963)に
よって計量的に確認されている。また Dawson・Prewit・Dawson は、
社会集団・家族・仲間集団・職場・学校などを介して政治上の出来事や
人の政治的な考えを知ることができると、自分の政治的態度が形成・強
化されると論じている。このことから集団との関わりは政治関心に影響
を与えると考えられる。一方、政治的無関心が生まれる理由としては、
1) 20 世紀以降、政治過程や政治機構が巨大化・複雑化してきているこ
とで人々が「自分に政治を動かす力はない」
・
「自分には政治が理解でき
ない」
などと感じること 2) 大衆消費文化の台頭 により非政治的領域に
対する関心が増すことが挙げられる(秋元 1993a, 398;秋元 1993b,
849;丸山 1954, 747-748;柳井 1993, 844)
。
しかしこれらの政治関心・政治的無関心に関する理論では、なぜ選挙
の種類によって関心度に差があるのかを説明できない。なぜなら教育や
104
人との接触を通して政治関心をはぐくむというメカニズムは地方選
挙・国政選挙に対する関心どちらにもあてはめることができるからであ
る。また地方政府と中央政府の政治過程や構造はどちらもある程度複雑
なので、人々が政治に対する無力感や理解力のなさを感じる対象が地方
政治・国政どちらであってもおかしくはない。
以上のことから、地方選挙と国政選挙の関心度の違いを説明するため
には、上記に紹介したものとは異なるメカニズムを想定しなければなら
ないと考えた。
本稿では、地方選挙と国政選挙の関心差を説明する要因として「ライ
ブリー・ポリティクス」を挙げて検証する。ライブリー・ポリティクス
(以下、
「LP」と略す)とは「生と生活に連関したいきいきとした政治」
のことをさす(篠原 1985, 9)
。
「
『生』に連関した政治」とは「生命体と
しての人間のあり方そのものに関する問題」であり、医療問題・食糧問
題・障害者支援や自己実現・生きがいを人々が得るための政治などをさ
す(篠原 1985, 16)
。そして「
『生活』に連関した政治」とは生活価値の
向上を目指すための政治であり、文字通り人々の生活に密着した事柄を
扱う。具体的には LP を説明するキーワードとして福祉・人権・教育・
都市問題・自然・環境・健康・エコロジー・人種・ジェンダー・コミュ
ニティなどがある(五十嵐 2010;五十嵐・シュラーズ 2012;角 1999;
篠原 1985)
。
五十嵐(2010)
・角(1999)
・篠原(1985)によると、日本では LP
が 1980 年代から取り上げられるようになったが、それ以外の時期には
3 つのポリティクスが展開されてきた。
1 つ目は「ハイ・ポリティクス」といい、1945 年から 1960 年の戦後
の時期に行われていた政治である。保革対立・資本主義対社会主義とい
ったイデオロギーに関連した政治のことをさし、安保闘争の時期に最も
盛んだった。
1960 年に池田政権が誕生し、野党との対話や所得倍増計画がなされ
たことからハイ・ポリティクスの時代は終わり、2 つ目の「インタレス
ト・ポリティクス」
(以下、
「IP と略す」
)の時代が到来する。IP とは物
質的利益を至上価値とする金融・経済に関連した政治のことをさす。こ
の時代に自民党は「経済的繁栄と物質的に豊かな生活の実現」を目標に
日本を高度経済成長へリードした(五十嵐 2010, 15)
。それをきっかけ
に利益分配や産業優先政策が行われた。そして「高成長か低成長か、赤
字財政か財政再建か」といったことが政治の場で多く議論され、たくさ
んの経済・金融政策がうまれた(五十嵐 2010, 16)
。
しかし経済大国から脱工業化社会へと日本が変容を始めた頃、IP によ
る副作用に気付いた人々が現れ、
「LP」が注目を浴びるようになる。急
速に工業化したことにより公害が生じ、それに不満を持った人々が住民
運動を行うようになったのである。この頃から、この節の第 4 段落で紹
介したような LP が新たな課題として取り組まれるようになった。IP が
105
物質的充実を図る政治であるのに対し、LP は精神的充実に価値を置く
ものである。
最後に 1990 年代から今日に至るまで「グローバリゼーション・ポリ
ティクス」が本格的に行われている。これはグローバリゼーションに伴
って発生する課題に対応するための政治である。具体的には外国との取
引に対する規制緩和やテロ対策などがある。
本稿では LP に関心のある人の方がそうでない人よりも地方選挙に関
心を持つと想定している。なぜなら地方選挙では自分の暮らす地域なら
ではの LP について議論される可能性があるからである。第一節で紹介
した「京都市長選挙マニフェスト志向型公開討論会」における両立候補
者の政策もまさに LP である。LP に関心がある人は、自分の地域で行
われている LP の政策についてもある程度知識があるか、興味を抱いて
いると思われる。そういった知識や関心を持っている有権者はそうでな
い者よりも、地元で行われる LP の政策がよりよいものになってほしい
という気持ちや、その政策の改善によって自分の暮らす地域が住みやす
い場所となってほしいという気持ちが強いと考えられる。このような有
権者であれば、関心のある LP を行うと主張している立候補者がいる地
方選挙やその LP が争点となっている地方選挙には興味を持つと私は考
えた。
また篠原(1985)も LP を実現する重要な担い手として地方自治体を
挙げている。彼は「ライブリー・ポリティクスとは人間のために、人間
らしい生と生活とを保障する自治体のためにある言葉であるといって
もよい」と述べている(篠原 1985, 22)
。さらに彼は、LP が地方政治の
場でどのように行われてきたのかについて例を用いて紹介している。3
彼の著書からも、LP に興味がある人は、それを担う地方政府の選挙に
興味を持つと考えられる。
国政選挙でも LP についての政策を掲げる立候補者がいる場合や LP
が争点である場合もあるため、LP が国政選挙の関心度に影響を与える
ことも考えられる。しかし国政選挙では消費税増税など国全体に関わる
大きな事柄が争点となる場合が多いうえに、地方選挙での方が生活に密
着した細かい LP が争点となりうる。
以上のことから、LP が選挙関心度に与える影響の大きさは、国政選
挙よりも地方選挙での方が大きいという仮説を立てた。4
106
3. 分析枠組み
3-1. 使用するデータ
ライブリー・ポリティクス仮説を検証するために「第 43 回衆議院議
員総選挙についての意識調査(2003 年)
」
・
「第 20 回参議院議員通常選
挙についての意識調査(2004 年)
」
・
「第 15 回統一地方選挙に関する意
識調査(2003 年)
」を利用する(以下、順番に「衆議院(2003)
」
・
「参
議院(2004)
」
・
「地方(2003)
」と略す)
。5 地方(2003)では知事選
挙・道府県議会議員選挙・市区町村長選挙・市区町村議会議員選挙の 4
つについて調査を行っており、それらすべてを使用する。6 明るい選挙
推進協会の意識調査の中から、LP の作業定義ができる質問があり、衆
院選・参院選・統一地方選の年が近いものを選んだ。
3-2. 分析方法
上記のデータを使用し、本稿では重回帰分析によって仮説を検証する。
図 2 に示すとおり従属変数を「選挙関心」
、独立変数を「LP」とする。
そしてそれらと共にコントロール変数として「IP」
・
「地域愛着度」
・
「居
住年数」
・
「社会集団加入」
・
「政党支持の強さ」
・
「教育程度」
・
「年齢」
・
「性
別」を入れる。
「地域愛着度」
・
「居住年数」
・
「社会集団加入」
・
「政党支持の強さ」
・
「教
育程度」
・
「年齢」
・
「性別」をコントロール変数として選んだのにはそれ
ぞれ理由がある。まず「地域愛着度」を入れたのは、それが LP への関
心と、
地方選挙への関心の 2 つに影響を与える可能性があるからである。
地元に愛着を持っている人ほど自分の地域で起こることには敏感だと
思われるので、地元で行われている LP の政策についてや、次の選挙で
地元の行政を担う者が誰になり、その者がどのように地元の暮らしを維
持・改善してくれるのかについて興味を持つと考えられる。「居住年
数」
・
「社会集団加入」
・
「政党支持の強さ」
・
「教育程度」
・
「年齢」
・
「性別」
7
は先行研究を参考に分析に入れた。
<地方選挙の場合>
+
LP
IP
図 2:モデル図
<国政選挙の場合>
LP
IP
選挙関心
+
選挙関心
コントロール変数
コントロール変数
予想される結果は選挙レベルによって異なる。地方選挙の場合は LP
が選挙関心度に与えるプラスの影響は大きいと予想される。国政選挙の
場合も LP が争点となることがありうるためプラスの影響がはたらくが、
107
その影響力は地方選挙に比べると小さいと予想される。
3-3. 作業定義
(1) 従属変数
調査が行われた選挙に対してどれくらい関心を持っていたか尋ねた
質問がすべての調査にあったため、それを政治関心の作業定義に使用し
た。そしてその回答である「非常に関心をもった」
・
「多少は関心をもっ
た」
・
「ほとんど関心をもたなかった」
・
「全く関心をもたなかった」を順
番に「3」から「0」と定義した。
「わからない」は欠損値として分析か
ら省いた。
(2) 独立変数
ライブリー・ポリティクス(&インタレスト・ポリティクス)
調査が行われた選挙において回答者がどのような問題を考慮したの
かを尋ねた質問がすべての調査にあったため、それをライブリー・ポリ
ティクスとインタレスト・ポリティクスの作業定義に使用した。
これらの質問における選択肢は選挙ごとに異なるので、五十嵐(2010)
、
五十嵐・シュラーズ(2012)
、角(1999)
、篠原(1985)を参考に選択
肢を LP、IP、ハイ・ポリティクス、グローバリゼーション・ポリティ
クス、どれにもあてはまらないものに分類し、表 1 にまとめた。地方
(2003)にはハイ・ポリティクスとグローバリゼーション・ポリティク
スにあてはまる選択肢がなかったため、IP のみをコントロール変数とし
て分析に入れた。
LP の作業定義の仕方は 3 つのパターンを考え、分析を 3 つに分けて
行った。
1 つ目は LP を選択した「個数」を点数化するという方法である。各
選挙において LP として考えられる選択肢は 6 あるいは 8 つある。そこ
で「LP を一つも選択しなかった人」に「0」というスコアーを与え、残
りの人は選択した個数をそのままコード化した。つまり衆議院(2003)
の場合は 0 から 6 のスコアーを取った変数ができる。この変数をつくる
のは、選挙において考慮している LP の数が多ければ多いほどそれに興
味を持っていると考えられるからである。
しかしこの尺度だけでは不充分だと考えたので、2 つ目に「ダミー変
数をつくる」という方法をとった。
「LP を一つも選択しなかった人」に
「0」
、
「LP を一つ以上選択した人」に「1」というスコアーを与えた。
なぜこのような尺度が必要かというと、LP の選択数が少なくてもそれ
に対して強い関心を抱いている場合もあるからである。例えば環境問題
について強い関心を持っていて、それに関連した市民運動に参加してい
る人がいたとする。その人は他の LP にはとくに興味がないため、1 つ
目の定義の仕方だと「1」という点数になり、6 あるいは 8 が最高値で
ある尺度の中では小さいスコアーとなる。しかしその人の環境問題に対
108
する熱心さを考慮すると、その人は 1 より大きな数字として定義された
人と同じくらい LP に関心を寄せていることになるかもしれない。
したがって LP について「広く浅く考えている人」と「狭く深く考え
ている人」がいると考え、これまでに紹介した 2 通りの LP 変数を作成
した。
表1:選挙で考慮した問題の分類
衆議院(2003)
参議院(2004)
地方(2003)
ライブリー
インタレスト
ハイ
グローバリゼーション
その他
福祉・介護
年金問題
環境・公害問題
景気・物価
財政再建
税金問題
政治倫理・政治改革
憲法問題
国際・外交問題
政権のあり方
行政改革
地方分権
土地・住宅問題
農林漁業対策
教育問題
福祉・介護
年金問題
中小企業対策
環境・公害問題
土地・住宅問題
農林漁業対策
教育問題
福祉・医療
税金問題
中小企業対策
地方分権
防衛問題
構造改革
中小企業対策
教育・文化
災害対策
公害・環境問題
地域振興
土地・住宅・交通問題
税金
財政再建
景気・物価
地方自治のあり方
市町村合併(※1)
景気・物価
財政再建
防衛問題
構造改革
政治倫理・政治改革
憲法問題
国際・外交問題
政権のあり方
行政改革
農業対策
各種の公共施設
※1 市長選・市議選のみ
※2 下線が引いてある選択肢は、LPの「個数統一指標」で使用した、3つの選挙で共通する選択肢をさす
3 つ目の定義の仕方は、
1つ目と同様に選択した個数を点数化するが、
選択肢をすべての選挙で統一するという方法である。本稿の目的は地方
選挙と国政選挙において、LP が選挙関心度に与える効果の違いを確認
することである。しかし補遺に示すとおり、LP を定義するために使用
する質問文の選択肢は選挙によって異なる。そこですべての選挙で共通
して聞かれている LP のみで分析を行う必要があると考え、この変数を
つくった。
なお共通する選択肢とは表 1 で下線を引いているものである。
本稿ではこれらの定義の仕方を順番に「個数指標」
・
「ダミー指標」
・
「個
数統一指標」と記述する。
なお、どの定義の仕方においても、
「0」には
(a) 「政策は考えなかった」を選択した人
(b) 何かしらの選択肢を選んだが、LP は 1 つも選択しなかった人
が含まれている。
また「わからない」
・
「その他」を選択した場合は欠損値として分析対
象から除外した。
109
IP も同じように 3 つの定義の仕方で操作化した。
(3) コントロール変数
地域愛着度
地域愛着度を定義する質問として、すべての調査に共通してあった
「あなたは、この市(区・町・村)にどの程度愛着を感じていますか。
この中からお答えください」を使用した。この質問の選択肢である「非
常に愛着を感じている」に「2」
、
「かなり愛着を感じている」に「1」
、
「ど
ちらともいえない」に「0」
、
「あまり愛着を感じていない」に「-1」
、
「ま
ったく愛着はない」に「-2」というスコアーを与えた。
居住年数
回答者が今住んでいる市区町村に何年くらい住んでいるか尋ねた質
問がすべての調査にあったため、それを居住年数の作業定義に使用した。
本稿ではこの変数のスコアーを実際の年数に近づけるために、選択肢の
「3 年未満」を「1.5」
、
「3 年以上」を「6」
、
「10 年以上」を「14.5」
、
「20
年以上(生まれてからずっと)
」を「20」と定義した。8
社会集団加入ダミー
社会集団に加入しているかどうかを定義する質問として、すべての調
査にある「あなたは、このような団体に加入していますか。あればいく
つでも結構ですからあげてください」を使用した。選択肢の中の「どれ
にも加入していない」を選んだ人に「0」
、いずれかの団体を 1 つでも選
択した人に「1」というスコアーを与えてダミー変数を作成した。9
政党支持の強さ
政党支持の強さは 2 つの質問文を用いて定義した。まず、すべての意
識調査にあった「あなたは、ふだん何党を支持していらっしゃいますか」
という質問に対して「支持政党なし」と答えた人に「0」というスコア
ーを与えた。上記の質問で何らかの政党を支持していると答えた人に対
し、その政党を強く支持しているか、それほどでもないか尋ねる質問が
すべての調査に用意されていた。そこで、それらの選択肢である「強く
支持している」を選択した人に「2」
、
「それほどでもない」を選択した
人に「1」というスコアーを与えた。
教育程度
教育程度の操作化に使用した質問は、すべての調査にあった「あなた
は学校はどこまでいらっしゃいましたか(在学中・中退は卒業とみなす)
」
である。本稿ではこの変数のスコアーを実際の修学年数に近づけるため
に、
「小・高小・新中卒」に「9」
、
「旧中・新高卒」に「12」
、
「新高専・
短大・専修学校卒」に「14」
、
「旧高専大・新大卒」に「16」
、
「大学院(修・
110
博)卒」に「19.5」というスコアーを与えた。10
年齢
年齢については、すべての選挙の調査に「あなたのお年は満でおいく
つですか」という質問があったためこちらを使用する。参議院(2004)
と地方(2003)では回答者の実年齢をそのまま分析に投入した。衆議院
(2003)では実年齢がそのままデータ化されておらず、同年代ごとに年
齢グループがつくられていたのでそれをコード化した。この尺度も実年
齢に近づけるために平均を取って、
「20~24 歳」に「22」
、
「25~29 歳」
に「27」
、
「30~39 歳」に「34.5」
、
「40~49 歳」に「44.5」
、
「50~59
歳」に「54.5」
、
「60~69 歳」に「64.5」
、
「70~79 歳」に「74.5」
、
「80
歳以上」に「84.5」というスコアーを与えた。
性別
「男性」と答えた人に「0」
、女性と答えた人に「1」というスコアー
を与えてダミー変数をつくった。
なおすべてのコントロール変数において「わからない」という選択肢
は欠損値として扱い分析から除外した。
すべての作業定義で使用した質問文と選択肢は補遺に掲載する。
4. 分析結果
分析枠組みを確認したところで分析結果を示す。
図 3-1-1 から図 3-1-3
は衆議院(2003)の分析結果で、図 3-2-1 から図 3-2-3 は参議院(2004)
の分析結果である。図 3-3-1 以降は地方(2003)の分析結果で、知事選・
県議選・市長選・市議選という順番に並んでいる。これらの図では独立
変数のベータ値(標準化偏回帰変数)を記号で示しており、その記号の
形で危険率をあらわしている。11 なお、推定結果の表は補遺に掲載す
る。
なお LP 変数の行の右端にある「○位」という記述は、独立変数のベ
ータ値の大きさを比較した時に LP のそれが何番目かを示している。順
位の値が小さいほど(1 位に近いほど)ベータ値が大きく、選挙関心度
に強い影響を与えている。LP のベータ値の順位をすべて図示したもの
が図 4 である。
111
図3-1-1:衆院選の分析結果(個数指標)
-0.1
0
0.1
0.2
0.3
LP
IP
地域愛着度
居住年数
社会集団加入ダミー
政党支持の強さ
教育程度
年齢
性別ダミー
0.4 (ベータ)
5位
N=1820 調整済みR二乗値:0.216
■:危険率1%未満 ◆:危険率1%以上5%未満 ▲:危険率5%以上10%未満
×:危険率10%以上
図3-1-2:衆院選の分析結果(ダミー指標)
-0.1
0
0.1
0.2
0.3
0.4 (ベータ)
6位
LP
IP
地域愛着度
居住年数
社会集団加入ダミー
政党支持の強さ
教育程度
年齢
性別ダミー
N=1820 調整済みR二乗値:0.218
■:危険率1%未満 ◆:危険率1%以上5%未満 ▲:危険率5%以上10%未満
×:危険率10%以上
図3-1-3:衆院選の分析結果(個数統一指標)
-0.1
0
0.1
0.2
0.3
0.4 (ベータ)
9位
LP
IP
地域愛着度
居住年数
社会集団加入ダミー
政党支持の強さ
教育程度
年齢
性別ダミー
N=1820 調整済みR二乗値:0.215
■:危険率1%未満 ◆:危険率1%以上5%未満 ▲:危険率5%以上10%未満
×:危険率10%以上
112
図3-2-1:参院選の分析結果(個数指標)
-0.1
0
0.1
0.2
0.3
0.4 (ベータ)
4位
LP
IP
地域愛着度
居住年数
社会集団加入ダミー
政党支持の強さ
教育程度
年齢
性別ダミー
N=1830 調整済みR二乗値:0.157
■:危険率1%未満 ◆:危険率1%以上5%未満 ▲:危険率5%以上10%未満
×:危険率10%以上
図3-2-2:参院選の分析結果(ダミー指標)
-0.1
0
0.1
0.2
0.3
LP
IP
地域愛着度
居住年数
社会集団加入ダミー
政党支持の強さ
教育程度
年齢
性別ダミー
0.4 (ベータ)
2位
N=1830 調整済みR二乗値:0.166
■:危険率1%未満 ◆:危険率1%以上5%未満 ▲:危険率5%以上10%未満
×:危険率10%以上
図3-2-3:参院選の分析結果(個数統一指標)
-0.1
0
0.1
0.2
0.3
LP
IP
地域愛着度
居住年数
社会集団加入ダミー
政党支持の強さ
教育程度
年齢
性別ダミー
0.4 (ベータ)
7位
N=1830 調整済みR二乗値:0.146
■:危険率1%未満 ◆:危険率1%以上5%未満 ▲:危険率5%以上10%未満
×:危険率10%以上
113
図3-3-1:知事選の分析結果(個数指標)
-0.1
0
0.1
0.2
0.3
LP
IP
地域愛着度
居住年数
社会集団加入ダミー
政党支持の強さ
教育程度
年齢
性別ダミー
0.4 (ベータ)
5位
N=543 調整済みR二乗値:0.177
■:危険率1%未満 ◆:危険率1%以上5%未満 ▲:危険率5%以上10%未満
×:危険率10%以上
図3-3-2:知事選の分析結果(ダミー指標)
-0.1
0
0.1
0.2
0.3
LP
IP
地域愛着度
居住年数
社会集団加入ダミー
政党支持の強さ
教育程度
年齢
性別ダミー
0.4 (ベータ)
3位
N=543 調整済みR二乗値:0.212
■:危険率1%未満 ◆:危険率1%以上5%未満 ▲:危険率5%以上10%未満
×:危険率10%以上
図3-3-3:知事選の分析結果(個数統一指標)
-0.1
0
0.1
0.2
0.3
LP
IP
地域愛着度
居住年数
社会集団加入ダミー
政党支持の強さ
教育程度
年齢
性別ダミー
0.4 (ベータ)
3位
N=543 調整済みR二乗値:0.184
■:危険率1%未満 ◆:危険率1%以上5%未満 ▲:危険率5%以上10%未満
×:危険率10%以上
114
図3-4-1:県議選の分析結果(個数指標)
-0.1
0
0.1
0.2
0.3
0.4 (ベータ)
1位
LP
IP
地域愛着度
居住年数
社会集団加入ダミー
政党支持の強さ
教育程度
年齢
性別ダミー
N=1677 調整済みR二乗値:0.185
■:危険率1%未満 ◆:危険率1%以上5%未満 ▲:危険率5%以上10%未満
×:危険率10%以上
図3-4-2:県議選の分析結果(ダミー指標)
-0.1
0
0.1
0.2
0.3
0.4 (ベータ)
1位
LP
IP
地域愛着度
居住年数
社会集団加入ダミー
政党支持の強さ
教育程度
年齢
性別ダミー
N=1677 調整済みR二乗値:0.203
■:危険率1%未満 ◆:危険率1%以上5%未満 ▲:危険率5%以上10%未満
×:危険率10%以上
図3-4-3:県議選の分析結果(個数統一指標)
-0.1
0
0.1
0.2
0.3
LP
IP
地域愛着度
居住年数
社会集団加入ダミー
政党支持の強さ
教育程度
年齢
性別ダミー
0.4 (ベータ)
1位
N=1677 調整済みR二乗値:0.181
■:危険率1%未満 ◆:危険率1%以上5%未満 ▲:危険率5%以上10%未満
×:危険率10%以上
115
図3-5-1:市長選の分析結果(個数指標)
-0.1
0
0.1
0.2
0.3
LP
IP
地域愛着度
居住年数
社会集団加入ダミー
政党支持の強さ
教育程度
年齢
性別ダミー
0.4 (ベータ)
1位
N=436 調整済みR二乗値:0.170
■:危険率1%未満 ◆:危険率1%以上5%未満 ▲:危険率5%以上10%未満
×:危険率10%以上
図3-5-2:市長選の分析結果(ダミー指標)
-0.1
0
0.1
0.2
0.3
LP
IP
地域愛着度
居住年数
社会集団加入ダミー
政党支持の強さ
教育程度
年齢
性別ダミー
0.4 (ベータ)
1位
N=436 調整済みR二乗値:0.223
■:危険率1%未満 ◆:危険率1%以上5%未満 ▲:危険率5%以上10%未満
×:危険率10%以上
図3-5-3:市長選の分析結果(個数統一指標)
-0.1
0
0.1
0.2
0.3
LP
IP
地域愛着度
居住年数
社会集団加入ダミー
政党支持の強さ
教育程度
年齢
性別ダミー
0.4 (ベータ)
2位
N=436 調整済みR二乗値:0.164
■:危険率1%未満 ◆:危険率1%以上5%未満 ▲:危険率5%以上10%未満
×:危険率10%以上
116
図3-6-1:市議選の分析結果(個数指標)
-0.1
0
0.1
0.2
0.3
0.4 (ベータ)
1位
LP
IP
地域愛着度
居住年数
社会集団加入ダミー
政党支持の強さ
教育程度
年齢
性別ダミー
N=1284 調整済みR二乗値:0.230
■:危険率1%未満 ◆:危険率1%以上5%未満 ▲:危険率5%以上10%未満
×:危険率10%以上
図3-6-2:市議選の分析結果(ダミー指標)
-0.1
0
0.1
0.2
0.3
LP
IP
地域愛着度
居住年数
社会集団加入ダミー
政党支持の強さ
教育程度
年齢
性別ダミー
0.4 (ベータ)
1位
N=1284 調整済みR二乗値:0.271
■:危険率1%未満 ◆:危険率1%以上5%未満 ▲:危険率5%以上10%未満
×:危険率10%以上
図3-6-3:市議選の分析結果(個数統一指標)
-0.1
0
0.1
0.2
0.3
LP
IP
地域愛着度
居住年数
社会集団加入ダミー
政党支持の強さ
教育程度
年齢
性別ダミー
0.4 (ベータ)
1位
N=1284 調整済みR二乗値:0.219
■:危険率1%未満 ◆:危険率1%以上5%未満 ▲:危険率5%以上10%未満
×:危険率10%以上
117
図 4:ライブリー・ポリティクスのベータ値の順位
個数指標
ダミー指標
影響力 順位
影響力 順位
大
小
大
1
← 県議・市長・市議
2
2
← 参議院
3
3
← 知事
1
← 県議・市長・市議
4
← 参議院
4
5
← 衆議院・知事
5
6
6
7
7
8
8
9
小
← 衆議院
9
個数統一指標
影響力 順位
大
1
← 県議・市議
2
← 市長
3
← 知事
4
5
6
7
← 参議院
8
小
9
← 衆議院
分析結果から、どの選挙でも、そしてどの指標でも LP は選挙関心度
と正の相関があることを確認できた。図 3-1-1 から図 3-6-3 の LP の危
険率をみると、それらはすべて 5%未満である。また LP のベータ値は
すべて正数である。つまり国政選挙・地方選挙の両方において、1 つで
も LP を選挙で考慮した人は全く考慮しなかった人より選挙関心が高い
こと、そして選択した LP の個数が多い人の方が少ない人よりも選挙関
心が高いという 2 つのことが計量的に確認できた。
しかし選挙関心度を LP がどの程度規定するのかが国政選挙と地方選
挙では大きく異なる。本稿では衆院選・参院選・統一地方選で異なるデ
ータを使用しているため、すべての分析結果のベータ値を比較して、ど
の選挙で LP がもっとも選挙関心度に影響を与えているかを確認するこ
118
とはできない。しかし分析に投入した 9 つある独立変数のベータ値を比
べることで、相対的にどの選挙において LP の効果が大きいかを確認す
ることはできる。したがってこの方法で議論を進める。
まず国政選挙では LP よりもいくつかのコントロール変数の方が選挙
関心度に影響を与えることが多いという結果が出た。図 4 に示すとおり
衆院選での LP のベータ値の順位は、個数指標では 5 位、ダミー指標で
は 6 位、個数統一指標では 9 位である。つまり衆院選において、LP と
選挙関心度の相関は、ほとんどのコントロール変数と選挙関心度の相関
に比べると小さいことが確認できる。次に、
参院選での LP の規定度は、
ダミー指標では 2 位と比較的大きいが、個数指標と個数統一指標では順
に 4 位・7 位とばらついている。やはり衆院選同様、コントロール変数
の方が選挙関心度を規定する場合が多い。
一方、ほとんどの地方選挙では国政選挙と異なる結果が出ており、選
挙関心度に対する LP の影響力がどのコントロール変数よりも大きいと
いう結果が確認できた。図 4 に示すとおり、県議選・市議選では 3 つの
指標すべてで、市長選では個数統一指標以外で、LP の順位が 1 位であ
った。とりわけ市長選のダミー指標(図 3-5-2)
・市議選の個数指標・ダ
ミー指標(図 3-6-1・図 3-6-2)では LP のベータ値とコントロール変数
のベータ値との間に大きな差があり、選挙関心に対する効果の大きさは
LP が他のコントロール変数よりもかなり大きいことが窺える。
以上のことから、国政選挙・地方選挙ともに LP は選挙関心度に対し
てプラスの影響を与えているが、国政選挙よりもほとんどの地方選挙で
の方がその影響力は大きいと解釈した。つまり LP 仮説はおおむね支持
された。
なお地方選挙の中で知事選だけはすべての分析で LP の規定度が相対
的に低かった。図 4 に示すとおり、LP のベータ値の順位は、個数指標
で 5 位、ダミー指標では 3 位、個数統一指標では 3 位であった。調査時
点で知事選が行われていたのは 11 の都道県だけであること、そしてサ
ンプル数が 662 と少なかったことが原因で、他 3 つの地方選挙とは異な
るパターンがあらわれたと思われる。12
5. まとめ
本稿では「どのような人が地方選挙に関心を持つのか」というリサー
チクエスチョンを設定し、その仮説として「LP に関心を寄せている人
は、そうでない人よりも地方選挙に関心を持つ」を挙げ、計量的に分析
した。その結果すべての選挙で LP に関心のある人はそうでない人より
も選挙関心度が高いが、両者の相関は国政選挙よりもほとんどの地方選
挙での方が大きいということが確認できた。つまり想定した結果をおお
むね確認することができた。
LP 仮説を検証するにあたって、本来は同じ被調査者を対象に、
「考慮
119
した問題」の効果が地方選挙と国政選挙で変わるのかを確認すべきであ
る。今回使用したものと似た質問があり、国政選挙・地方選挙両方につ
いて聞いている調査があれば、本稿と同じ分析モデルで検証を行うこと
で、この仮説を更に追究することができるだろう。
そして、有権者が選挙で考慮する問題は、同じレベルの選挙であって
も年によって変わる可能性がある。例えば 2005 年の衆院選は郵政選挙
と呼ばれ、当時の首相であった小泉氏が掲げた「郵政民営化」が大きな
争点となった。この選挙について聞いた「第 44 回 衆議院議員総選挙に
ついての意識調査」にも「今回の選挙で、どのような問題を考慮しまし
たか。この中にあればいくつでもあげてください」という質問があり、
「郵政民営化」という選択肢があった。そして回答者の約 46.3%がそれ
を選んでいる。つまり選挙によっては特異な問題が争点となっている場
合があるため、LP が選挙関心度に影響を与えるかどうかは選挙が実施
された年の状況によっても変わるかもしれない。
LP は今、ますます多様化している。京都市が発行している「きょう
と市民しんぶん」の紙面には、たくさんの LP が登場する。2013 年 12
月 1 日号には「平成 25 年 市政この 1 年」という特集があり、京都市
政の取り組みを紹介している。その中には「119 番通報などの多言語通
訳サービスを開始」
したことや
「男性のための DV 電話相談窓口を開設」
したことなど、LP に関連した政策がたくさんあった。13 日本に外国人
居住者が少なかった頃、そして DV というフレーズが浸透していなかっ
た頃には考えられなかった政策であろう。今後もこういった細かな LP
のニーズが出てくることで、LP の効果が地方選挙の関心度に、今以上
に影響を与えるようになるかもしれない。
【謝辞】
本論文を書くにあたって、たくさんの方にお世話になりました。西
澤先生には、本論文のリサーチデザインや分析の仕方などについて、
丁寧に指導していただきました。
「分かりやすい論文」を書くための
術など、たくさんのことを学ぶことができました。また西澤ゼミの
皆様には、学園祭期間中にも関わらず論文に目を通していただき、
多くのアドバイスをいただきました。自分では気付くことのできな
かった間違いや分かりにくい箇所を修正することができました。こ
の論文を『法と政治のディスクール 同志社大学学生論集 2013 年
度』に掲載するにあたっては、編集委員の先生方にもお世話になり
ました。詳細で貴重な査読コメントをいただけて、大変勉強になり
ました。この場を借りて、皆様に心よりお礼を申し上げます。
(1)
2012 年 1 月 14 日に開催された『京都市長選挙マニフェスト志向
型公開討論会』
(主催=社団法人 京都青年会議所)の配布資料であ
る「参考資料 事前アンケートの質問に対する立候補予定者からの回
答(門川大作氏)
」を引用した。
120
(2)
注(1)で述べた討論会の配付資料である「参考資料 事前アンケー
トの質問に対する立候補予定者からの回答(中村和雄氏)
」を引用し
た。
(3)
例えば「
『池子米軍住宅に反対して自然と子供を守る会』の住民投
票条例直接請求と市長リコール運動、またそれにつづく逗子市市長
選の勝利」が LP として紹介されている(篠原 1985, 25)
。
(4)
この仮説には、前段階として「ライブリー・ポリティクスは地方
選挙の関心に影響を与える」という仮説があると『法と政治のディ
スクール 2013 年度』の編集委員の先生からご指摘をいただいた。
本稿では、この仮説があてはまるという前提のもとで議論を進める。
(5)
「第 43 回 衆議院議員総選挙についての意識調査(2003 年)
」
・
「第
44 回 衆議院議員総選挙についての意識調査(2005 年)
」
・
「第 20 回
参議院議員通常選挙についての意識調査(2004 年)
」は明るい選挙
推進協会が実施した世論調査である。それをレヴァイアサン・デー
タ・バンク(LDB、木鐸社)より同志社大学が購入したものを使用
した。いずれも、同志社大学・法学部の西澤由隆先生のご指導と便
宜により利用することができた。
また「第 15 回 統一地方選挙に関する意識調査(2003 年)
」
(蒲島
郁夫・明るい選挙推進協会)の個票データは東京大学社会科学研究
所附属社会調査・データアーカイブ研究センターSSJ データアーカ
イブから提供を受けた。
これらのデータを公開・寄託され、利用できるようにしてくださ
った方々に感謝いたします。
(6)
地方(2003)の調査が行われた時点で知事選・県議選・市長選・
市議選が行われた地域とそうでない地域があったため、それぞれ回
答数が異なる。回答数はそれぞれ、知事選が 662、県議選が 2012、
市長選が 510、市議選が 1522 である。
(7)
「居住年数」
・
「政党支持の強さ」
・
「教育程度」
・
「性別」は三宅・
木下・間場(1967)の研究を参考に分析に入れた。彼らは京都府宇
治市の市民を対象にパネル調査を行っている。そして参院選・府知
事選・市長選の関心度を規定するものをクロス表分析によって確認
している。その結果「政党支持の強さ」
・
「教育程度」
・
「性別」はす
べての選挙において統計的に有意な結果が出た。そして「居住年数」
は市長選でのみ有意であった。
「年齢」は三宅・木下・間場(1967)の分析では有意にならなか
った。一方、京都市選挙管理委員会(1980, 執筆者:蒲島郁夫)が
行った、
「市政への関心度」を従属変数に、そして 6 つのデモグラフ
ィックな要因を独立変数にした林数量化 I 類の分析によると、年齢
と市政への関心度には「ほぼ直線的な正の相関」があった(京都市
選挙管理委員会 執筆者:蒲島郁夫 1980, 16)
。したがって本稿では
121
「年齢」も分析に入れた。
「社会集団加入」を分析に入れた理由は、社会集団が人の政治的
態度を形成する重要なアクターだということを川上(1994)
、
Dawson・Prewitt・Dawson(1977)
、大矢(1995)
、蒲島(1988)
などが論じているからである。彼らによると、社会集団に加入する
ことで、人は政治や社会について話す機会が増えたり社会の仕組み
が理解できるようになったりするため、政治的有効性感覚・政治関
心が強化されたり、政治参加が促されたりする。したがって社会集
団に入っている人の方がそうでない人よりも選挙について関心を持
つようになる機会が多いと考えた。
(8)
「3 年未満」
・
「3 年以上」
・
「10 年以上」という選択肢については、
平均年数を取ることで順番に「1.5」
・
「6」
・
「14.5」と定義した。
「20
年以上(生まれてからずっと)
」については、年数の最高値が分から
ず平均値を算出できないため「20」と定義した。
(9)
社会集団加入をダミー変数として操作化した理由は、政治色のな
い社会集団であっても政治的態度が形成されるということが先行研
究で実証されているからである。Almond・Verba (1963) が 5 つの
民主主義国を対象に行った実証研究によると、たとえ非政治的な団
体であっても、それに加入している人の方が団体に一つも加入して
いない人より 1) 政治的有力感がある 2) 政治について議論する 3)
政治問題について意見表明をする意志が大きいということが確認さ
れた。また蒲島(1988)も日本人を対象に、どのような組織への加
入が政治参加を促すのかについて林数量化Ⅰ類を用いて分析してい
る。その結果によると非政治的団体の加入者は非加入者よりも投
票・選挙運動・地域運動や住民運動に参加することが確認された。
以上の理由から本稿では、どのような社会集団に入っている人でも、
入っていない人よりは選挙に関心を持つと考え、ダミー変数を作成
した。
(10)
文部科学省 生涯学習政策局 生涯学習推進課 専修学校教育振興
室(2013)によると、専修学校では学科によって学ぶ年数が異なる
(1 年制から 4 年制まで)
。また短期大学も学校によって修業年数が
異なるが、2 年制である場合が多い。したがって本稿では「新高専・
短大・専修学校卒」を選択した人が全員 2 年制の学校に通ったと仮
定し操作化した。
大学院も学校によって修業年数が異なるが、一般的には修士課程
の就業年数は 2 年、博士課程は修士課程修了後の 3 年である。した
がって、修士課程で卒業した者が 18 年間、博士課程で卒業した者が
21 年間学校に通ったと仮定して、その間を取った「19.5」というス
コアーを「大学院(修・博)卒」に与えた。
(11) 標準化偏回帰係数とは独立変数が 1 標準偏差分変化した時の、従
属変数の変化量をあらわす(中野 2004, 135)
。これは独立変数と従
122
属変数を「平均 0、分散 1 に標準化した上で求めた偏回帰係数」な
ので、独立変数の間で、従属変数に対する効果の大きさを比較する
ことが可能である(寺島 2007, 168)
。
(12) 11 の都道県とは北海道・岩手県・東京都・神奈川県・福井県・三
重県・鳥取県・島根県・福岡県・佐賀県・大分県である。
(13) 『きょうと市民しんぶん』 2013 年 12 月 1 日号 「
【特集】平成
25 年 市政この 1 年」 8-9 面.
123
<補遺>
●推定結果の表
表 2-1-1:衆議院の分析結果(個数指標)
ベータ
危険率
LP
IP
地域愛着度
居住年数
社会集団加入ダミー
政党支持の強さ
.067
.047
.056
-.063
.064
.277
.004
.046
.010
.006
.003
.000
教育程度
年齢
性別ダミー
.206
.000
.257
.000
-.069
.001
N=1820 調整済み R 二乗値:0.216
表 2-1-2:衆議院の分析結果(ダミー指標)
ベータ
危険率
LP
IP
地域愛着度
居住年数
社会集団加入ダミー
.068
.071
.058
-.061
.064
.002
.001
.008
.007
.003
政党支持の強さ
教育程度
年齢
性別ダミー
.274
.205
.259
-.070
.000
.000
.000
.001
N=1820 調整済み R 二乗値:0.218
表 2-1-3:衆議院の分析結果(個数統一指標)
ベータ
危険率
LP
IP
地域愛着度
居住年数
.054
.055
.057
-.063
.017
.016
.010
.006
社会集団加入ダミー
政党支持の強さ
教育程度
年齢
性別ダミー
.063
.003
.277
.206
.000
.000
.262
-.066
.000
.002
N=1820 調整済み R 二乗値:0.215
124
表 2-2-1:参議院の分析結果(個数指標)
ベータ
危険率
LP
IP
地域愛着度
居住年数
社会集団加入ダミー
政党支持の強さ
教育程度
年齢
.135
.072
.000
.002
.079
-.068
.050
.173
.144
.217
.001
.004
.027
.000
.000
.000
性別ダミー
-.085
.000
N=1830 調整済み R 二乗値:0.157
表 2-2-2:参議院の分析結果(ダミー指標)
ベータ
危険率
LP
.173
.000
IP
地域愛着度
居住年数
社会集団加入ダミー
政党支持の強さ
.069
.076
-.067
.051
.171
.002
.001
.005
.023
.000
.155
.000
.217
.000
-.079
.000
N=1830 調整済み R 二乗値:0.166
教育程度
年齢
性別ダミー
表 2-2-3:参議院の分析結果(個数統一指標)
ベータ
LP
IP
地域愛着度
居住年数
社会集団加入ダミー
.072
.094
.081
-.067
.053
危険率
.002
.000
.000
.006
.019
.177
.000
教育程度
.143
.000
年齢
.223
.000
性別ダミー
-.077
.001
N=1830 調整済み R 二乗値:0.146
政党支持の強さ
125
表 2-3-1:知事選の分析結果(個数指標)
ベータ
危険率
LP
IP
地域愛着度
居住年数
社会集団加入ダミー
政党支持の強さ
教育程度
年齢
.123
.164
.005
.000
.106
.039
.095
.139
.190
.129
.012
.399
.020
.001
.000
.008
性別ダミー
.032
.419
N=543 調整済み R 二乗値:0.177
表 2-3-2:知事選の分析結果(ダミー指標)
ベータ
危険率
LP
.183
.000
IP
地域愛着度
居住年数
社会集団加入ダミー
政党支持の強さ
.212
.115
.030
.087
.133
.000
.005
.508
.029
.001
.184
.000
.119
.012
.027
.491
N=543 調整済み R 二乗値:0.212
教育程度
年齢
性別ダミー
表 2-3-3:知事選の分析結果(個数統一指標)
ベータ
LP
IP
地域愛着度
居住年数
社会集団加入ダミー
.151
.161
.108
.043
.090
危険率
.000
.000
.011
.347
.026
.144
.001
教育程度
.186
.000
年齢
.122
.011
性別ダミー
.026
.518
N=543 調整済み R 二乗値:0.184
政党支持の強さ
126
表 2-4-1:県議選の分析結果(個数指標)
ベータ
危険率
LP
IP
地域愛着度
居住年数
社会集団加入ダミー
政党支持の強さ
教育程度
年齢
.202
.139
.000
.000
.118
.022
.076
.132
.022
.139
.000
.361
.001
.000
.380
.000
性別ダミー
.016
.472
N=1677 調整済み R 二乗値:0.185
表 2-4-2:県議選の分析結果(ダミー指標)
ベータ
危険率
LP
.223
.000
IP
地域愛着度
居住年数
社会集団加入ダミー
政党支持の強さ
.169
.111
.011
.071
.130
.000
.000
.648
.002
.000
.025
.307
.144
.000
.013
.554
N=1677 調整済み R 二乗値:0.203
教育程度
年齢
性別ダミー
表 2-4-3:県議選の分析結果(個数統一指標)
ベータ
LP
IP
地域愛着度
居住年数
社会集団加入ダミー
.191
.148
.117
.024
.078
危険率
.000
.000
.000
.314
.001
.135
.000
教育程度
.023
.351
年齢
.140
.000
性別ダミー
.007
.751
N=1677 調整済み R 二乗値:0.181
政党支持の強さ
127
表 2-5-1:市長選の分析結果(個数指標)
ベータ
危険率
LP
IP
地域愛着度
居住年数
社会集団加入ダミー
政党支持の強さ
教育程度
年齢
.170
.051
.001
.300
.161
.096
.114
.079
-.054
.106
.001
.051
.014
.088
.276
.042
性別ダミー
.057
.202
N=436 調整済み R 二乗値:0.170
表 2-5-2:市長選の分析結果(ダミー指標)
ベータ
危険率
LP
.267
.000
IP
地域愛着度
居住年数
社会集団加入ダミー
政党支持の強さ
.099
.146
.083
.102
.054
.028
.001
.077
.024
.225
-.057
.228
.104
.038
.057
.193
N=436 調整済み R 二乗値:0.223
教育程度
年齢
性別ダミー
表 2-5-3:市長選の分析結果(個数統一指標)
ベータ
LP
IP
地域愛着度
居住年数
社会集団加入ダミー
.143
.068
.164
.093
.114
危険率
.004
.158
.000
.060
.014
.084
.069
教育程度
-.044
.370
年齢
.111
.034
性別ダミー
.052
.248
N=436 調整済み R 二乗値:0.164
政党支持の強さ
128
表 2-6-1:市議選の分析結果(個数指標)
ベータ
危険率
LP
IP
地域愛着度
居住年数
社会集団加入ダミー
政党支持の強さ
教育程度
年齢
.265
.067
.000
.010
.134
.048
.108
.165
-.049
.094
.000
.080
.000
.000
.067
.001
性別ダミー
.025
.313
N=1284 調整済み R 二乗値:0.230
表 2-6-2:市議選の分析結果(ダミー指標)
ベータ
危険率
LP
.325
.000
IP
地域愛着度
居住年数
社会集団加入ダミー
政党支持の強さ
.096
.133
.044
.101
.150
.000
.000
.101
.000
.000
-.053
.043
.093
.001
.020
.403
N=1284 調整済み R 二乗値:0.271
教育程度
年齢
性別ダミー
表 2-6-3:市議選の分析結果(個数統一指標)
ベータ
LP
IP
地域愛着度
居住年数
社会集団加入ダミー
.241
.073
.135
.055
.114
危険率
.000
.005
.000
.046
.000
.173
.000
教育程度
-.047
.086
年齢
.092
.002
性別ダミー
.014
.574
N=1284 調整済み R 二乗値:0.219
政党支持の強さ
129
●記述統計
図5:選挙関心度の分布(選挙別)
0
10 20 30 40 50 60 70(%)
衆院選(N=2119)
非常に関心を
もった
多少は関心を
もった
ほとんど関心を
もたなかった
全く関心をもた
なかった
参院選(N=2127)
知事選(N=662)
県議選(N=1998)
市長選(N=509)
市議選(N=1510)
図6-1:ライブリー・ポリティクスとインタレスト・ポリティクスを選択した人
の割合(国政選挙) ※複数選択
0 10 20 30 40 50 60 70 (%)
LP
IP
福祉・介護
年金問題
環境・公害問題
土地・住宅問題
農林漁業対策
教育問題
景気・物価
財政再建
税金問題
中小企業対策
政策は考えなかった
衆院選
(N=2138)
参院選
(N=2143)
130
図6-2:ライブリー・ポリティクスとインタレスト・ポリティクスを選択した人
の割合(地方選挙) ※複数選択
0
10 20 30 40 50 60 70 (%)
福祉・医療
教育・文化
災害対策
LP
公害・環境
地域振興
土地・住宅・交通問題
農業対策
各種の公共施設
中小企業対策
IP
知事選 (N=662)
県議選 (N=2012)
市長選 (N=510)
市議選 (N=1522)
税金
財政再建
景気・物価
政策は考えなかった
●分析に使用した質問項目
選挙関心
衆議院(2003)
:
「今回の選挙について、あなた自身は、どれくらい関心をもちましたか。
」
参議院(2004)
:
「あなたは、今回の選挙について、どれくらい関心をもちましたか。
」
地方(2003)
:
「今回の○○選挙について、あなた自身は、どれくらい関心をもちまし
たか。
」
○選択肢はすべての選挙において共通
1. 非常に関心をもった 2. 多少は関心をもった 3. ほとんど関心を
もたなかった 4. 全く関心をもたなかった 5. わからない
ライブリー・ポリティクス&インタレスト・ポリティクス
衆議院(2003)
:
「今回の選挙で、どのような問題を考慮しましたか。この中にあればい
くつでもあげてください。
(M.A.)
」
参議院(2004)
:
「今回の選挙で、あなたはどのような問題を考慮しましたか。この中に
ありましたらおっしゃってください。いくつでも結構です。
(M.A.)
」
○上記 2 選挙の選択肢は共通
1. 福祉・介護 2. 景気・物価 3. 財政再建 4. 税金問題 5. 年金問
131
題 6. 政権のあり方 7. 環境・公害問題 8. 土地・住宅問題 9.
農林漁業対策 10. 中小企業対策 11. 政治倫理・政治改革 12. 行
政改革 13. 地方分権 14. 国際・外交問題 15. 憲法問題 16. 防
衛問題 17. 教育問題 18. 構造改革 19. その他 20. 政策は考
えなかった 21. わからない
地方(2003)
:
「○○選挙で、あなたはどのような問題を考慮しましたか。この中にあ
ればあげてください。
(M.A.)
」
○知事選・県議選の選択肢
1. 地方自治のあり方 2. 福祉・医療 3. 教育・文化 4. 災害対策 5.
公害・環境問題 6. 地域振興 7. 土地・住宅・交通問題 8. 農業
対策 9. 中小企業対策 10. 各種の公共施設 11. 税金 12. 財政
再建 13. 景気・物価 14. その他 15. 政策は考えなかった 16.
わからない
○市長選・市議選の選択肢
1. 地方自治のあり方 2. 福祉・医療 3. 教育・文化 4. 災害対策 5.
公害・環境問題 6. 地域振興 7. 土地・住宅・交通問題 8. 農業
対策 9. 中小企業対策 10. 各種の公共施設 11. 税金 12. 財政
再建 13. 景気・物価 14. 市町村合併 15. その他 16. 政策は考
えなかった 17. わからない
地域愛着度
すべての選挙において共通
「あなたは、この市(区・町・村)にどの程度愛着を感じていますか。
この中からお答えください。
」
1. 非常に愛着を感じている 2. かなり愛着を感じている 3. どちら
ともいえない 4. あまり愛着を感じていない 5. まったく愛着は
ない 6. わからない
居住年数
参議院(2004)
:
「あなたは、この市(区・町・村)に何年くらい住んでいますか。
」
衆議院(2003)&地方(2003)
:
「あなたは、この市(区・町・村)に何年ぐらい住んでいますか。
」
○選択肢はすべての選挙で共通
1. 3 年未満 2. 3 年以上 3. 10 年以上 4. 20 年以上(生まれてからず
っと) 5. わからない
社会集団加入
すべての選挙において共通
「あなたは、このような団体に加入していますか。あればいくつでも結
構ですからあげてください。
(M.A.)
」
1. 自治会 2. 婦人会 3. 青年団・消防団 4. 老人クラブ(会) 5.
PTA 6. 農協その他の農林漁業団体 7. 労働組合 8. 商工業関係
132
の経済団体 9. 宗教団体 10. 同好会・趣味のグループ 11. 住民
運動・消費者運動・市民運動の団体 12. その他 13. どれにも加
入していない 14. わからない
政党支持の強さ
衆議院(2003)
:
「あなたは、ふだん何党を支持していらっしゃいますか。
」
1. 自由民主党 2. 民主党 3. 公明党 4. 日本共産党 5. 保守新党
6. 社会民主党 7. 無所属の会 8. 自由連合 9. その他 10. 支
持政党なし 11. わからない
「
(上の質問で 1 から 9 を選択した人に対して)政党を支持する度合い
についてお聞きします。その政党を強く支持していますか、それほ
どでもありませんか。
」
1. 強く支持している 2. それほどでもない 3. わからない
参議院(2004)
:
「あなたは、ふだん何党を支持していらっしゃいますか。
」
1. 自由民主党 2. 民主党 3. 公明党 4. 日本共産党 5. 社会民主党
6. 無所属の会 7. 自由連合 8. その他 9. 支持政党なし 10.
わからない
「
(上記の質問で 1 から 8 を選択した人に対して)強く支持されていま
すか、それほどでもありませんか。
」
1. 強く支持している 2. それほどでもない 3. わからない
地方(2003)
:
「あなたは、ふだん何党を支持していらっしゃいますか。
」
1. 自由民主党 2. 民主党 3. 公明党 4. 日本共産党 5. 自由党 6.
社会民主党 7. 保守新党 8. その他 9. 支持政党なし 10. わか
らない
「
(上記の質問で 1 から 8 を選択した人に対して)強く支持されていま
すか、それほどでもありませんか。
」
1. 強く支持している 2. それほどでもない 3. わからない
教育程度
すべての選挙において共通
「あなたは学校はどこまでいらっしゃいましたか。
(在学中・中退は卒
業とみなす)
」
1. 小・高小・新中卒 2. 旧中・新高卒 3. 新高専・短大・専修学校卒
4. 旧高専大・新大卒 5. 大学院(修・博)卒 6. わからない
年齢
衆議院(2003)
:
「あなたのお年は満でおいくつですか。
」
1. 20~24 歳 2. 25~29 歳 3. 30~39 歳 4. 40~49 歳 5. 50~59 歳
6. 60~69 歳 7. 70~79 歳 8. 80 歳以上
参議院(2004)
:
「あなたのお年は満でおいくつですか。
(具体的に記入)
」
133
地方(2003)
:
「あなたのお年は満でおいくつですか。
(記入) 」
性別
すべての選挙において共通
1. 男 2. 女
●シンタックス
※ 衆議院(2003)と参議院(2004)のシンタックスはほぼ同じである。
また地方(2003)の 4 つの地方選挙のシンタックスもほぼ同じであ
る。したがって本稿では紙面の節約のため、衆議院(2003)と地方
(2003)の知事選のシンタックスのみ掲載する。
衆議院(2003)
*選挙関心度.
fre elecint.
recode elecint (1=3)(2=2)(3=1)(4=0)(5=99) into elecint2.
MISSING VALUES elecint2 (99).
fre elecint2.
cro elecint by elecint2.
*地域愛着度.
fre attached.
recode attached (1=2)(2=1)(3=0)(4=-1)(5=-2)(6=99) into aichaku.
MISSING VALUES aichaku (99).
fre aichaku.
cro attached by aichaku.
*居住年数.
fre resdyear.
recode resdyear (1=1.5)(2=6)(3=14.5)(4=27)(5=99) into liveyear.
MISSING VALUES liveyear (99).
fre liveyear.
cro resdyear by liveyear.
*社会集団加入ダミー.
COMPUTE SG=99.
fre noneorg.
if (noneorg=1) SG=0.
EXECUTE.
fre SG.
if (naorg=1) SG=99.
fre neighb.
134
fre women.
fre yngfire.
fre oldmen.
fre pta.
fre farm.
fre union.
fre prof.
fre religion.
fre dokokai.
fre smoveorg.
if (neighb=1 or women=1 or yngfire=1 or oldmen=1 or pta=1 or
farm=1 or union=1 or prof=1 or religion=1 or dokokai=1 or
smoveorg=1) SG=1.
EXECUTE.
fre SG.
MISSING VALUES SG (99).
fre SG.
list
neighb, women, yngfire, oldmen, pta, farm, union, prof, religion,
dokokai, smoveorg, otherog, noneorg, naorg, SG
/cases from 1 to 200.
*政党支持の強さ.
fre partsupt.
compute PS = 99.
recode partsupt (10=0) into PS.
fre PS.
fre partstrn.
recode partstrn (1=2)(2=1)(3=99) into PS.
MISSING VALUES PS (99).
fre PS.
list
partsupt, partstrn, PS
/CASES from 1 to 200.
*教育程度.
fre educ.
recode educ (1=9)(2=12)(3=14)(4=16)(5=18)(6=99) into edu.
MISSING VALUES edu (99).
fre edu.
cro educ by edu.
135
*年齢.
fre age.
recode age
(1=22)(2=27)(3=34.5)(4=44.5)(5=54.5)(6=64.5)(7=74.5)(8=84.5)
into agegroup.
fre agegroup.
cro age by agegroup.
*性別ダミー.
fre sex.
recode sex (1=0)(2=1) into gender.
fre gender.
cro sex by gender.
*****ライブリー・ポリティクス&インタレスト・ポリティクス*****
***個数指標***
*ライブリー・ポリティクス.
compute LPshugisum = SUM (issue01, issue05, issue07, issue08,
issue09, issue17).
EXECUTE.
fre LPshugisum.
recode issue19 (1=99) into LPshugisum.
recode issue21 (1=99) into LPshugisum.
MISSING VALUES LPshugisum (99).
fre LPshugisum.
*インタレスト・ポリティクス.
compute IPshugisum = SUM (issue02, issue03, issue04, issue10).
EXECUTE.
recode issue19 (1=99) into IPshugisum.
recode issue21 (1=99) into IPshugisum.
MISSING VALUES IPshugisum (99).
fre IPshugisum.
list
issue01, issue05, issue07, issue08, issue09, issue17, issue02, issue03,
issue04, issue10, issue19, issue20, issue21, LPshugisum,
IPshugisum
/cases from 1 to 200.
*重回帰分析.
reg var = elecint2 gender agegroup edu PS SG liveyear aichaku
IPshugisum LPshugisum
/dependent elecint2
/method=enter.
136
***ダミー指標***
*ライブリー・ポリティクス.
compute LPshugidummy = 99.
EXECUTE.
if (issue01=1 or issue05=1 or issue07=1 or issue08=1 or issue09=1 or
issue17=1) LPshugidummy=1.
EXECUTE.
fre LPshugidummy.
if
(issue01=0 and issue05=0 and issue07=0 and issue08=0 and
issue09=0 and issue17=0) LPshugidummy=0.
EXECUTE.
fre LPshugidummy.
if
(issue19=1 or issue21=1) LPshugidummy=99.
MISSING VALUES LPshugidummy (99).
fre LPshugidummy.
*インタレスト・ポリティクス.
compute IPshugidummy=99.
EXECUTE.
if
(issue02=1 or issue03=1 or issue04=1 or issue10=1)
IPshugidummy=1.
EXECUTE.
fre IPshugidummy.
if
(issue02=0 and issue03=0 and issue04=0 and issue10=0)
IPshugidummy=0.
EXECUTE.
fre IPshugidummy.
if
(issue19=1 or issue21=1) IPshugidummy=99.
MISSING VALUES IPshugidummy (99).
fre IPshugidummy.
list
issue01, issue05, issue07, issue08, issue09, issue17, issue02, issue03,
issue04, issue10, issue19, issue20, issue21, LPshugidummy,
IPshugidummy
/cases from 200 to 350.
137
*重回帰分析.
reg var = elecint2 LPshugidummy IPshugidummy aichaku
liveyear SG PS edu agegroup gender
/dependent elecint2
/method=enter.
***個数統一指標***
*ライブリー・ポリティクス.
compute LPshugisum2 = SUM (issue01, issue07, issue08, issue09,
issue17).
EXECUTE.
fre LPshugisum2.
recode issue19 (1=99) into LPshugisum2.
recode issue21 (1=99) into LPshugisum2.
MISSING VALUES LPshugisum2 (99).
fre LPshugisum2.
*インタレスト・ポリティクス.
個数指標と同じ.
list
issue01, issue07, issue08, issue09, issue17, issue02, issue03, issue04,
issue10, issue19, issue20, issue21, LPshugisum2, IPshugisum
/cases from 1 to 200.
*重回帰分析.
reg var = elecint2 gender agegroup edu PS SG liveyear aichaku
IPshugisum LPshugisum2
/dependent elecint2
/method=enter.
地方(2003)の知事選
*選挙関心度.
fre CHIJIINT.
RECODE CHIJIINT (1=3)(2=2)(3=1)(4=0)(5=99) into chijiint2.
MISSING VALUES chijiint2 (99).
fre chijiint2.
cro chijiint by chijiint2.
*地域愛着度.
fre attached.
recode attached (1=2)(2=1)(3=0)(4=-1)(5=-2)(6=99) into
aichaku.
MISSING VALUES aichaku (99).
fre aichaku.
cro attached by aichaku.
138
*居住年数.
fre resdyear.
recode resdyear (1=1.5)(2=6)(3=14.5)(4=27)(5=99) into liveyear.
MISSING VALUES liveyear (99).
fre liveyear.
cro resdyear by liveyear.
*社会集団加入ダミー.
COMPUTE SG=99.
fre noneorg.
if (noneorg=1) SG=0.
EXECUTE.
fre SG.
recode naorg (1=99) into SG.
MISSING VALUES SG (99).
fre SG.
fre neighb.
fre women.
fre yngfire.
fre oldmen.
fre pta.
fre farm.
fre union.
fre prof.
fre religion.
fre dokokai.
fre smoveorg.
if (neighb=1 or women=1 or yngfire=1 or oldmen=1 or pta=1 or
farm=1 or union=1 or prof=1 or religion=1 or dokokai=1 or
smoveorg=1) SG=1.
EXECUTE.
fre SG.
list
neighb, women, yngfire, oldmen, pta, farm, union, prof, religion,
dokokai, smoveorg, otherorg, noneorg, naorg, SG
/cases from 1 to 200.
*政党支持の強さ.
compute PS =99.
fre ptysupt1.
139
recode ptysupt1 (1=2)(2=1)(3=99) into PS.
MISSING VALUES PS (99).
fre PS.
fre ptysupt0.
recode ptysupt0 (9=0) into PS.
fre PS.
list
ptysupt0, ptysupt1, PS
/cases from 1 to 50.
*教育程度.
fre educ.
recode educ (1=9)(2=12)(3=14)(4=16)(5=18)(6=99) into edu.
MISSING VALUES edu (99).
fre edu.
cro educ by edu.
*年齢.
fre age.
*性別ダミー.
fre sex.
recode sex (1=0)(2=1) into gender.
fre gender.
cro sex by gender.
*****ライブリー・ポリティクス&インタレスト・ポリティクス*****
***個数指標***
*ライブリー・ポリティクス.
fre chijis02.
recode chijis02 (1=1)(2=0) into issueC02.
fre issueC02.
fre chijis03.
recode chijis03 (1=1)(2=0) into issueC03.
fre issueC03.
fre chijis04.
recode chijis04 (1=1)(2=0) into issueC04.
fre issueC04.
fre chijis05.
recode chijis05 (1=1)(2=0) into issueC05.
fre issueC05.
fre chijis06.
140
recode chijis06 (1=1)(2=0) into issueC06.
fre issueC06.
fre chijis07.
recode chijis07 (1=1)(2=0) into issueC07.
fre issueC07.
fre chijis08.
recode chijis08 (1=1)(2=0) into issueC08.
fre issueC08.
fre chijis10.
recode chijis10 (1=1)(2=0) into issueC10.
fre issueC10.
compute LPchijisum = SUM (issueC02, issueC03, issueC04,
issueC05, issueC06, issueC07, issueC08, issueC10).
EXECUTE.
fre LPchijisum.
recode chijis14 (1=99) into LPchijisum.
recode chijis16 (1=99) into LPchijisum.
MISSING VALUES LPchijisum (99).
fre LPchijisum.
*インタレスト・ポリティクス.
fre chijis09.
recode chijis09 (1=1)(2=0) into issueC09.
fre issueC09.
fre chijis11.
recode chijis11 (1=1)(2=0) into issueC11.
fre issueC11.
fre chijis12.
recode chijis12 (1=1)(2=0) into issueC12.
fre issueC12.
fre chijis13.
recode chijis13 (1=1)(2=0) into issueC13.
fre issueC13.
compute IPchijisum = SUM (issueC09, issueC11, issueC12,
issueC13).
EXECUTE.
fre IPchijisum.
recode chijis14 (1=99) into IPchijisum.
recode chijis16 (1=99) into IPchijisum.
MISSING VALUES IPchijisum (99).
141
fre IPchijisum.
list
issueC02, issueC03, issueC05, issueC06, issueC10, issueC07,
issueC08, issueC04, issueC09, issueC11, issueC12, issueC13,
chijis14, chijis15, chijis16, LPchijisum, IPchijisum
/cases from 1 to 300.
*重回帰分析.
reg var = chijiint2 gender age edu PS SG liveyear aichaku
IPchijisum LPchijisum
/dependent chijiint2
/method=enter.
***ダミー指標***
*ライブリー・ポリティクス.
compute LPChijidummy = 99.
EXECUTE.
if (chijis02=1 or chijis03=1 or chijis04=1 or chijis05=1 or chijis06=1 or
chijis07=1 or chijis08=1 or chijis10=1) LPChijidummy=1.
EXECUTE.
if (chijis02=2 and chijis03=2 and chijis04=2 and chijis05=2 and
chijis06=2 and chijis07=2 and chijis08=2 and chijis10=2)
LPChijidummy=0.
EXECUTE.
if (chijis14=1 or chijis16=1) LPchijidummy=99.
fre LPchijidummy.
MISSING VALUES LPchijidummy (99).
fre LPchijidummy.
*インタレスト・ポリティクス.
compute IPchijidummy=99.
EXECUTE.
if (chijis09=1 or chijis11=1 or chijis12=1 or chijis13=1)
IPchijidummy=1.
EXECUTE.
fre IPchijidummy.
if (chijis09=2 and chijis11=2 and chijis12=2 and chijis13=2)
IPchijidummy=0.
EXECUTE.
fre IPchijidummy.
if (chijis14=1 or chijis16=1) IPchijidummy=99.
142
MISSING VALUES IPchijidummy (99).
fre IPchijidummy.
list
chijis02, chijis03, chijis04, chijis05, chijis06, chijis07, chijis08,
chijis10, chijis09, chijis11, chijis12, chijis13, chijis14, chijis15,
chijis16, LPchijidummy, IPchijidummy
/cases from 200 to 350.
*重回帰分析.
reg var = chijiint2 PS age aichaku gender edu liveyear SG
LPchijidummy IPchijidummy
/dependent chijiint2
/method=enter.
***個数統一指標***
*ライブリー・ポリティクス.
compute LPchijisum2 = SUM (issueC02, issueC03, issueC05,
issueC07, issueC08).
EXECUTE.
fre LPchijisum2.
recode chijis14 (1=99) into LPchijisum2.
recode chijis16 (1=99) into LPchijisum2.
MISSING VALUES LPchijisum2 (99).
fre LPchijisum2.
*インタレスト・ポリティクス.
個数指標と同じ.
*重回帰分析.
reg var = chijiint2 PS age aichaku gender edu liveyear SG
LPchijisum2 IPchijisum
/dependent chijiint2
/method=enter.
<参考文献>
・明るい選挙推進協会 2012. 『第 17 回 統一地方選挙全国意識調査 ―
―調査結果の概要――』
URL:http://www.akaruisenkyo.or.jp/wp/wp-content/uploads/2012/
07/17toituagiyo.pdf 2013/10/22 参照.
・秋元律郎 1993a. 「現代型無関心」 森岡清美・塩原勉・本間康平編 『新
社会学辞典』 有斐閣 所収.
・秋元律郎 1993b. 「政治的無関心」 森岡清美・塩原勉・本間康平編 『新
社会学辞典』 有斐閣 所収.
・Almond, Gabriel A., and Sidney Verba. 1963.『現代市民の政治文化』
石川一雄・片岡寛光・木村修三・深谷満雄訳 勁草書房 1974.
・Dawson, Richard E., Kenneth Prewitt, and Karen S. Dawson. 1977.
143
『政治的社会化』 加藤秀治郎・中村昭雄・青木英実・永山博之訳 芦
書房 1989.
・五十嵐暁郎 2010. 『日本政治論』 岩波書店.
・五十嵐暁郎, ミランダ・A. シュラーズ 2012. 『女性が政治を変える
とき―議員・市長・知事の経験』 岩波書店.
・蒲島郁夫 1988. 『政治参加』 東京大学出版会.
・角一典 1999. 「日本の地域政治における『新しい政治』
」 『北海道
大學文學部紀要』 48(2): 59-80.
・川上和久 1994. 「若者にとっての政治の意味」 飽戸弘編 『政治行
動の社会心理学』 福村出版 所収.
・京都市選挙管理委員会(蒲島郁夫執筆) 1980. 『京都市民の投票行
動 ―京都府・市議会議員選挙(昭和 54 年 4 月)を素材として―』.
・丸山真男 1954. 「政治的無関心」 政治学事典編集部編 『政治学事
典』 平凡社 所収.
・三宅一郎・木下富雄・間場寿一 1967. 『異なるレベルの選挙におけ
る投票行動の研究』 創文社.
・文部科学省 生涯学習政策局 生涯学習推進課 専修学校教育振興室
2013. 『専修学校』
URL:http://www.mext.go.jp/a_menu/shougai/senshuu/1280727.ht
m 2013/12/2 参照.
・中野康人 2004. 「回帰」 小野寺孝義・山本嘉一郎編 『SPSS 事典
-BASE 編-』 ナカニシヤ出版 所収.
・大矢吉之 1995. 「任意集団」 大矢吉之・加堂裕規・慶野義雄・久保
山和人・富岡宣之『デモクラシーと現代政治――政治を動かすもの』
嵯峨野書院 所収.
・篠原一 1985. 「
『ライブリー・ポリティクス』とは何か」篠原一編『ラ
イブリー・ポリティクス ――生活主体の新しい政治スタイルを求め
て――』 総合労働研究所 所収.
・寺島拓幸 2007.「重回帰分析」村瀬洋一・高田洋・廣瀬毅士編『SPSS
による多変量解析』 オーム社 所収.
・柳井道夫 1993. 「政治意識」 森岡清美・塩原勉・本間康平編 『新
社会学辞典』 有斐閣 所収.
144
10
電子投票に賛成する
有権者とは
―京都府京都市に着目して―
水田 紗希子
1. はじめに
「電子政府の実現」というのは 2000 年以降日本の国家目標である。
この国家目標は具体的に言うと「我が国が 5 年以内に世界最先端の IT
国家になる」というものであり、内閣に設置された IT 戦略本部により
2001 年 1 月に発表された (岩崎 2005, 7)。以降、住民基本台帳ネット
ワークシステムや各申請をネットで行えるようにするなど各地で様々
な取り組みがなされてきた。早稲田大学電子政府・自治体研究所が 2005
年以降毎年行っている電子政府世界ランキングをみても、日本は毎年上
位 10 位以内に入っており、世界的に見ても日本の電子行政としての能
力の高さが見てとれる(早稲田大学, 2008)
。
ここで注目したいのが、第一回電子政府世界ランキングでの調査報告
の中の「電子調達、電子入札、電子申告などは各国で導入が進みつつあ
るが電子投票については国の規模、国情によって様々」という点である
(株式会社日経 BP, 2004)
。電子政府世界ランキング上位である日本で
は、どれぐらい電子投票の導入が進んでいるのだろうか。
前提として、電子投票は一体どのようなものであるか。現在日本では、
金融機関の ATM のような「タッチパネル式」の機器を用いて投票を行
うことを電子投票という。投票所の場所が指定された学校や公民館など
であること・入場券は予め郵送されており、持参する必要があることは
従来の自書式投票と変わりない。電子投票のメリットとしては投開票の
簡素化・無効票の削減、デメリットとしては投票したという実感の無
さ・機器の故障などがあげられる。
国内の自治体単位での電子投票の導入具合をみていこう。電子投票は
2002 年以降現在までに計 23 回 9 つの自治体で行われてきた。その中で
岐阜県可児市・神奈川県海老名市(共に 2003 年)
・宮城県白石市(2004
年)で機器の故障によるトラブルが発生し、選挙の無効や自書式でのや
り直しが行われるなど電子投票そのものの信頼性を損ねる事態が発生
した。その後、これらのトラブルや自治体の財政難などを理由に電子投
票の休止や廃止の条例が制定する自治体が続出してしまったのだ
145
(GMO インターネット株式会社, 2002)
。
加えて、世界的にみると日本の電子投票化は、まだまだ遅れていると
言わざるをえない。IT 産業の盛んなエストニアでは、国政選挙レベルで
もインターネットを用いての電子投票が実施されており、イタリアでも
複数の州でオンライン投票の実験を行っている(DATAHOTEL, 2012)
。
岩崎(2005)の言うとおり、
「日本が電子投票の先進国だとは、お世辞
にもいえない」状況である(岩崎 2005, 19)
。
しかし、そんな逆境にも負けず現在まで継続的に電子投票を行ってい
る自治体も存在する。岡山県新見市・京都府京都市・福島県大玉村・青
森県六戸町の 4 つの自治体である。本稿ではこれらの自治体に焦点を当
てたい。私は、そこに住む有権者は他の自治体の有権者にはない電子投
票に対する強い賛成の気持ちがあるとし、それこそが電子投票を現在ま
で継続出来たカギであると考えている。そして、その賛成の気持ちがど
のような要因によって形成されたのかを検証するのが本稿の目的であ
る。
以上より、本稿では、
「どのような人が電子投票に賛成しているのか」
をリサーチクエスチョンとして設定し、筆者が電子投票への賛成意欲を
増幅させると考えた要因に着目して分析を行う。調査対象は電子投票実
施経験のある 9 つの自治体[岡山県新見市・京都府京都市(東山区・上
京区)
・福島県大玉村・青森県六戸町・三重県四日市市・神奈川県海老
名市・岐阜県可児市・広島県広島市(安芸区)
・宮城県白石市]とする。
本稿の構成は、以下の通りである。まず、第 2 節で扱う 3 つの仮説を
モデル図とともに提示する。第 3 節では分析に用いる 2 つのデ―タ・作
業定義・分析手順を説明する。第 4 節では分析結果を提示し、最後の第
5 節では本稿のまとめを行う。
2. 仮説のメカニズム
分析を行うため、本稿では第1節で挙げたリサーチクエスチョンに対
応する仮説として、3 つの仮説を提示する。それら 3 つの仮説に基づい
た分析をおこなうことで、電子投票に賛成する有権者はどのような人物
であるのかを明らかにする。本節の最後に仮説のモデル図を示す。
2-1. 模擬投票参加仮説
まず 1 つ目は、
「自治体が独自に行う模擬投票を経験した人は、電子
投票に賛成する」という仮説である。電子投票を導入している自治体は、
啓発活動の一環として必ず模擬投票を実施してきている。その方法とし
ては、選挙管理委員会が常設している模擬投票所・成人式などの地域の
イベントでの体験・インターネット上での電子投票体験コンテンツがあ
る。体験コンテンツについては特に利用に制限は無く、年齢・所在地問
わず誰でも無料で電子投票のやり方を知ることができる。ここでの経験
146
を通して、有権者は本投票を行う前に電子投票の手軽さを知ることがで
きるのだ。さらに模擬投票の際、口頭や文書での電子投票についての詳
細な説明もあるため、電子投票のメリットをより理解することが可能に
なる。それにより、電子投票への信頼度が増し、電子投票に賛成する意
識が高まるだろう。
2-2. 機械慣れ仮説
2 つ目の仮説は、
「普段から ATM 等の機械操作に慣れている人は、電
子投票に賛成する」というものである。電子投票に用いる機械は先述の
通りタッチパネル式で、候補者の名前をタッチすることで投票が完了す
る。つまり、日々ATM でお金の決済等を行っており、タッチパネルの
操作に慣れている人なら、さほど抵抗感なく電子投票に臨めるのではな
いだろうか。また、継続自治体はどこも 2 回以上電子投票を実施してき
ているため、電子投票の機器自体にも慣れが生じていると考えられる。
そのため、他の廃止・休止した自治体よりも電子投票に賛成しやすいは
ずである。
2-3. 近い投票所希望仮説
最後に第 3 の仮説として、
「投票所が近くになることを望む人は、電
子投票に賛成する」を挙げる。福元・堀内(2012)が横浜市を対象に行
った研究によると、投票所が遠ければ「投票コスト」の高さから投票に
行かなくなるという。つまり、投票所の近さと投票に行くかどうかには
密接な関係があるといえる(株式会社日経 BP, 2012)
。
投票所の近さと電子投票にはどんな関係があるのか。岩崎(2005)に
よると、2002 年発表された総務省の「電子機器利用による選挙システ
ム研究会」の報告書の中には、電子投票システムには 3 つの段階がある
と書かれている。現在の日本で実施されているのが「選挙人が指定され
た投票所において電子投票機を用いて投票する段階」
(第一段階)であ
る。ちなみに、この報告書では電子投票の第二段階として「指定された
投票所以外の投票所においても投票できる段階」
、第三段階として「投
票所での投票を義務づけず、個人の所有するコンピュータ端末を用いて
投票する段階」が定められているが、なりすましやプライバシー漏洩の
可能性も考えられるため、日本では実施段階には至っていないとのこと
だ(岩崎 2004, 16)
。だが、先に挙げたエストニアでは日本でいうとこ
ろの第三段階が実施されており、日本で実現が不可能とまではいえない
だろう。であれば、投票所が近いことを望む人は今より近くて手軽に投
票が出来る電子投票の導入とそしてその電子投票の更なる発展(第一段
階から第二・第三段階へというような)を望むのではないだろうか。そ
してそういった人々こそ電子投票に賛成するはずである。
147
図 1 仮説のモデル図
模擬投票参加
電子投票への賛成
機械慣れ
近い投票所希望
3. 分析枠組み
3-1. 使用するデータ
分析をおこなうにあたり、本稿では 2 つのデータを用いる。1 つ目は
現日本大学法学部政治経済学科教授 岩崎正洋氏がおこなった「電子投
票に関する有権者意識調査」である。これは現在までおこなわれた電子
投票のうち、東日本大震災直後に青森県六戸町で実施された平成 23 年
度の町議選と平成 24 年度の京都府京都市東山区・上京区の市長選を除
いた計 21 回の電子投票についてのアンケート調査である。分析は電子
投票を継続的に行っている 4 つの自治体[岡山県新見市・京都府京都市・
福島県大玉村・青森県六戸町]と廃止・休止した 5 の自治体「三重県四
日市市・神奈川県海老名市・岐阜県可児市・広島県広島市(安芸区)
・
宮城県白石市」とに分けておこない、前者を「継続自治体」
、後者をま
とめて「廃止自治体」とする。1
2 つ目は京都市選挙管理委員会が実施した「電子投票に関するアンケ
ート」である。京都市は現在まで継続的に電子投票をおこなっている上、
他の自治体と異なり電子投票導入当時の有権者意識調査結果が残って
いた。2 そのため、今回は京都市に着目した分析をおこなう。なお、電
子投票が実施されたのは京都市内の上京区・東山区の 2 区のみで、意識
調査は東山区で 2004 年に、上京区で 2008 年に実施された。
なお、二つのデータについては以下岩崎教授のものを「I データ」
、京
都市のものを「K データ」と表記する。また、仮説 1 については I デー
タ・K データの両方に同じような質問があったためどちらのデータを用
いても検証が可能だが、アンケートの質問項目の都合上、仮説 2 につい
ては I データのみ、仮説 3 については K データのみで検証をおこなう。
3-2. 作業定義
本稿では上記の 2 つのデータを用いて仮説を検証していく。従属変数
は「電子投票への賛成」
、独立変数は仮説 1 では「模擬投票への参加の
有無」
、仮説 2 では「ATM の慣れの程度」
、仮説 3 では「希望する投票
所の場所」である。従属変数にあたる質問は各データから 1 つずつ計 2
つ使用し、独立変数にあたる質問は各データから 2 つずつ計 4 つ使用し
た。詳しくは補遺を参照して頂きたい。
148
<従属変数>
本稿での電子投票への賛成について、K データでは「あなたは、選挙
の際の投票について、これまでのような自書式の投票がいいと思います
か。それとも今回のような電子投票がいいと思いますか」を使用する。
自書式を「反対」
、電子投票を「賛成」とした。I データでは「あなたは、
電子投票機の導入について、賛成ですか反対ですか(○は一つだけ)
」
を使用した。
「賛成」
・
「どちらかというと賛成」を「賛成」
・
「どちらか
というと反対」
・
「反対」を「反対」としてまとめた。どちらも無回答や
「どちらでもない」は欠損値として処理した。
<独立変数>
・模擬投票への参加の有無
I データでは、
「あなたは、今回の選挙に際して、事前に模擬投票所で
の電子投票を体験されましたか」を使用した。
「はい」を「体験した」
、
「いいえ」を「体験しなかった」とした。K データでは「今回の選挙で
は,電子投票の模擬投票の体験コーナーを,上京区(東山区)総合庁舎
内や上京保健所には常設型を設置するとともに,各種の会合などには移
動型を派遣しました。あなたは模擬投票を体験されましたか(○は一つ
だけ)
」を用いた。こちらも「体験した」と「体験しなかった」でわけ
た。どちらも無回答は欠損値として処理した。
・ATM の慣れの程度
K データの「あなたは、機械の操作への慣れは、いかがですか(○は
一つだけ)
」を使用した。
「銀行の ATM(現金自動預け払い機)を自分
で操作できる」を「慣れている」とした。次に「銀行のATM(現金自
動預け払い機)を人に確認しながら操作できる」を「やや慣れている」
とした。そして「銀行の ATM(現金自動預け払い機)の操作に抵抗が
ある」を「慣れていない」とした。無回答は欠損値として処理した。
・希望する投票所の場所
I データの「将来、電子投票が本格的に導入された場合に、あなたは
次にあげるどの場所から投票できるようになったらいいと思いますか。
一つだけあげてください」を使用した。
「現在の投票所」は現状と変化
がないため 2-3 節であげた「第一段階」にあたるとした。
「図書館や病
院などの公共施設」
・
「駅」
・
「コンビニエンスストア」は現在の投票所よ
りも近い場所になることを希望していると考えられるため「第二段階」
にあたるとした。そして「自宅」は自宅の電子機器を用いた投票になる
と考えられるので「第三段階」にあたるとした。無回答や「その他」は
欠損値として処理した。
149
3-3. 分析手順
分析方法はクロス表分析である。まず、仮説 1 の検証のため模擬投票
への参加の有無と電子投票への賛成、次に仮説 2 の検証のため ATM の
慣れの程度と電子投票への賛成、最後に仮説 3 の検証のため希望する投
票所の場所と電子投票への賛成でクロス表分析をおこなった。
4. 分析結果
以下で 3 つ分析の結果を順に示していく。今回は分析の各度数(N)
の割合が 5%未満になるものが非常に多かったため、分析結果の正当性
に疑わしい部分があることだけ注意したい。
4-1. 模擬投票参加仮説
表 1 継続自治体での模擬投票への参加の有無と電子投票への賛成のクロス表分析
(I データ)
電子投票(%)
反対
模擬投票
合計(%)
(N)
賛成
体験していない
7.7
92.3
100
(1355)
体験した
7.6
92.4
100
(1179)
7.7
92.3
100
(2534)
合計
カイニ乗値=.002 危険率=.969
表 2 廃止自治体での模擬投票への参加の有無と電子投票への賛成のクロス表分析
(I データ)
電子投票(%)
反対
模擬投票
合計
合計(%)
(N)
賛成
体験していない
10.8
89.2
100
(1890)
体験した
12.6
87.4
100
(1332)
11.5
88.5
100
(3222)
カイニ乗値=2.531 危険率=.112
150
表 3 上京区における模擬投票への参加の有無と電子投票への賛成のクロス表分析
(K データ)
電子投票(%)
反対
模擬投票
体験していない
体験した
合計
合計(%)
(N)
賛成
10.8
89.2
100
(297)
11.1
88.9
100
(27)
10.8
89.2
100
カイニ乗値 .003
(324)
危険率 =.957
表 4 東山区における模擬投票への参加の有無と電子投票への賛成のクロス表分析
(K データ)
電子投票(%)
反対
模擬投票
合計(%)
(N)
賛成
体験していない
5.8
94.2
100
(276)
体験した
1.7
98.3
100
(59)
5.1
94.9
合計
100
(335)
カイニ乗値 1.698 危険率= .193
表 1 から表 4 まで、有意な結果となったものはなかった。つまり、こ
の仮説は支持されなかったと言える。また、危険率の高低を問わず全体
の結果をみたところ、東山区を除く 3 つの分析では「模擬投票に行った
からこそ電子投票に賛成」または「模擬投票に行かなかったからこそ電
子投票には反対」といった明確な差異は現れなかった。つまり、模擬投
票への参加の有無と電子投票に賛成するかどうかにはそもそも関係が
無い可能性が高そうである。
4-2. 機械慣れ仮説
表 5 上京区における機械慣れの程度と電子投票への賛成のクロス表分析(K データ)
電子投票(%)
反対
機械慣れ
(N)
賛成
慣れていない
29.6
70.4
100
(27)
あまり慣れていない
11.1
88.9
100
(9)
9.2
90.8
100
(284)
慣れている
合計
合計(%)
10.9
151
89.1
100
カイニ乗値=10.611
(320)
危険率=.005
表 6 東山区における機械慣れの程度と電子投票への賛成のクロス表分析(K データ)
電子投票(%)
反対
(N)
賛成
16.0
84.0
100
(25)
あまり慣れていない
7.1
92.9
100
(14)
慣れている
5.1
94.9
100
(295)
6.0
94.0
100
カイニ乗値=4.912
慣れていない
機械慣れ
合計(%)
合計
(334)
危険率=.086
表 5・6 より、上京区のほうは 1%水準で有意、東山区は 10%水準で
有意な結果となった。機械に慣れている人ほど電子投票に賛成し、反対
に機械に慣れていない人ほど電子投票に反対していることも表の数字
からはっきりわかるので、この仮説は支持されたといえる。
4-3. 近い投票所希望仮説
表 7 継続自治体での希望する投票段階と電子投票への賛成のクロス表分析(I データ)
電子投票(%)
反対
投票段階
合計(%)
(N)
賛成
第一段階
8.0
92.0
100
(1350)
第二段階
6.5
93.5
100
(263)
第三段階
7.3
92.7
100
(850)
5.0
78.3
100
(2463)
カイニ乗値=2.11 危険率=.348
合計
表 8 廃止自治体での希望する投票段階と電子投票への賛成のクロス表分析(I データ)
電子投票(%)
反対
投票段階
合計
合計(%)
(N)
賛成
第一段階
12.4
87.6
100
(1569)
第二段階
10.7
89.3
100
(469)
第三段階
9.8
90.2
100
(1117)
11.2
88.8
100
(3155)
カイニ乗値=4.322 危険率=.115
表 7・8 より、分析結果は危険率が高く、有意な結果とはならなかっ
た。しかし危険率の高さをひとまず置いて表をみると、継続・廃止自治
体問わず第一段階(現状通りの投票所)と第三段階(自宅)を希望する
152
有権者の数が第二段階(コンビニエンスストアや公共機関など)と比べ
て 3 倍近く多いことから、第二段階に対してマイナスな印象を多くの有
権者が持っているのではないかと考えた。コンビニエンスストアや駅な
どは不特定多数に自分の投票している所が見られたりする可能性があ
るのではないかと考え、自宅や今の投票所の方が安心だと捉えたのでは
ないか。この仮説は支持されなかったが、有権者が投票に対して楽に投
票できることよりもプライバシー漏洩の防止を重んじているのではな
いか、という新たな仮説を見つけることができた。
5. さいごに
以上の分析により、有意な結果となったものが 2 つ目の分析のみであ
ったことから、本稿の分析で確認された仮説は、
「京都府京都市上京区・
東山区の有権者で普段から ATM 等の機械に慣れている人は電子投票に
賛成する」というものとなった。京都市に焦点を当てたとはいえ、本来
の目的である電子投票を継続している自治体全般を説明することがで
きなかったことは非常に残念である。
第一節でも述べたが、日本は電子投票先進国とは言えない。しかし、
データ収集のため各選挙管理委員会に連絡を取った際、電子投票を廃
止・休止した自治体の中には「機械の安全性が確保されれば是非また復
活させたい」という声や、継続している自治体の中からは「これからも
実施し続けたい」という声を頂いており、決して退行しているわけでは
ない。また、現時点で電子投票は地方選挙のみでの実施であるが、岩崎
(2004)によると 2004 年に超党派の議員連盟である「電子式投開票シ
ステム研究会」が同年 7 月の参議院議員選挙での電子投票の導入に向け
法改正を目指すといった具体的な動きも見られたとのことだ。つまり、
電子投票の将来は決して暗いものではないといえるだろう。
今後の課題として、3 点ほどあげたい。まず、1 点目は機械慣れと電
子投票賛成の関係については機械慣れの程度を大きく左右する可能性
のある年齢という変数を考慮することである。若い人ほど機械慣れし、
高齢の人ほど機械に不慣れのはずなので、この変数を加えた場合、分析
結果はまた違ったものになったと思われる。今回は電子投票の各有効度
数が著しく低いことがほとんどであったため、年齢まで変数として用い
ることができなかった。2 点目はメリット・デメリットを有権者がどれ
ほど認知しているのかも言及することである。これをおこなうことで、
電子投票に賛成する人・反対する人とはどのような人かをより詳しく説
明できたであろう。3 点目は、今回は新たな仮説として提示するのみに
留まった、投票場所の第 2 段階に対する有権者の意識はどのようなもの
かまでしらべることである。特に 2 点目を研究することで、電子投票を
今後新規導入または再開する自治体にとっても啓発活動の方針を決め
る際にも役立つこともあるのではないだろうか。
153
これからの日本における電子投票と電子投票に纏わる研究のさらな
る発展を願いつつ、本稿の終わりとする。
注
(1) 本分析に当たり、現日本大学法学部政治経済学科教授 岩崎正洋氏
よりデータの提供を受けました。このデータ提供して下さった岩崎
教授、調査を行われた当時の杏林大学社会科学部・岩崎研究会所属
学生の方々、そして利用についてご指導くださった同志社大学法学
部の西澤由隆教授には、この場を借りて深く感謝申し上げます。
(2) 本分析に当たり、京都府京都市選挙管理委員会よりデータの提供を
受けました。データ提供して下さった選挙管理委員会の方々とご指
導・ご便宜賜りました西澤由隆教授には、この場を借りて深く感謝
申し上げます。
<補遺>
●電子投票への賛成
K データ質問文:
「あなたは、選挙の際の投票について、これまでのよ
うな自書式の投票がいいと思いますか。それとも今回のような電子
投票がいいと思いますか。
」
K データ質問項目:
「自書式 電子投票」
I データ回答項目:
「あなたは、電子投票機の導入について、賛成です
か反対ですか。
(○は一つだけ)
」
I データ回答項目:
「1,賛成 2,どちらかというと賛成 3,どちらでもな
い 4,どちらかというと反対 5,反対」
●模擬投票への参加
I データ質問文:
「あなたは、今回の選挙に際して、事前に模擬投票所
での電子投票を体験されましたか。
」
I データ回答項目:
「はい いいえ」
K データ質問文:
「今回の選挙では,電子投票の模擬投票の体験コーナ
ーを,上京区(東山区)総合庁舎内や上京保健所には常設型を設置
するとともに,各種の会合などには移動型を派遣しました。あなた
は模擬投票を体験されましたか。
(○は一つだけ)
」
K データ回答項目:
「1,体験した 2,体験しなかった」
●ATM への慣れ
K データ質問文:
「あなたは,機械の操作への慣れは,いかがですか。
(○は一つだけ)
」
K データ回答項目:
「1,銀行の ATM(現金自動預け払い機)を自分で操
作できる 2,銀行のATM(現金自動預け払い機)を人に確認しな
がら操作できる 3,銀行の ATM(現金自動預け払い機)の操作に抵
抗がある」
154
●希望する投票所の場所
I データ質問文:
「将来、電子投票が本格的に導入された場合に、あな
たは次にあげるどの場所から投票できるようになったらいいと思い
ますか。一つだけあげてください。
」
I データ回答項目:
「現在の投票所 図書館や病院などの公共施設 駅
コンビニエンスストア 自宅 その他」
【シンタックス】
・K データ
(従属変数)
compute 電子投票 = Q6.
recode Q6 (1=1)(2=1)(3=6)(4=0)(5=0) into 電子投票.
missing values 電子投票(6).
value labels 電子投票 0'反対' 1'賛成'.
fre 電子投票.
(独立変数)
compute 模擬投票 = Q2.
recode Q2 (1=1)(2=0) into 模擬投票.
missing values 模擬投票(3).
value labels 模擬投票 1'行った' 0'行っていない'.
fre 模擬投票.
compute 機械慣れ= Q9ATM.
recode Q9ATM (1=2)(2=1)(3=0) into 機械慣れ.
missing values 機械慣れ(4).
value labels 機械慣れ 2'慣れている' 1'あまり慣れていない'
0'慣れていない'.
fre 機械慣れ.
(分析①)
crosstabs tables = 模擬投票 BY 電子投票
/cells count row
/statistics=chisq.
(分析②)
crosstabs tables = 機械慣れ BY 電子投票
/cells count row
/statistics=chisq.
・I データ
(従属変数)
compute 電子投票 I =投票方法 .
recode 投票方法 (1=0)(2=1) into 電子投票 I.
missing values 電子投票 I(3).
value labels 電子投票 I 1'賛成' 0'反対'.
fre 電子投票 I.
155
(独立変数)
compute 場所 = 投票場所.
recode 投票場所 (1 = 0)(2 thru 4 = 1)(5=2) into 場所.
missing values 場所(6).
value labels 場所 0'第一段階' 1'第二段階' 2'第三段階'.
fre 場所.
compute 模擬投票 I = 事前体験 .
recode 事前体験 (1=1)(2=0) into 模擬投票 I.
value labels 模擬投票 I 1'行った' 0'行っていない'.
fre 模擬投票 I.
(分析①)
crosstabs tables = 模擬投票 I BY 電子投票 I
/cells count row
/statistics=chisq.
(分析③)
crosstabs tables = 場所 BY 電子投票 I
/cells count row
/statistics=chisq.
<参考文献>
・岩崎正洋 2005. 『電子投票』日本経済評論社.
・岩崎正洋 2009. 『e デモクラシーと電子投票』日本経済評論社.
・蒲島郁夫 1988. 『政治参加』 東京大学出版会.
・電子投票システム調査検討会 2006. 『電子投票システムの向上に向
けた方策の基本的方向』
・早稲田大学 『OBI LAB Homepage』
http://www.e-gov.waseda.ac.jp/ranking_jp.htm
2013 年 11 月 15 日参照.
・GMO インターネット株式会社 『電子投票普及協業組合』
http://evs-j.com/
2013 年 11 月 20 日参照.
・株式会社 日経 BP 『早稲田大学が主要 23 カ国の電子政府ランキン
グを発表、1 位は米国、日本は 7 位でトップ 10 入り』
http://itpro.nikkeibp.co.jp/free/NGT/govtech/20050414/159280/
2013 年 11 月 15 日参照.
・DATAHOTEL 『電子投票が浸透する日はやってくるのか?』
http://wired.jp/2012/11/21/electronic_voting/2/
2013 年 11 月 15 日参照.
・株式会社日経 BP 福元健太郎・堀内勇作 『
「政治不信が高まると投
票率が低くなる」は本当か』
http://business.nikkeibp.co.jp/article/report/20120508/231768/?rt
=nocnt
2013 年 11 月 15 日参照.
156
11
未成年期の擬似的な
政治参加経験の影響
力
―生徒会選挙・模擬選挙に注目して―
山内 逸平
1. はじめに
「若者の投票率が低い」ということは、もはや常識と言えるほど世間
に定着した社会問題であろう。表 1 にもあるように、直近 3 回の衆議院
議員選挙での年代別得票率では、20 歳代の投票率はどの世代の投票率よ
りも低いものとなっている。このような現状から、若年層の政治への無
関心を少しでも改善し投票率を上昇させるべく、各地域の選挙管理委員
会や明るい選挙推進協会などが様々な対策を行っている。
表 1 最近の衆議院議員選挙の年代別投票率
2005 年
2009 年
2012 年
20 歳代
46.20
49.45
37.89
30 歳代
59.79
63.87
50.10
40 歳代
71.94
72.63
59.38
50 歳代
77.86
79.69
68.02
60 歳代
83.08
84.15
74.93
70 歳以上
69.48
71.06
63.30
出所:明るい選挙推進協会(2013)
この問題への取り組みが行われる中で、度々議論になるのが「成人年
齢を引き下げてはどうか」というものだ。これは成人年齢を引き下げる
と同時に、選挙権を付与する年齢をも引き下げることで、少しでも世代
による投票率の隔たりによる世代間格差を解消しようとするものであ
る。これは選挙権を付与する年齢を引き下げることで若年層の有権者数
そのものを増やし、高齢層の人口との差を是正するべく提案されたもの
だ。しかし、成人年齢の引き下げを拙速だとする意見もあり、この議論
157
がどのような結末を迎えるかまだよく分からない。
こうした若者と政治との関わりについて考える際に着目したいのが、
学校教育の中で触れる機会のある擬似的な政治参加経験である。これは
具体的には、特別活動の一つである生徒会活動の一環として行われる生
徒会・児童会選挙や、社会科の授業の中で行われる模擬選挙などを用い
た体験型学習のことを指す。前者は必ずしも政治に対する関心の醸成を
目的として行っているものではない。また、後者は社会科の授業の中で
採られる授業形態の一つに過ぎない。しかしながら、このような活動を
経験することにより、民主主義の本質や自身の一票の価値の重要性に対
する生徒達の理解が促されるのではないだろうか。
本稿では、
「未成年期に擬似的な政治参加経験をしたことが、その後
の政治関心にどのような影響を与えるのか」をリサーチクエスチョンに
設定する。国民投票法の成立により、国民投票ができる条件が「満 18
歳以上の日本国民」とされたことは記憶に新しい。1 また、2000 年代
初頭には市町村合併に関する住民投票の権利を未成年者にも認めると
いう自治体が多数存在していた。2 これらの傾向にも見られるように、
国や自治体の制度として未成年者が政治に関わる門戸は、以前よりも確
実に開かれてきていると言えよう。
このリサーチクエスチョンの解明は、未成年期の擬似的な政治参加経
験の効果を知ることができる点で意義のあることだと言える。この経験
がその後いかに人々の政治参加に影響を及ぼしたのか、また若年層への
効果がどの程度あるのかを明らかにできれば、若年層が政治に無関心で
あり投票率が停滞しているという現状の解決策を示すことができるだ
ろう。また、客観的なデータを用いた分析とその結果を提示することで、
成人年齢を引き下げるべきか否かという議論や政治関心上昇の効果的
な方法は何なのかという疑問にも説得力ある示唆を与えることができ
るだろう。
以上のことを論じるため、本稿では以下の構成をとる。第 2 節では、
本稿の最も重要な概念である擬似的な政治参加経験について具体的に
その詳細を紹介し、この経験と政治関心の関係について述べる。第 3 節
では、第 2 節で取り上げた擬似的な政治経験に則して実際に検証を行う
2 つの仮設を提示し、どのようなメカニズムで仮説が成立するのかを説
明する。第 4 節では、分析で用いるデータと作業定義について言及し、
具体的にどのような手順で分析を行ったのかを説明する。そして、第 5
節において仮説の分析結果とその解釈を示した上で、第 6 節ではその考
察を行う。この分析結果からは新たな謎が生まれた。第 7 節ではその新
たに生まれた謎を解くべく、さらに踏み込んだ分析を行い、その結果と
それが意味することを示した。そして、最後の第 8 節では本研究の総括
と今後の課題について述べる。
158
2. 学校教育の中での擬似的な政治参加経験
以下では、学校教育の中での擬似的な政治参加経験の現状や、その実
態について紹介し、なぜそれが政治関心に影響をもたらすようになるの
かについて説明する。
2-1. 生徒会・児童会活動について
まず、生徒会活動の定義やそれが何を目的として存在しているのかに
注目したい。
生徒会活動は「選挙、議会運営、決定事項の施行、問題の処理など、
社会でなされていること」を「体験的に学習する場所」として想定され
た存在であると考えられている(伊東 2011, 136)
。3 具体的には、大
多数の生徒は代表者を選出し、自分たちの意見を反映させるという経験
をする。そして、選出された側の生徒は意見の集約・調整などを行い、
意見の実現に向けて学校側に働きかけるといった経験をする。また、生
徒会活動の目標としては「生徒会活動を通して、望ましい人間関係を形
成し、集団や社会の一員としてよりよい学校生活づくりに参画し、協力
して諸問題を解決しようとする自主的、実践的な態度を育てる」ことが
学習指導要領に掲げられている(文部科学省 2013)
。生徒会活動に参加
することで、学校という「小さな社会」から実社会にも通用する態度の
獲得を期待していることが、この記述からも窺える。各学校で実施され
ている生徒会活動の内容やその取り組みの度合いは、それぞれで異なる
ことが予想される。しかしながら、いずれの学校も上記の目標を実現す
べく活動を行っていることは間違いないだろう。
ゆえに、こうした活動に関わることは、実社会の一員としての意識を
養い、政治関心を上昇させることにも繋がるのではないかと考えた。な
ぜなら、生徒会・児童会選挙のような形式ではあるものの、学校内で「投
票」の経験をすることが、生徒達に選挙や政治への親近感を抱かせ、選
挙権を得た時に投票を行う際のコストを下げるように作用すると考え
るからである。これは、単なる知識として社会や政治の仕組みを学ぶよ
りも、実際に考えて投票を行った経験の方が記憶に残りやすいからだ。
また、そのような経験があれば、成人し選挙権を得た際に選挙制度の受
容がスムーズに行われるという点でも影響力があると考えるからだ。
2-2. 選挙に関する体験型学習
次に社会科の授業内で行われる体験型学習について注目する。体験型
学習と一口に言っても様々なものがあると言える。具体的には、選挙を
想定して生徒が投票を行うという模擬選挙を行うケース・候補者や政党
の意見を調べさせ発表を行わせるケース・議会や議員事務所の見学を行
うといったフィールドワークを実施するケースなど枚挙に暇がない。
こうした学習形態の中でも特に着目したいのが、模擬選挙である。4
159
なぜならこれは生徒会・児童会活動の中での選挙と同様に、
「投票」と
いう行為を伴うため、選挙についての関心がより向上すると考えるから
である。選挙管理委員会が企画してこうした取り組みを学校に提案する
ことや、選挙に用いる道具を貸し出すことなどで模擬選挙を行う授業の
バックアップを行うこともしているそうだ。朝日新聞によると、都立杉
並総合高校では、区選挙管理委員会の支援の下で、2013 年 7 月に行わ
れた参議院選挙と同様の候補者に投票することや開票作業を体験して
もらうという授業を展開したとのことである(朝日新聞 2013)
。
まだ有権者ではない生徒たちが、本来であれば踏み入れることのでき
ない選挙を未成年のうちに体感できることは、彼らが有権者になった際
に大きな影響を及ぼすに違いない。また授業の中でそういった活動が行
われることで、政治や選挙の知識と関連して記憶に定着し、理解も深ま
る可能性が高い。こうした経緯で政治への関心が醸成される可能性も高
まると言えるだろう。
本節では、学校内で経験し得る擬似的な政治経験について具体的に 2
つの事例に焦点を当ててきた。これは学校内での擬似的な「投票」に過
ぎないが、蒲島(1988)は、投票を「最も多くの市民が参加する政治活
動である」としている(蒲島 1988, 7)
。政治関心が投票行動と大きな関
わりがあることを鑑みると、こうした最も一般的な政治参加の手段に類
似したものを未成年のうちに経験することは、非常に大きな意味合いを
持つと考えた。
3. 仮説およびそのメカニズムと研究意義
2 節で見た学校教育の中での 2 つの事例から、これらの経験の有無が
いかに実際に行われている政治や選挙への関心へと繋がるかが分かる。
この事例を踏まえ、1 節で挙げたリサーチクエスチョンに対応する仮
説として「未成年期に擬似的な政治参加の経験をした人は、政治への関
心が高く積極的に政治参加する」という未成年期政治経験仮説を提示す
る。さらに、2 節でみた 2 つの事例に則した、より具体性のある 2 つの
仮説を提示し検証を行いたい。まず、1 つ目の仮説としては「生徒会選
挙の経験は、政治への関心を高める」という生徒会選挙仮説を挙げる。
そして、2 つ目の仮説としては「授業内での模擬選挙の経験は、政治へ
の関心を高める」という模擬選挙仮説を挙げる。両者ともに未成年期で
の擬似的な政治経験と政治関心の関係を探るものである。以下の図 1 は
そのモデル図を表したものだ。
160
図 1 未成年期の政治参加経験と政治への興味・関心の関係
未成年期の政治参加経験
生徒会選挙
政治関心
政治参加度
模擬選挙
生徒会選挙仮説では、学校内で行われる生徒会選挙に親しんだ経験が
あればあるほど、政治・選挙への関心が高まることを想定している。な
ぜなら、学校生活の中で投票を通じて自身の考えを学校運営に反映させ
るという営みは、まさしく民主主義的な営みであるからだ。自身の考え
が反映されようとされまいと、一票を行使した経験は選挙や政治への関
心の向上に繋がるに違いない。
また、模擬選挙仮説では、社会科の授業内で体験学習の経験がある人
ほど、政治・選挙への関心も高まるということを想定している。政治や
選挙という単元を単なる知識としてではなく、模擬選挙やディベート等
といったよりイメージしやすい教材・手段を用いて受容したことで、扱
われる政治・選挙への印象も大きく変化するはずである。
なお、本研究と同様に生徒会選挙などの学校内での選挙経験、とりわ
け「模擬投票」や未成年期の政治経験・政治的社会化に着目した先行研
究としては、川辺(2008)や川口(2012)がある。川辺(2008)の研
究では JEDS2000 を用いた分析・考察が行われている。これは、2000
年のデータという少し古いものに基づいているため、ここから最新の動
向を把握するのは難しいというのが本音である。また、川口(2012)の
研究では実際の教育現場に密着して展開されている。しかしながら、こ
の研究で使用されているデータは、関西圏のある中学校の生徒会選挙の
ものを用いており、かなり限定的な議論だと言わざるを得ない。
本稿では、先行研究からさらに踏み込んだ、新たな示唆を与えること
ができると言えるだろう。なぜなら、比較的最近である 2009 年に集計
された「若い有権者の意識調査」を用いて分析を行うことができたから
である。また、次節で説明するとおり、若者調査と有権者調査の分析・
比較が可能であった点も意義のあることのように思う。
4. 分析手法
4-1. 仮説の検証に用いる変数および使用するデータ
3 節で示した 2 つの仮説を検証するために、以下のように変数を設定
した。生徒会選挙仮説では独立変数を「生徒会選挙経験の有無」
、従属
変数を「政治関心」
、統制変数を「年齢」とした。また、模擬選挙仮説
では、独立変数を「模擬選挙経験の有無」
、従属変数を「政治関心」
、統
161
制変数を「年齢」とした。
また、これを検証するためのデータとしては、2009 年に行われた若
い有権者の意識調査(第 3 回)を用いる。5 これは調査対象者が満 16
歳から 29 歳までのものと、20 歳以上の人の 2 つの調査から構成されて
いる。筆者が想定していた未成年期の政治経験についての質問が存在し、
なおかつ質問内容は対象者の年齢に関わらず同じであるため、両調査の
分析結果を比較することも可能である。なお、前者を「若者調査」
、後
者を「有権者調査」と呼ぶ。
4-2. 作業定義
<従属変数>
政治への関心
本稿での「政治への関心」は、国や地方の政治にどの程度関心がある
かを訊いている質問を使用する。
「非常に関心がある」
・
「ある程度関心
がある」を「関心あり」とし、これに 1 の値を与えた。また、
「あまり
関心がない」
・
「全然関心がない」を「関心なし」とし、これに 0 の値を
与えた。なお、
「わからない」
・
「無回答」は欠損値として処理をしてい
る。
<独立変数>
未成年期の擬似的な政治経験を代表するものとして、以下の 2 つの変
数を用いるものとする。
生徒会選挙経験の有無
これは仮説 1 で使用する独立変数である。
「高校までの学校で投票に
よる児童会・生徒会選挙の有無」を聞いている質問項目を使用し、小学
校から高校までの期間で、生徒会の選挙を経験したことがあるかどうか
を調べた。なお、
「小・中・高すべてで経験した人」には 3、
「いずれか
の学校で 2 度経験した人」には 2、
「いずれかの学校で 1 度だけ経験し
た人」には 1、
「経験していない人」には 0 の値をそれぞれ与え分析を行
った。
模擬選挙経験の有無
これは仮説 2 で使用する独立変数である。学校教育の中で学んだ政治
や選挙の具体的内容を聞いている質問項目を使用し、学校教育の中で
「ディベートや模擬選挙などの体験型学習」を小学校から高校までの間
で行ったことがあるかどうかで分類を行った。なお、経験している場合
は 1、そうでない場合は 0 の値を与えている。
<統制変数>
以下の統制変数は、
「未成年期の擬似的な政治経験」以外で政治関心
162
に影響を与えうる要因だと考えられるため導入したものだ。
年齢
分析では「年齢」を統制変数として用いた。これは「あなたのお年は
満でおいくつですか」という質問項目を使用している。なお、若者調査
のクロス表分析にある「未成年」とは 16 歳から 19 歳の人を、
「20 代前
半」とは 20 歳から 24 歳の人を、
「20 代後半」とは 25 歳から 29 歳の人
とそれぞれ定義している。
この変数の投入により、加齢による政治関心の高まりを統制すること
が可能である。また、未成年期の経験がどの年代まで有効なのかという
ことを知ることができるのも、これを用いた理由である。
5. 分析結果
3 節で設定した 2 つの仮説と 4 節での作業定義に基づき、生徒会選挙
仮説・模擬選挙仮説ともにクロス表分析と重回帰分析を行った。以下で
はその分析結果を示す。なお、生徒会選挙仮説・模擬選挙仮説の順で分
析結果を提示していく。
163
5-1. 生徒会選挙仮説の結果
表 2 生徒会選挙経験と政治関心のクロス表分析(若者調査)
若者調査(N=1972)
政治への関心
関心なし
関心あり
未成年
合計(%)
N
なし
64.0
36.0
100
25
いずれかであり
48.4
51.6
100
62
小中・中高・小高であり
46.5
53.5
100
245
小中高すべてであり
41.5
100
130
生徒会選挙経験
58.5
χ2乗検定 = 4.449
政治への関心
関心なし
関心あり
危険率 = 0.217
合計(%)
N
なし
60.3
39.7
100
73
いずれかであり
49.2
50.8
100
124
小中・中高・小高であり
35.9
64.1
100
273
小中高すべてであり
37.2
100
223
20代前半 生徒会選挙経験
62.8
χ2乗検定 = 18.850
政治への関心
関心なし
関心あり
危険率 = 0.000
合計(%)
N
なし
47.6
52.4
100
124
いずれかであり
37.0
63.0
100
165
小中・中高・小高であり
36.9
63.1
100
298
小中高すべてであり
30.4
100
230
20代後半 生徒会選挙経験
69.6
χ2乗検定 = 10.214
危険率 = 0.017
表 3 生徒会選挙経験と政治関心の重回帰分析(若者調査)
B
危険率
生徒会選挙経験
0.082
0.000
年齢
0.021
0.000
調整済み R2 乗値 = 0.020
表 2・3 では若者調査における生徒会選挙仮説の分析結果を示してい
る。まず、クロス表分析の結果である表 2 を見て頂きたい。この危険率
に注目すると 20 代前半では有意確率 0.1%水準で有意、20 代後半では
有意確率 5%水準で有意であった。また、未成年では有意ではなかった。
これより、未成年期の生徒会経験は、未成年期では有意ではないが、20
代になると仮説で予想していた通り政治関心に影響を与えると言うこ
164
とができる。特に 20 代前半では、仮説通りの傾向が見られる。しかし
ながら、20 代後半ではその傾向が少し薄れ、生徒会選挙経験に関わらず
政治関心が高いということが読み取れる。
次に、重回帰分析の結果である表 3 を見て頂きたい。生徒会選挙経験・
年齢ともに危険率は 0.1%水準で有意であった。また、B 値から分かる
ように、こちらでも生徒会選挙経験と政治関心の関係は仮説で予測した
通り正の相関関係にあると言える。
表 4 生徒会選挙経験と政治関心のクロス表分析(有権者調査)
有権者調査(N=2193)
政治への関心
関心なし
関心あり
20代
N
なし
50.0
50.0
100
30
いずれかであり
41.4
58.6
100
58
小中・中高・小高であり
44.7
55.3
100
103
小中高すべてであり
25.3
74.7
100
95
生徒会選挙経験
χ2乗検定 = 10.582
政治への関心
関心なし
関心あり
30代
合計(%)
危険率 = 0.014
合計(%)
N
なし
40.0
60.0
100
60
いずれかであり
31.5
68.5
100
73
小中・中高・小高であり
18.8
81.3
100
96
小中高すべてであり
28.4
100
102
生徒会選挙経験
71.6
χ2乗検定 = 8.714
165
危険率 = 0.033
政治への関心
関心なし
関心あり
40代
N
なし
41.3
58.7
100
46
いずれかであり
23.8
76.3
100
80
小中・中高・小高であり
21.1
78.9
100
114
小中高すべてであり
19.8
100
121
生徒会選挙経験
80.2
χ2乗検定 = 9.289
政治への関心
関心なし
関心あり
50代
合計(%)
危険率 = 0.026
合計(%)
N
なし
20.8
79.2
100
77
いずれかであり
16.5
83.5
100
91
小中・中高・小高であり
13.6
86.4
100
132
小中高すべてであり
9.6
100
114
生徒会選挙経験
90.4
χ2乗検定 = 4.983
政治への関心
関心なし
関心あり
危険率 = 0.173
合計(%)
N
なし
12.4
87.6
100
322
いずれかであり
3.6
96.4
100
222
小中・中高・小高であり
5.3
94.7
100
170
小中高すべてであり
5.7
94.3
100
87
60代以上 生徒会選挙経験
χ2乗検定 = 17.113
危険率 = 0.001
表 5 生徒会選挙経験と政治関心の重回帰分析(有権者調査)
B
危険率
生徒会選挙経験
0.078
0.000
年齢
0.013
0.000
調整済み R2 乗値 = 0.093
表 4・5 では有権者調査における生徒会選挙仮説の分析結果を示して
いる。まず、クロス表分析の結果である表 4 を見て頂きたい。危険率に
着目すると、60 代以上が有意確率 0.1%水準で有意、20 代・30 代・40
代が有意確率 5%水準で有意であった。また、50 代でだけ有意な結果は
得られなかった。そして、特に 20 代・30 代・40 代では、仮説で予測し
ていた結果に概ね合致する結果が得られた。しかしながら、60 代以上の
年代では、生徒会選挙経験に関わらず政治への関心が高いことが読み取
れる。
次に重回帰分析の結果である表 5 に注目して頂きたい。生徒会選挙経
験・年齢ともに危険率は 0.1%水準で有意であった。また、B 値からも
分かるように、こちらでも生徒会選挙経験と政治関心の関係は仮説で予
166
測した通り正の相関関係にあると言える。
5-2. 模擬選挙仮説の結果
表 6 模擬選挙経験と政治関心のクロス表分析(若者調査)
若者調査(N=1688)
政治への関心
関心なし
関心あり
未成年
合計(%)
N
なし
47.1
52.9
100
359
あり
32.8
67.2
100
64
模擬選挙経験
χ2乗検定 = 4.466
政治への関心
関心なし
関心あり
20代前半
危険率 = 0.035
合計(%)
N
なし
37.8
62.2
100
513
あり
32.5
67.5
100
83
模擬選挙経験
χ2乗検定 = 0.856
政治への関心
関心なし
関心あり
20代後半
危険率 = 0.355
合計(%)
N
なし
33.3
66.7
100
600
あり
23.2
76.8
100
69
模擬選挙経験
χ2乗検定 = 2.913
危険率 = 0.088
表 7 模擬選挙経験と政治関心の重回帰分析(若者調査)
B
危険率
模擬選挙経験
0.155
0.003
年齢
0.023
0.000
調整済み R2 乗値 = 0.019
表6・7 では若者調査における模擬選挙仮説の分析結果を示している。
まず、クロス表分析の結果である表 6 を見て頂きたい。危険率に注目す
ると未成年は有意確率 5%水準で有意、20 代後半では有意確率 10%水準
で有意であった。また、20 代前半では有意ではなかった。これより、未
成年期の授業内での模擬選挙の経験は、政治関心に対して未成年期と 20
代後半では影響を持つと言うことができる。しかしながら、こちらの仮
説でも 20 代後半になると、模擬選挙が与える影響が少し薄れ、模擬選
挙経験に関わらず政治関心が高いようになることは興味深いことであ
る。
そして次に、重回帰分析の結果である表 7 を見て頂きたい。模擬選挙
経験・年齢の危険率はともに 0.1%水準で有意であった。また B 値が示
167
しているように、模擬選挙経験と政治関心の関係は、予想した通り正の
相関関係にあると言える。
表 8 模擬選挙経験と政治関心のクロス表分析(有権者調査)
有権者調査(N=1688)
政治への関心
関心なし
関心あり
20代
なし
32.7
あり
34.4
67.3
65.6
政治への関心
関心なし
関心あり
なし
27.6
あり
11.1
72.4
88.9
政治への関心
関心なし
関心あり
100
32
危険率 = 0.849
合計(%)
N
100
257
100
9
危険率 = 0.273
合計(%)
N
なし
21.2
78.8
100
283
あり
0
100
100
4
政治への関心
関心なし
関心あり
危険率 = 0.300
合計(%)
N
なし
11.0
89.0
100
299
あり
0
100
100
1
模擬選挙経験
χ2乗検定 = 0.124
政治への関心
関心なし
関心あり
60代以上
202
模擬選挙経験
χ2乗検定 = 1.072
50代
100
模擬選挙経験
χ2乗検定 = 1.201
40代
N
模擬選挙経験
χ2乗検定 = 0.036
30代
合計(%)
危険率 = 0.725
合計(%)
N
なし
3.7
96.3
100
402
あり
0
100
100
4
模擬選挙経験
χ2乗検定 = 0.155
危険率 = 0.694
表 9 模擬選挙経験と政治関心の重回帰分析(有権者調査)
B
危険率
模擬選挙経験
0.151
0.092
年齢
0.014
0.000
調整済み R2 乗値 = 0.109
表 8・9 では有権者調査における模擬選挙仮説の分析結果を示してい
る。まず、クロス表分析の結果である表 8 を見て頂きたい。この表の危
168
険率に注目するといずれの年代でも危険率が高く、有意な結果は得られ
なかった。これは模擬選挙などの体験型の学習をした回答をした人が、
有権者調査の中では極端に少なかったからではないかと思われる。この
結果から、こうした学習活動は 20 代ではかなり定着しているが、それ
以上の年代ではほとんど定着していなかったということも分かる。6
一方で、重回帰分析の結果である表 9 の危険率に着目すると、こちら
は 10%水準で有意であった。また B 値から分かるように、予想した通
り模擬選挙経験と政治関心は正の相関関係にあると言える。
6. 考察
5 節の 2 つの仮説の分析結果からも明らかなように、20 代(とりわけ
20 代前半)では予想した通りの結果が得られた。しかし、それ以上の年
代になるとその傾向は弱まるということが分かった。つまり、
「未成年
期政治経験仮説」は若年層に限って支持されたということである。
また、分析によって新たに明らかになったことが 2 点ある。まず 1 点
目は、政治経験の種類によって政治関心に影響を持つ時期・年代が異な
るということだ。具体的には、模擬選挙の経験は未成年にも有効である。
一方で、生徒会選挙の経験は未成年には有効とは言えず、20 代になると
影響を持つようになることが分析結果から言える。このような違いは、
模擬選挙などの体験型学習の方がより記憶に残りやすく強烈な印象を
与えるために、選挙や政治への関心を即座に上昇させる経験と言えるの
かもしれない。すなわち、疑似的な政治参加経験にも、より効果的なも
のとそうでないものとが存在するということである。
そして 2 点目は、未成年期の疑似的な政治参加経験の影響力が徐々に
弱まるということである。表 2・4 からも分かるように、20 代前半では
生徒会選挙経験が政治関心に与える影響は顕著であるにも関わらず、20
代後半以降から次第にその傾向は弱まるのだ。つまり、未成年期の政治
経験が政治関心へと与える影響は、加齢とともに何らかの要因によって
かき消されてしまうということである。これは新たに生まれた疑問でも
ある。
次節では後者の事実に焦点を当て、この新たな疑問を明らかにするべ
くさらなる分析・考察を行いたい。
7. 追加分析
本節では、20 代前半で顕著であった生徒会選挙の経験が、20 代後半
になると弱まるのはなぜなのかを明らかにする。
その原因は、この時期に個人を取り巻く環境が大きく変化することで、
政治への関心が高まることなのではないかと筆者は考えた。すなわち、
加齢とともに訪れる身の回りの環境の変化により、未成年期の擬似的な
169
政治参加経験の影響力は相対的に低下するということである。ここで言
う「身の回りの環境の変化」とは具体的には、20 代前半から後半という
時期に多くの人が経験するであろう「結婚」と「就職」ではないかと推
測する。前者により、税金・将来のこと・子どものことなどを考える機
会が増え、その結果純粋に政治への関心も増すようになる。また後者に
より、それまでの扶養される立場ではなく、社会の中で働き所得を得る
こととなる。またそれに伴い、様々な団体に加入する機会も増加する。
7-1. 概要
これを明らかにするべく、生徒会選挙仮説でも用いた第 3 回若い有権
者の意識調査の若者調査・有権者調査を用いて分析を行う。分析方法は
重回帰分析を用いるが、生徒会選挙仮説で用いた独立変数に加え、
「就
業の有無」と「結婚の有無」という変数を新たに投入する。
以下は作業定義である。
「就業の有無」の作業定義は、質問項目の「あ
なたは何か仕事をしていますか。
」という項目を用いた。就業している
場合は 1、専業主婦・学生・無職の場合は 0 の値を与え、無回答は欠損
値として扱った。また、
「結婚の有無」の作業定義は、質問項目の「あ
なたは結婚していらっしゃいますか」という項目を用いた。こちらも同
様に、結婚している場合は 1、結婚していない場合は 0 の値を与え、無
回答は欠損値として扱った。
7-2. 追加分析の結果と考察
以下の表 10 が追加分析を行った結果を示している。なお、表中の数
字は、重回帰分析を行い算出されたベータ値を表している。若者調査で
は就業の有無・結婚の有無は有意な結果は得られなかったが、有権者調
査では 5%水準で有意な結果が得られた。
表 10 政治関心と各変数の重回帰分析
若者調査
有権者調査
生徒会選挙経験
*0.105
*0.118
年齢
*0.131
*0.309
就業の有無
-0.041
**0.043
結婚の有無
0.010
**0.057
調整済み R2 乗値
0.021
0.097
*:0.1%水準で有意 **:5%水準で有意
170
まず、それぞれの変数の値に着目したい。就業の有無・結婚の有無の
ベータ値はともに正であるため、20 代以上の有権者の間では、
「就業し
ていると政治関心が高まる」
・
「結婚をしていると政治関心は高まる」と
いうことが読み取れる。しかし、ベータ値を比較して分かるように就業
の有無・結婚の有無の政治関心への影響力は生徒会選挙経験よりも弱い
ものだと言える。
以上の結果から、本節の冒頭で述べた「就業や結婚といった環境の変
化が生徒会選挙の経験の効果を相対的に弱める」という予想は支持され
たとは言えない。就業・結婚どちらも政治関心を改善させる力を持つが、
その影響力は生徒会選挙経験や年齢の影響力に比べれば微々たるもの
である。
また、若者調査での生徒会選挙経験のベータ値は 0.105、年齢のベー
タ値は 0.131 と比較的近い値になっている。しかしながら、有権者調査
での生徒会選挙のベータ値は 0.118、年齢のベータ値は 0.309 と大きく
差が開いている。これは、他の要因の出現により生徒会選挙経験の影響
力が相対的に弱まったということではなく、年齢の影響力がより強まっ
ているということを示している。すなわち、加齢こそが生徒会選挙経験
の効果は弱める原因となっていると言えよう。
8. おわりに
学校内における擬似的な政治参加経験に着目し、分析・考察を行って
きた。仮説を実証する過程で、擬似的な政治経験は政治関心と正の相関
関係にあるということが分かった。すなわち、未成年の時期に選挙に類
似した形で「投票」したことがあれば、政治関心の向上に効果があると
いうことだ。また、追加分析の結果から、加齢により政治関心の向上が
見られ、16 歳から 29 歳の若者に対して生徒会選挙経験はかなり強い影
響力を有しているということも明らかとなった。なお、分析においての
課題としては、学校生活の中で複数回経験できる模擬選挙経験を、どの
時期にどれだけ経験してきたのかをカウントせず、経験したか否かの 2
択でしか分析を行えなかったことが挙げられる。これは、データの制約
に関わることでもあるが、どの時期にどの程度の模擬選挙を経験したか
を分析に投入することができれば、さらに詳しい分析や考察を行えたの
で残念である。
昨今の教育制度や学校を取り巻く環境の変化から、教育現場では様々
なことに柔軟に対応しなければならない状況にあると言える。それは具
体的には、授業時間数の削減・教える内容の増加などである。こうした
状況において、課外活動である生徒会活動や入念な事前準備が必要な模
擬選挙を積極的に行うことは、現場のさらなる負担に繋がり敬遠されて
しまうことなのかもしれない。しかしながら、疑似的な政治参加経験を
少しでも積んだことのある生徒を増やすことで、若年層の政治への関心
171
の低さが改善されることが分かった以上、これらの活動を少しでも経験
した人が増えるように学校・教員側が働きかける必要があるように思う。
学校は生徒たちにとって知識を学ぶだけでなく、社会のことも学ぶ場所
だという認識は共有されて然るべきだろう。
一方で、擬似的な政治参加経験を学校だけに押し付けるのではなく、
社会全体でそうした取り組みに協力することで未成年者達はより幅広
い経験を積むことができるだろう。成人年齢の引き下げを行うことで単
に若年層の母数を増やすことを志向するのではなく、政治関心に直結す
るような経験を積ませるように、社会全体を巻き込んで取り組むことを
促すのも重要ではないだろうか。例えばアメリカでは、大統領選挙の際
に子供が保護者と投票所に行き、子供も投票を行うという「キッズ投票
(Kids Voting)
」という取り組みが行われている州もあるそうだ(横江
2004)
。この取り組みは、子供だけでなく保護者の政治への関心をも向
上させるという効果も期待しているとのことだ。また、ドイツではアメ
リカのこの制度を参考にした「ジュニア選挙(Juniorwahl)
」が 18 歳
以下の未成年者を対象として始められ、各学校で積極的に導入されたそ
うだ(
『未来を拓く模擬選挙』編集委員会 2013)
。こうした海外での事
例をそのまま日本に取り入れることは不可能かもしれないが、これらを
参考にした制度を試験的に導入し、効果を測定してみるといったことが
行われても良いのではないかと思う。
若年層の政治関心をいかに上昇させるかは、今後も重要な課題であり
続けるだろう。これに対する効果的な方法が様々なアプローチで常に模
索されているように思う。しかし、それらが目に見えて効果を持つには
時間もコストもかかるというのが現実である。効果は微々たるものかも
しれないが、長い目で見て取り組みが継続されていく必要があるだろう。
また、先にも述べたが教育機関や選挙管理委員会に全てを委ねるのでは
なく、社会全体でこの課題に向き合う姿勢が重要になるのではないかと
感じる。未成年者・若者と政治意識についてのさらなる詳しいデータの
収集や研究の蓄積と、今後の若年層の政治への関心が少しでも改善され
ることを期待したい。
注
(1) 世界各国の選挙権付与年齢については井田(2003)が詳しい。なお、
この論文でも明治大学の学生へのアンケートを用いた分析から未成
年者の政治意識をいかに上昇させるかが触れられている。
2
( ) 本研究を行う前の段階として、塩沢(2009)の先行研究をもとに、
自治体の合併の際の住民投の権利を未成年にも認めた地域に焦点を
当て、
「未成年期の住民投票というフォーマルな政治参加経験は、政
治参加にどのように影響を持つのか」というリサーチクエスチョン
について研究しようとしていた。しかしながら、時間的な制約から
データを収集することができずに断念し、本研究に変更したという
172
(3)
(4)
(5)
(6)
経緯がある。もしデータ作成というハードルがクリアできれば、非
常に興味深い研究になると思うので、今後誰かが筆者の意思を継い
で研究してくれることを期待したい。
生徒会というフレーズが初めて学習指導要領に登場した 1951 年の
改訂版の学習指導要領によると「生徒は、生徒会活動によって、民
主主義の原理を理解することができ」ると書かれている(伊東 2011,
134)
。ここからもわかるように、そもそも生徒会は「民主主義社会
のトレーニングの場」としての機能を期待されているということも
言える(伊東 2011, 135)
。
授業内でいかにして模擬選挙を行うかは、それぞれの教員の腕に依
るところが大きいが、杉浦(2008)や『未来を拓く模擬』選挙編集
委員会(2013)では事前指導や授業の進め方・学習指導案が詳しく
説明・紹介されている。一方で、教育基本法第 14 条において、
「法
律に定める学校」では「特定の政党を支持し、又はこれに反対する
ための政治教育その他の政治的活動をしてはならない」とされてい
る(電子政府の総合窓口 2013)
。この条件をクリアするため、政党
や候補者に十分考慮して授業を展開する際には細心の注意を払う必
要があると言えるだろう。
本分析に当たり、東京大学社会科学研究所附属社会調査・データア
ーカイブ研究センターSSJ データアーカイブから「若い有権者の意
識調査(第 3 回),2009」
(明るい選挙推進協会)の個票データの提
供を受けました。このデータを寄託され、学生にも利用できるよう
にしてくださった明るい選挙推進協会には、この場を借りて深く感
謝申し上げます。
川口(2012)は未成年模擬選挙推進ネットワークという NPO 法人
代表の林大介氏へのインタビューを行っている。このインタビュー
からも、模擬選挙は戦後から散在的に行われてきたが、2002 年に同
法人が設立され、その後一気に定着するようになったということが
分かる。
<補遺>
分析で使用した質問と回答を以下に示す。
・政治関心
Q4:
「あなたは国や地方の政治にどの程度関心がありますか。1 つ選ん
で番号に○をつけてください。
」
非常に関心がある/ある程度関心がある/あまり関心がない/全然関
心がない/わからない/無回答
・模擬選挙経験
Q18SQ2:
「次のことを学びましたか。あてはまるものを、すべて選ん
で番号に○をつけてください。
」
(高校までの学校の授業で、政治や
選挙のことを学んだ記憶がある人に対しての質問)
国民主権や多数決などの民主主義の基本/選挙区制や選挙権年齢など
の選挙のしくみ/普通選挙権実現の歴史/選挙の意義と投票参加の
重要性/投票所における投票の方法/ディベートや模擬投票などの
体験型学習/その他/わからない
173
・生徒会選挙経験
Q20:
「あなたの高校までの学校では、投票による児童会・生徒会選挙
がありましたか。あてはまるものを。すべて選んで番号に○をつけ
てください。
」
小学校であった/中学校であった/高校であった/ない/わからない
/無回答
・年齢
F2:
「あなたのお年は満でおいくつですか。
」
・就職
F4:
「あなたは何か仕事をしていますか」
仕事をしている/専業主婦/学生/無職
・結婚
F5:
「あなたは結婚していらっしゃいますか。
」
結婚している/結婚していない/無回答
シンタックス
分析でのシンタックスを以下に示す。なお、有権者調査でもほぼ同様の
ことを行っているため、紙面の節約上ここでは若者調査の分析で用
いたシンタックスのみを記すこととする。
【生徒会選挙仮説】
・クロス表分析
fre q20_1. fre q20_2. fre q20_3.
compute 生徒会選挙経験 = q20_1 + q20_2 + q20_3.
value labels 生徒会選挙経験 0'なし' 1'いずれかであり' 2'小中・中高・
小高であり' 3'小中高全てであり'.(q20_1・q20_2・q20_3 のいずれ
も回答している場合は 1、回答していない場合は 0 である。
)
fre 生徒会選挙経験.
fre f2.
recode f2 (16 thru 19=1)(20 thru 24 =2)(25 thru 29 =3)(else =-9).
VARIABLE LABELS f2 '年齢'.
VALUE LABELS f2 1'未成年' 2'20 代前半' 3'20 代後半'.
MISSING VALUES f2 (-9).
fre q4.
recode q4 (1 thru 2 =1) (3 thru 4 =0) (else=-9).
value labels q4 1'関心あり' 2 '関心なし'.
missing values q4 (-9).
crosstabs tables 生徒会選挙経験 by q4 by f2
/cells = count row
/statistics = chisq.
174
・重回帰分析
fre q4.
recode q4 (1=3)(2=2)(3=1)(4=0)(else=-9).
missing values q4(-9).
VALUE LABELS q4 0'全然関心なし' 1'あまり関心なし' 2'ある程度関
心あり' 3'関心あり'.
fre f2.
compute 生徒会選挙経験 = q20_1 + q20_2 + q20_3.
value labels 生徒会選挙経験 0'なし' 1'いずれかであり' 2'小中・中高・
小高であり' 3'小中高全てであり'.
REGRESSION
/MISSING LISTWISE
/STATISTICS COEFF OUTS R ANOVA
/CRITERIA=PIN(.05) POUT(.10)
/NOORIGIN
/DEPENDENT q4
/METHOD=ENTER 生徒会選挙経験 f2.
【模擬選挙仮説】
・クロス表分析
re q18S2_1. fre q18S2_2. fre q18S2_3. fre q18S2_4.
fre q18S2_5. fre q18S2_6.
recode q18S2_6 (1=10) (0=0).
value labels q18S2_6 10'あり' 0 'なし'.
compute 模擬選挙経験 =q18S2_1 + q18S2_2 + q18S2_3 + q18S2_4
+ q18S2_5 +q18S2_6.
VALUE LABELS 模擬選挙経験 0 'なし' 1 'なし 2' 2 'なし 3' 3 'なし 4' 4
'なし'5 5 'なし 6' 10'あり 1' 11 'あり 2' 12 'あり 3' 13 'あり 4' 14'あり
5' 15 'あり 6'.
recode 模擬選挙経験 (0 thru 5 =0) (10 thru highest =1).
value labels 模擬選挙経験 1'あり' 0 'なし'.
fre 模擬選挙経験.
fre f2.
recode f2 (16 thru 19=1)(20 thru 24 =2)(25 thru 29 =3)(else =-9).
VARIABLE LABELS f2 '年齢'.
VALUE LABELS f2 1'未成年' 2'20 代前半' 3'20 代後半'.
MISSING VALUES f2 (-9).
fre q4.
recode q4 (1 thru 2 =1) (3 thru 4 =0) (else=-9).
value labels q4 1'関心あり' 2 '関心なし'.
175
missing values q4 (-9).
crosstabs tables 模擬選挙経験 by q4 by f2
/cells = count row
/statistics = chisq.
・重回帰分析
fre q4.
recode q4 (1=3)(2=2)(3=1)(4=0)(else=-9).
missing values q4 (-9).
VALUE LABELS q4 0'全然関心なし' 1'あまり関心なし' 2'ある程度関
心あり' 3'関心あり'.
fre f2.
fre q18S2_1. fre q18S2_2. fre q18S2_3. fre q18S2_4.
fre q18S2_5. fre q18S2_6.
recode q18S2_6 (1 = 10) (0 = 0).
value labels q18S2_6 10'あり' 0 'なし'.
compute 模擬選挙経験 =q18S2_1 + q18S2_2 + q18S2_3 + q18S2_4 +
q18S2_5 +q18S2_6.
VALUE LABELS 模擬選挙経験 0 'なし' 1 'なし 2' 2 'なし 3' 3 'なし 4' 4
'なし'5 5 'なし 6' 10'あり 1' 11 'あり 2' 12 'あり 3' 13 'あり 4' 14'あり
5' 15 'あり 6'.
fre 模擬選挙経験.
recode 模擬選挙経験 (0 thru 5 = 0) (10 thru highest = 1).
value labels 模擬選挙経験 1'あり' 0 'なし'.
fre 模擬選挙経験.
REGRESSION
/MISSING LISTWISE
/STATISTICS COEFF OUTS R ANOVA
/CRITERIA=PIN(.05) POUT(.10)
/NOORIGIN
/DEPENDENT q4
/METHOD=ENTER 模擬選挙経験 f2.
【追加分析】
fre f2.
fre f5.
recode f5 (1=1)(2=0).
MISSING VALUES f5(9).
176
fre f4.
recode f4 (1 = 1)(2 thru 4 = 0)(9 = 9).
MISSING VALUES f4(9).
VALUE LABELS f4 1'職あり' 0'職なし'.
fre q4.
recode q4 (1 = 3)(2 = 2)(3 = 1)(4 = 0)(else = -9).
missing values q4(-9).
VALUE LABELS q4 0'全然関心なし' 1'あまり関心なし'
2'ある程度関心あり' 3'関心あり'.
compute 生徒会選挙経験 = q20_1 + q20_2 + q20_3.
value labels 生徒会選挙経験 0'なし' 1'いずれかであり'
2'小中・中高・小高であり' 3'小中高全てであり'.
REGRESSION
/MISSING LISTWISE
/STATISTICS COEFF OUTS R ANOVA
/CRITERIA=PIN(.05) POUT(.10)
/NOORIGIN
/DEPENDENT q4
/METHOD=ENTER 生徒会選挙経験 f2 f4 f5.
<参考文献>
・朝日新聞 2013 年 9 月 3 日 『高校生、リアルな模擬参院選』
・明るい選挙推進協会編 2013.
URL:http://www.akaruisenkyo.or.jp/070various/072sangi/679/
2013 年 10 月 26 日参照.
・伊東毅 2011. 『未来の教師におくる特別活動論』 武蔵野美術大学出
版局.
・井田正道 2003. 「18 歳選挙権に関する考察」. 明治大学 『政経論
叢』,71 号, 141-165.
・蒲島郁夫 1988. 『政治参加』 東京大学出版会.
・川辺悠子 2008. 『未成年期における政治的社会化の再検討
JEDS2000 のデータ分析』 同志社大学法学研究科 学位論文.
・川口あずさ 2012. 『有権者教育 模擬選挙の効果』 同志社大学法学
研究科 学位論文.
・塩沢健一 2009. 「市町村合併をめぐる住民投票 ―若年層・未成年者
の投票参加―」 『総合政策研究』中央大学総合政策学部創立 15 周
年記念特別号 183-201.
・杉浦真理 2008. 『主権者を育てる模擬投票 ―新しいシティズンシッ
プ教育を目指して―』 きょういくネット.
・電子政府の総合窓口編 2013.
URL:http://law.e-gov.go.jp/htmldata/H18/H18HO120.html
2013 年 11 月 15 日参照.
・
『未来を拓く模擬選挙』編集委員会編 2013. 『実践シティズンシップ
教育 未来を拓く模擬選挙』悠光堂.
177
・文部科学省編 2013.
URL:http://www.mext.go.jp/a_menu/shotou/new-cs/youryou/chu/t
oku.htm#seitokai
2013 年 11 月 7 日参照.
・横江公美 2004. 『判断力はどうすれば身につくのか アメリカの有権
者教育レポート』 PHP 研究所.
178
多岐亡羊 Vol.17
2014 年 3 月 20 日
2013 年度西澤ゼミ学生論文集
初版
著 者 2013 年度西澤ゼミ生
発行者 西澤由隆
発行所 同志社大学法学部・政治学科 西澤由隆研究室
〒602-8580 京都市上京区今出川通東入る
TEL: 075-251-3597
E-mail: [email protected]
HP: http://ynishiza.doshisha.ac.jp/
製本所 ナカバヤシ株式会社
Copyright © 2014 Nishizawa Seminar Printed in Japan